Está en la página 1de 100

Universidad La Salle.

Facultad Mexicana de Medicina.


Curso de Extensión Universitaria para la Preparación del Examen Nacional para
Aspirantes a Residencias Médicas.
Examen Módulo II de Pediatría.
Modalidad a Distancia.

1.- Un niño de 7 meses presenta fiebre alta desde hace 3 días, acompañada de hiperemia
faríngea. La fiebre cede al cuarto día de la enfermedad, momento en que aparece un
exantema morbiliforme, que desaparece en un plazo de 3 días. El diagnóstico más probable
es:

a) Rubéola.
b) Reacción medicamentosa a antitérmicos.
c) Infección por enterovirus.
d) Infección por herpes virus humano 6.

Nos encontramos ante un caso de exantema súbito tópico (también llamado roséola infantil
o 6ª enfermedad).
Se trata de una enfermedad exantemática, de mayor incidencia entre los 6 y 12 meses
(como el caso de la pregunta) y que se presenta con una frecuencia del 90% antes de los
años de vida. No muestra preferencia por ningún sexo y es algo más frecuente al final de la
primavera y principios del verano.
En el 80-92% de los casos de exantema súbito el agente etiológico es el Herpes Virus
Humano.
Lo más característico de este período febril es la ausencia de hallazgos físicos suficientes
para explicar la fiebre, así como el buen estado general del lactante a pesar de la misma.
Al tercer o cuarto día cede la fiebre y aparece una erupción maculosa o maculopapulosa que
comienza en tronco y se extiende a los brazos y al cuello. También afecta algo a piernas y
cara. La erupción desaparece en unos 3 días, normalmente no dejando descamación ni
despigmentación residual.

El diagnóstico de este proceso es fundamentalmente clínico, por la edad y la secuencia de


las manifestaciones clínicas.
Es característico una BHC realizada en 24 -36h puede mostrar leucocitosis con neutrofilia,
pero más allá de 48h aparece el patrón típico d leucopenia con neutropenia absoluta y
linfocitosis relativa.
Nelson, Tratado de Pediatría 15ª Ed., págs. 1120-21
2.- Un niño de 10 años acude a su pediatra con fiebre e inflamación dolorosa de la rodilla
izquierda. Hace 2 semanas tenía la garganta irritada, y un cultivo fue positivo para
estreptococo hemolítico. El niño se trató desde entonces con un ciclo de 3 días de
penicilina oral. ¿Cuál de las siguientes ofrece la mejor evidencia, por si sola, del diagnóstico
de fiebre reumática aguda?

a) Velocidad de sedimentación eritrocitaria elevada.


b) Alta valoración de anticuerpos estreptocócicos.
c) Soplo sistólico apical.
d) Intervalo PR prolongado visible en el electrocardiograma.

La carditis es el síntoma diferenciador de fiebre reumática, y un soplo sistólico apical, es


el hallazgo inicial más común. Los demás casos enumerados en la lista son propios de
pacientes con fiebre reumática, pero también se encuentran en otros trastornos.
Manifestaciones Mayores
Carditis
Poliartritis
Corea (Sydenham)
Nódulos subcutáneos
Eritema marginado
Manifestaciones Menores
Clínicas
Artralgias
Fiebre
Antecedentes de brote
Reumático
Laboratorio y Gabinete
Elevación de reactantes de fase aguda
Prolongación del intervalo PR
Evidencia de infección
Estreptocócica (Grupo A):

Antiestreptolisinas
Exudado faríngeo

Referencia:

PAC. Libro 1, Parte AFiebre ReumáticaPag. 3,7.


Dr. José F. Guadalajara Boo.

• Jefe de Servicio Clínico. Instituto Nacional de Cardiología Ignacio Chávez.


• Miembro numerario de la Academia Nacional de Medicina.
• Gobernador por México del american College of Cardiology.
• Presidente de la Sociedad Mexicana de Cardiología.
3.- Un niño de 2 años acude al centro de urgencias con letargo después de haber estado
jugando sin vigilancia. El niño hasta entonces estaba sano, con un desarrollo normal. En la
historia familiar aparece como dato importante que uno de sus hermanos tiene epilepsia y
toma fenobarbital. La exploración revela un niño letárgico sin fiebre que abre brevemente
los ojos ante estímulos dolorosos. No hay signos de traumatismo ni anomalías focales. Las
pupilas son pequeñas pero reactivas. ¿Cuál es la etiología más probable para el letargo del
niño?

a) Convulsiones no observadas.
b) Intoxicación.
c) Hemorragia intracraneal.
d) Traumatismo craneoencefálico no observado. .

La causa mas común de inicio agudo de somnolencia en un niño que empieza a caminar, es la
intoxicación. La presencia de agentes tóxicos potenciales en la casa es importante en la
historia clínica. La miosis también sugiere intoxicación y la falta de signos de traumatismo
o anomalías focales hace que una hemorragia sea poco probable.

LECTURA RECOMENDADA
PROGRAMA DE ACTUALIZACION CONTINUA EN PEDIATRIA
INTOXICACIONES EN PEDIATRÍA
Dr. Miguel Angel Montoya Cabrera.
Jefe del Departamento de Admisión Continua y Toxicología, Hospital de Pediatría, Centro
Médico Nacional Siglo XXI, Instituto Mexicano del Seguro Social. Miembro de la Academia
Nacional de Medicina, de la Academia Mexicana de Cirugía y de la Academia Mexicana de
Pediatría.

4.- Un niño de 5 años no inmunizado acude a consulta con una historia de 2 semanas de
evolución de tos paroxística, fiebre de bajo grado, emesis post-tos y descarga nasal
viscosa. El EF revela otitis media bilateral y conjuntivitis hemorrágica. Se auscultan
estertores inspiratorios bilateralmente. BH con LT 45 000, con 95% de linfocitos. ¿Cuál de
los siguientes es el diagnóstico más probable?

a) Neumonía por Chlamydia.


b) Tosferina.
c) Bronquiolitis.
d) Neumonitis por VSR.

La tos ferina es una enfermedad infecciosa altamente contagiosa causad por un bacilo gram
(-) llamado Bordetella pertussis.
El comienzo suele ser insidioso con una fase catarral, con tos irritante que poco a poco se
vuelve paroxística, por lo regular en el término de una a dos semanas, y que dura de uno a
dos meses o más. Los paroxismos se caracterizan por accesos repetidos y violentos de tos;
cada serie de ellos comprende innumerables toses sin inspiración intermedia y puede ser
seguida por un estridor respiratorio de tono alto característico. Los paroxismos con
frecuencia culminan con la expulsión de mucosidades claras y adherentes, a menudo seguida
de vómito. Los lactantes menores de 6 meses de edad, los adolescentes y los adultos
frecuentemente no tienen el cuadro típico de estridores o tos paroxística.
Resumen:
Etiología: Bordetella pertussis.
Otros: Bordetella parapertussis, Bordetella bronchiseptica, Mycoplasma pneumoniae,
Chlamydia trachomatis, Chlamydia pnuemoniae y adenovirus.
Fuentes de contagio adolescentes y adultos.
Infección intradocimiciliaria en el 80 % de los no vacunados.
Contagio: Fase catarral y hasta 2 semanas de iniciada la tos.
Periodo de incubación de 10 a 21 días.
Cultivo nasofaríngeo Bordet-Gengou (Dacrón o alginato de calcio).
Negativo en fase temprana, > 4 semanas sin estaba vacunada y > 5 días si recibió
tratamiento.
Leucocitosis con linfocitosis absoluta.

Tratamiento:
• Apoyo: Apnea, hipoxia y otras complicaciones.
• Antibiótico en fase catarral es efectivo.
• Frenar la propagación del germen.
• Eritromicina 40-50 mgkgd cada 6 hrs. x 14 días.
• Claritromicina 15-20 mgkgd cada 12hrs. x 7 días.
• Azitromicina 10-12 mgkgd cada 24 hrs. x 5 días.
• TMP SMZ 8 mgkgd cada 12 hrs. x 14 días.

Bibliografía
• Abul K. Abbas, Andrew H. Lichtman, Jordan S. Pober. Inmunología Celular y
Molecular. McGrall-Hill Interamericana. Cuarta edición 2001.
• Napoleón González Saldaña y Mercedes Macías Parra. Vacunas en Pediatría.
McGrall-Hill Interamericana. Primera edición 1999.
• Report of the Committee on Infectious Diseases. American Academy of Pediatrics.
Red Book 2000. 25 edición 2000.
• Stanley A. Plotkin, Walter A. Orenstein. Vaccines. W. S. Saunders Company.
Tercera edición 1999.
5.- Un lactante varón de 4 semanas de vida acude a consulta inicialmente con una historia
de 2 semanas de vómitos crecientes y escasa ganancia ponderal. Los vómitos, a menudo
proyectados, han persistido pese a que se ha cambiado varias veces de fórmula. En las
últimas 24 horas tiene vómitos en posos de café. Las heces han sido firmes y el niño defeca
un día si y otro no. La exploración física revela un lactante delgado e irritable con abdomen
plano. En el hipocondrio derecho se nota una masa firme en forma de aceituna. ¿Cuál de los
siguientes estudios de laboratorio debe acometerse primero?

a) Radiografía de abdomen.
b) Ecografía abdominal.
c) Valoración de electrólitos en suero.
d) Recuento sanguíneo completo.

La historia y la exploración física son típicas de un diagnostico de estenosis hipertrófica


del piloro. En este caso, la consideración mas importante es el estado hidroelectrolítico del
paciente, ya que en este trastorno pueden observarse anomalías graves. Es más común una
alcalosis metabólica hipoclorémica, y la terapia inicial anterior a la corrección quirúrgica,
debe incluir normalización de los electrolitos del suero. En muchos lactantes con estenosis
Pilarica se observa hematemesis, normalmente como resultado de una gastritis superficial.
Sin embargo, rara vez es de consecuencia hemodinámica. La confirmación del diagnóstico
puede realizarse fácilmente en la mayoría de los casos mediante ecografía de abdomen,
que revela un canal pilórico alargado y engrosado. La radiografía de abdomen puede
mostrar un estomago distendido y lleno de aire. La radiografía con contraste revela un
canal pilórico alargado con retraso del vaciamiento gástrico. Un recuento sanguíneo
completo no tiene ningún valor en el diagnóstico de la estenosis Pilórica.

LECTURA RECOMENDADA
Acta Pediatr Mex 2010;31(2):50-54 INP
Estenosis hipertrófica del píloro. Estudio clínico-epidemiológico
Dr. Carlos Baeza-Herrera,* Dra. Alín Villalobos-Castillejos,** Dr. Arturo Arcos-Aponte,***
Dr. Javier López-Castellanos,**** Dr. Luis Manuel García-Cabello*****

6.- Recién nacido de 6 días, producto de 36 semanas de gestación, que desde el tercer día
presenta una ictericia que ha ido en aumento. La madre es primigesta, tiene un grupo
sanguíneo A (Rh negativo) y el niño es O (Rh positivo). El 6o día tiene una bilirrubina total
de 12.5 mg/dL, detectándola en fracción indirecta. El recién nacido presenta con buen
estado general y los valores de hematocrito, hemoglobina y reticulocitos son normales. ¿En
qué causa de hiperbilirrubinemia, debemos sospechar?

a) Hepatitis neonatal.
b) Enfermedad hemolítica Rh.
c) Ictericia fisiológica.
d) Atresia de vías biliares.
Ictericia fisiológica. Ictericia monosintomática de inicio a partir del segundo día de vida,
con un pico máximo de B de 12-15 MG/Dl. en el 3°-5° día, no persistiendo mas allá del 7día.
No requiere tratamiento pero sí observación y seguimiento por si se tratase de una
ictericia patológica. Se debe a una limitación del hígado para metabolizar el exceso de B
producida en los primeros días de vida.

Metabolismo de la bilirrubina e ictericia fisiológica.

Durante la etapa fetal, la mayor parte de la bilirrubina atraviesa la placenta y es


metabolizada por el hígado materno. En el momento del nacimiento este proceso se corta
bruscamente y al igual que lo que ocurre con otros procesos fisiológicos, debe ser asumida
por los órganos y sistemas del recién nacido los cuales deben adaptarse y madurar para
asumir el proceso en forma eficiente. La Figura 7.1 muestra las distintas etapas del
metabolismo de la bilirrubina. Ellas nos permitirán comprender las causas que llevan a la
elevación de la bilirrubina sérica en los primeros días de vida.

Metabolismo de la Bilirrubina

Producción de la bilirrubina: En el período neonatal, la producción de bilirrubina está


aumentada debido a la alta masa eritrocitaria del neonato (hematocritos de 61% ± 7,4) y a
que la vida media del glóbulo rojo es más corta, alrededor de 90 días comparada con 120
días en el adulto.

Transporte de la bilirrubina: La bilirrubina no conjugada (BNC) circula en el plasma unida


a la albúmina. Normalmente en estas condiciones no atraviesa la barrera hematoencefálica.
Puede aparecer BNC libre (no unida a la albúmina) en condiciones en que la cantidad de
bilirrubina supera la capacidad de unión de la albúmina. Esto puede ocurrir porque hay
cifras muy altas de bilirrubina, hipoalbuminemia o presencia de substancias y factores que
desplazan o debilitan la unión de la bilirrubina con la albúmina. La presencia de BNC libre es
siempre anormal y resulta en paso de esta al SNC y eventual daño del cerebro

Captación, conjugación y excreción hepáticas: La bilirrubina es captada por receptores


específicos del polo sinusoidal del hepatocito y luego transportada por proteínas
específicas al retículo endoplasma. Aquí es conjugada con ácido glucurónico por acción de la
enzima glucuronil transferasa. Se obtiene así la llamada bilirrubina conjugada (BC) que se
caracteriza por ser soluble en agua y no difundir a través de las membranas celulares. La
actividad de la glucuronil transferasa es mas baja en los primeros días de vida El principal
estímulo para aumentar su actividad son los niveles séricos de bilirrubina. También puede
ser estimulada con fenobarbital.

La excreción de la BC es un proceso de transporte activo a través de la membrana del


hepatocito hacia los canalículos biliares. Luego es transportada como un componente de la
bilis al intestino.

Circulación enterohepática de la bilirrubina: La BC que llega al duodeno es en parte


eliminada por las deposiciones, previa transformación en urobilinógeno y similares, por la
acción de las bacterias y en parte reabsorbida pasando nuevamente a la circulación, luego
de haber sido desconjugada del ácido glucurónico en el intestino por acción de la enzima
betaglucuronidasa. En el recién nacido hay varios factores que favorecen la reabsorción
intestinal de la bilirrubina, especialmente en los primeros días de vida:

• Ausencia de bacterias.
• Menor movilidad especialmente si hay ayuno.
• Niveles altos de betaglucuronidasa, enzima que hidroliza la BC en BNC, la cual
puede ser reabsorbida.

En resumen los principales factores que explican la ictericia fisiológica del recién nacido
son:

• Aumento de la producción de la bilirrubina.


• Limitaciones en la captación y conjugación de la bilirrubina.
• Aumento de la reabsorción intestinal de la BNC.

Bibliografía

1. AAP Subcommittee on Neonatal Hyperbilirubinemia. Neonatal jaundice and


kernicterus. Pediatrics 2001 Sep;108(3):763-5.
2. American Academy of Pediatrics. Practice Parametres: Management of
Hiperbilirubinemia in the Healthy Term Newborn. Pediatrics 1994; 94; 558-565.
3. Bezerra JA, Balistreri WF. Cholestatic syndromes of infancy and childhood. Semin
Gastrointest Dis 2001 Apr;12(2):54-65.
4. Harris MC, Bernbaum JC, Polin JR, Zimmerman R, Polin RA. Developmental follow-up
of breastfed term and near-term infants with marked hyperbilirubinemia,
Pediatrics 2001 May;107(5):1075-80.
5. Connolly AM y Volpe JJ: Clincal Features of Bilirubin Encephalopathy. Clin Perinatol
1990; 17: 371-379.
6. Gartner LM, Herschel M. Jaundice and breastfeeding.Pediatr Clin North Am. 2001
Apr;48(2):389-99.
7. Geiger AM, Petitti DB, Yao JF Rehospitalisation for neonatal jaundice: risk factors
and outcomes. Paediatr Perinat Epidemiol 2001 Oct;15(4):352-8.
8. Kemper K, Forsyth B, McCarthy P: Jaundice, terminating breast-feeding, and the
vulnerable child. Pediatrics. 1989; 84; 773-778.
9. Kramer Ll: Advancement of dermal icterus in the jaundiced newborn. AJDC. 1969;
118: 454.
10. Hansen TW. Bilirubin brain toxicity. J Perinatol 2001 Dec;21 Suppl 1:S48-51.

7.- Madre primigesta de 28 años con control prenatal regular, cursa con ruptura de
membranas a las 30 semanas de gestación, se interrumpe el embarazo por cesárea.
Es probable que el recién nacido prematuro disminuya la posibilidad de presentar
enfermedad de membranas hialinas por la siguiente medicación materna:

a) Oxitocina IV.
b) Sulfato de magnesio IV.
c) Antibioticos IV.
d) Esteroides IM.

Uso prenatal, un pilar en neonatología

Por su efecto en la maduración fetal, los corticoesteroides se han utilizado


prenatalmente desde hace más de tres décadas. Liggins y Howie en 1972, reportaron por
primera vez los efectos benéficos de los esteroides, administrados prenatalmente, en la
maduración pulmonar y en la disminución de la incidencia de SDR. Desde ese primer
reporte a la fecha, se han descrito otros efectos positivos. Su uso "adecuado" y oportuno
ha demostrado tener beneficio en el recién nacido pretérmino.Ahora se sabe que la
administración de esteroides a la madre embarazada se ha asociado a disminución en la
incidencia de SDR, hemorragia intraventricular, displasia bronco–pulmonar y mortalidad
neonatal. Se ha observado que el máximo beneficio en el recién nacido se obtiene de 24
horas a siete días después de la administración materna de esteroides. Sin embargo, aún
antes de 24 horas de administrados, se ha reportado beneficio. Aunque los cambios
bioquímicos generados con los esteroides desaparecen después de siete días, los cambios
estructurales persisten. Algunos estudios clínicos han demostrado efecto benéfico de
esteroides aún después de siete días de administrados.

Un metanálisis de 12 estudios, realizado por Crowley y col. en 1990, demostró que el uso
prenatal de esteroides disminuía la incidencia de SDR en aproximadamente 50%. Es claro
que el efecto más importante se encuentra en los fetos menores de 34 semanas de edad
gestacional; sin embargo, también se ha demostrado disminución en la incidencia de SDR
en neonatos de mayor edad gestacional.18

Probablemente el efecto benéfico más importante asociado al uso prenatal de esteroides


sea la disminución en la mortalidad neonatal. Crowley y col. demostraron en el metanálisis,
con más de 3 000 neonatos, que ésta disminuía considerablemente (razón de momios 0.59,
intervalo de confianza =0.47–0.75). La disminución en mortalidad se ha detectado aún en
pacientes menores de 800 g.

Referencias:

1. Bol. Med. Hosp. Infant. Mex. v.62 n.5 México sep. /oct. 2005
2. Esteroides en Neonatología: entusiasmo, uso, abuso y desuso.
¿Dónde está el justo medio?
3. Steroids in neonatology: enthusiasm, use, abuse and disuse. Is there
an equilibrium?
4. Liggins GC, Howie RN.A controlled trial of antepartum glucocorticoid treatment
for prevention of the respiratory distress syndrome in premature infants.
Pediatrics. 1972; 50: 515–25. [
5. Crowley P, Chalmers I, Keirse MJNC. The effects of corticosteroid
administration before preterm delivery:An overview of the evidence from
controlled trials. Br J Obstet Gynaecol. 1990; 97: 11–25. [
6. Van Marter LJ, Leviton A, Kuban KCK. Maternal glucocorticoid therapy and
reduced risk of bronchopulmonary dysplasia. Pediatrics. 1990; 86: 331 –6.
7. Morales WJ, Diebel D, Lazar AJ, Zadrozny D.The effect of antenatal
dexamethasone administration on the prevention of respiratory distress
syndrome in preterm gestations with premature rupture of membranes. Am J
Obstet Gynecol 1986; 154:591–5.
8. Gamsu HR, Mullinger BM, Donnai P, Dash CH. Antenatal administration of
betamethasone to prevent respiratory distress syndrome in preterm infants:
Report of a UK multicentre trial. Br J Obstet Gynaecol. 1989; 96:401–10.
9. Young BK, Klein SA, Katz M. Intravenous dexamethasone for prevention of
neonatal respiratory distress: A prospective controlled study. Am J Obstet
Gynecol. 1980; 138:203–9.
10. Doyle LW, Permezel MJ, Kitchen WH. Is there a lower limit for birth–
weight/gestational age and antenatal steroid therapy? Aust NZ J Obstet
Gynaecol. 1992; 32: 193–5.
8.- Doce días después de una infección de vías aéreas altas un niño de 11 años de edad
desarrolla debilidad de miembros inferiores , la cual es progresiva en días hasta afectar el
tronco, a la exploración física se encuentra arreflexia atrofia muscular y dolor en
miembros inferiores. El LCR solo muestra proteinuria, el diagnostico más probable es:

a) Parálisis de Bell.
b) Distrofia muscular.
c) Síndrome de Guillian Barré.
d) Enfermedad de Charcot Marie Tooth.

Se conoce como síndrome de Guillain- Barré a una serie heterogénea de neuropatías


periféricas de alivio espontáneo mediadas inmunológicamente.
El hallazgo común en ellas es la polirradiculoneuropatía de evolución rápida que se
desencadena casi siempre después de un proceso de tipo infeccioso. Se manifiesta más
frecuentemente con parálisis motora simétrica, con o sin pérdida de la sensibilidad, y en
ocasiones con alteraciones de tipo autonómico.
La debilidad de los músculos se agrava al máximo en las dos o tres semanas posteriores al
inicio del cuadro y la recuperación parcial o total ocurre en semanas o meses.

Por lo general evoluciona hacia la curación sin dejar secuelas clínicas evidentes, aunque
pueden surgir complicaciones riesgosas para la vida del enfermo. Hasta hoy constituye un
mal sin remedio, pues las terapias aplicadas se limitan a disminuir la gravedad del trastorno
y a acelerar la recuperación de la mayor parte de los pacientes.

Etiopatogenia
La gran mayoría de los pacientes se recoge el antecedente de una infección respiratoria o
gastrointestinal, bacteriana o viral, varias semanas antes del comienzo de los síntomas
neurológicos. Dentro de los antecedentes infecciosos en este síndrome se encontró una
mayor frecuencia de infección por Campylobacter jejuni, Cytomegalovirus y virus de
Epstein
Barr, también detectó infecciones por Mycoplasma pneumoniae, virus de la hepatitis,
herpes simple, mononucleosis infecciosa y SIDA (HIV). También se ha asociado con
vacunación (influenza, antirrábica, etc.), enfermedades sistémicas (enfermedad de Hodgkin,
lupus eritematoso sistémico, sarcoidosis) y cirugía.

Diagnóstico
Se basa en los criterios clínicos considerando los antecedentes, hallazgos en el líquido
cefalorraquídeo (LCR), serología para anticuerpos específicos y criterios
electromiográficos.

Criterios diagnósticos para Síndrome de Guillain-Barré son:


1. Hallazgos necesarios para hacer el diagnóstico:
• Debilidad progresiva en varias extremidades
• Arreflexia
2. Hallazgos que apoyan fuertemente el diagnóstico
a) Datos clínicos en orden de importancia
• Progresión desde unos días a 4 semanas
• Relativa simetría
• Alteraciones sensoriales leves
• Compromiso de pares craneales incluyendo el facial
• Recuperación que comienza 2 a 4 semanas después de detenerse la progresión
• Disfunción autonómica
• Ausencia de fiebre una vez instalado el síndrome
b) Estudio del líquido cefalorraquídeo
• Proteínas elevadas después de una semana
• Menos de 10 linfocitos /mm3
c) Pruebas electrofisiológicas
• Conducción nerviosa lenta
• Latencias distales prolongadas
• Respuestas tardías anormales
3. Hallazgos que hacen el diagnóstico dudoso
• Existencia de un nivel sensorial
• Marcada asimetría de síntomas y signos
• Disfunción severa y persistente de vejiga e intestino
• Más de 50 células/mm3 en LCR
4. Hallazgos que excluyen el diagnóstico
• Diagnóstico de botulismo, miastenia, poliomielitis o neuropatía tóxica
• Metabolismo alterado de las porfirinas
• Difteria reciente
• Síndrome sensorial puro sin fatiga

BIBLIOGRAFIA

1. Valls JS, Casademont JP, Berciano BD. Enfermedades de los nervios periféricos. En:
Farreras VP, Rozman CR y col. Medicina Interna. 14 ed. Madrid: Ed. Harcourt SA; 2000:
Vol 2: 1753-70.
2. Newswanger DL. Guillain-Barré Syndrome. Am FammPhysician [online] mayo 2004 [fecha
de acceso 15 de febrero 2007]; 69(10) URL.
3. Melano Carranza E, Carrillo Maravilla E, Gulias Herrero A y col. Síndrome de Guillain-
Barré en el anciano: un estudio retrospectivo. Arch. Neurocien. (Méx DF).

9.- Un niño de 7 meses de edad presenta una historia de estreñimiento por un mes. Expulsa
heces duras una vez a la semana. Por lo demás ha estado bien. Su exploración física es
normal. ¿Cuál de las siguientes es la causa más probable de su problema?

a) Hipotiroidismo.
b) Envenenmiento por plomo.
c) Estreñimiento funcional.
d) Enfermedad de Hirschprung.
La enfermedad de Hirschsprung (EH) es considerada una enfermedad congénita
caracterizada por una ausencia de células ganglionares en el plexo mientérico de Auerbach
y en el submucoso de Meissner, en el recto y otros segmentos del colon en forma
ascendente (de caudal a cefálico) (1). Esta alteración produce una anormalidad de la
motilidad intestinal, que se manifiesta más frecuentemente como una obstrucción intestinal
(2).

La EH puede ser clasificada según el segmento intestinal comprometido. Así puede


dividirse en: 1.- Segmento corto: Cuando no compromete más allá de la unión
rectosigmoidea;
2.- Ultracorto: Si sólo afecta esfínter interno o algunos centímetros próximos a dicho
esfínter;
3.-Segmento largo cuando el segmento agangliónico afecta más allá de la unión
rectosigmoidea. Otros autores sólo distinguen dos tipos: segmento corto y segmento largo,
siendo controversial la existencia de EH de segmento ultracorto, ya que esta compromete
menos de 5 cm. del recto distal .
La mayoría de los casos corresponde a EH de segmento corto (75 a 80%), una décima parte
de ellos sería un aganglionismo ultracorto. El 20% restante pertenecería a EH de segmento
largo, incluyéndose en este grupo aquellos que afectan los segmentos proximales al ángulo
esplénico.
La EH forma parte de los trastornos conocidos como disganglionismos que incluyen también
el hipoganglionismo y la displasia neuronal intestinal.

Expresión Clínica
Dentro de los síntomas que permiten una sospecha precoz, se encuentra el estreñimiento o
constipación, definida en el recién nacido como el retraso en la eliminación de meconio
mayor a 48 horas asociada a distensión abdominal, y en los niños mayores como
deposiciones infrecuentes de consistencia aumentada (El 98% de los lactantes elimina el
meconio en las primeras 48 horas de vida. Los prematuros eliminan más tardíamente el
meconio, pero la EH es rara en prematuros. De los pacientes con EH, sólo el 60% elimina el
meconio después de las 48 horas, por lo que este signo no es patognomónico de la
enfermedad.
La mayoría de los niños que presentan aganglionosis congénita, son sintomáticos los
primeros días o las primeras semanas luego del nacimiento (2). Alrededor de dos tercios de
los pacientes presenta síntomas dentro de los tres primeros meses de vida y 80%
desarrolla síntomas dentro del primer año de vida. Sólo un 10% de los pacientes inicia
síntomas entre los 3 y 14 años de edad y en general se trata de pacientes con enfermedad
de segmento ultracorto. (13)
Los recién nacidos y lactantes pequeños presentan con frecuencia signos de obstrucción
intestinal, distensión abdominal, vómitos biliosos e intolerancia a la alimentación. La
inspección anal y la radiografía pueden orientarnos hacia una causa mecánica de obstrucción,
pero no descarta EH. Si la obstrucción no tiene una causa mecánica, además de pensar en
una EH, debe plantearse el diagnóstico diferencial con hipotiroidismo, insuficiencia
suprarrenal, hipokalemia, hipercalcemia, hipomagnesemia, y en casos excepcionales
alteraciones neuromusculares.
Cuando la sintomatología es poco evidente.
Puede presentarse como un cuadro de constipación crónica, con historia de dificultad en la
eliminación de deposiciones, masas fecales palpables en fosa ilíaca izquierda y un tacto
rectal en que no se encuentran deposiciones en la ampolla rectal y esfínter anal hipertónico.
En muchas ocasiones la estimulación rectal provoca salida explosiva de heces líquidas de
olor fétido (17).

Por lo tanto, frente a pacientes con constipación crónica, en los cuales se ha descartado
causa mecánica de obstrucción intestinal, que no cede a las medidas dietéticas ni
farmacológicas, debe plantearse el diagnóstico de EH. También puede encontrarse
dilatación de asas intestinales, adelgazamiento de la pared abdominal, alteraciones de la
nutrición y el crecimiento.
En niños mayores, los síntomas más comunes incluyen constipación crónica progresiva,
impactación fecal recurrente, mal incremento ponderal y malnutrición.

Rev. Ped. Elec. 2008, Vol 5, N° 1. ISSN 0718-0918.


Servicio Salud Metropolitano Norte.
Facultad de Medicina Hospital Clínico de Niños.
Departamento de Pediatría y Cirugía Infantil Roberto Del Río.

Referencias:
1. De Manueles J. Enfermedad de Hirschsprung.
Protocolos diagnósticos y terapéuticos en pediatría. Sociedad Española de Pediatría. Pag.
56-60.
2. Feldmon T., Wershil B. Hirschsprung Disease. Pediatrics in review. Vol 23. N 11, August
2003.
3. Luis L.A., Encinas J.L., Avila L.F., et cols.
Enfermedad de Hirschsprung: enseñanzas de los últimos 100 casos. Cir Pediatr 2006;
19:177- 181.
4. J.M. Gil-Vener y cols. Diagnóstico dieferncial de Hirschsprung-neurodisplasia intestinal.
Fiabilidad de las pruebas diagnósticas. Cir Pediatr 2006; 19: 91-94.
5. M. López, y cols. Índices de fiabilidad de la manometría anorrectal para el diagnóstico de
la enfermedad de Hirschsprung en cualquier edad. Cir Pediatr 2005; 18:13-16.
6. Hernández F., Rivas S., Ávila L.F., Díaz M., ET cols. Aganglionismos extensos.
Tratamiento y resultados a largo plazo. Cir Pediatr 2003; 16: 54-57.
7. Goulet O. y cols. Intestinal transplantation in children: preliminary experience en Paris.
JPEN J Parenter Enteral Nutr 1999; 23 (5 Suppl)
8. Peña A. Enfermedad de Hirschsprung. Los avances y las preguntas no contestadas. Cir
Pediatr 2002: 15:46-47.
9. Polliotto S, Heinen F, Anduna G, Korman R. Evaluación de resultado a tres años de
nuestra primera experiencia en el tratamiento laparoscópico de la enfermedad de
Hirschsprung. Cir Pediatr 2001; 14: 85-87.
10.- Femenino de 8 meses refiere la madre que ha presentado cuadros frecuentes de
vómitos, movimientos incordiándoos y crisis convulsivas de difícil control, se le ha puesto el
pelo más claro que al resto de sus hermanos, piel seborreica, microcefalia, maxilar
prominente, dientes espaciados y retraso en el crecimiento ¿Cuál es el diagnóstico más
probable es?

a) Citrulinemia.
b) Homocistinuria.
c) Fenilcetonuria.
d) Enfermedad de Jarabe de Maple.

La fenilcetonuria, es un tipo de hiperfenilalaninemia, también conocida como PKU, es una


alteración del metabolismo en el que el organismo no puede metabolizar el aminoácido
tirosina a partir de fenilalanina en el hígado. Esta enfermedad es genética y es provocada
por la carencia de enzima fenilalanina hidroxilasa .

La fenilcetonuria tiene como rasgo principal la herencia genética autosómica recesiva, es


decir, los padres son portadores de los genes defectuosos y al ser traspasados de ambos
progenitores, la enfermedad se expresa en los descendientes.

La causa de la enfermedad es la carencia de la enzima fenilalanina hidroxilasa (FAOH) o de


la dihidropterina reductasa (DHPR) (también llamada tirosina hidroxilasa). Ambas enzimas
son responsables de la hidroxilación del aminoácido fenilalanina en la reacción que produce
tirosina. Por ello, el defecto o falta de alguna de ellas determina un incremento de la
concentración sanguínea de fenilalanina al impedirse su transformación en tirosina. También
se aumenta la transaminación de la fenilalanina como vía metabólica alternativa, y asimismo
se acumulan los metabolitos fenilpiruvato, fenilactato y fenilacetato. El defecto en la
síntesis de FAOH se debe a una anomalía génica localizada en el cromosoma 12, y el de la
DHPR en el cromosoma 4. Existen también formas de la enfermedad con déficits parciales.

El fenilpiruvato es un neurotori que afecta gravemente al cerebro durante el crecimiento y


el desarrollo. Los efectos de su acumulación causan oligofrenia fenilpirúvica, caracterizada
por un cociente intelectual inferior a 20. Los primeros meses de vida, los niños que padecen
esta enfermedad parecen estar sanos. Entre los tres y los seis meses pierden el interés
por el entorno, y al año se evidencia un retraso importante en su desarrollo. Los síntomas
suelen ser retraso psicomotor, cuadros psicóticos de tipo autista, convulsiones, síndrome
de West, convulsiones generalizadas y un eczema facial muy rebelde. Por lo general su
desarrollo físico es bueno, tienden a tener el cabello más claro que sus hermanos, piel clara,
y presentan un olor característico a paja mojada.

Cuadro clínico

La enfermedad se manifiesta, por primera vez, algunas semanas después del nacimiento,
iniciándose con una elevación en el plasma de la fenilalanina hasta un nivel 30 veces superior
al normal y por la excreción de ácido fenilpirúvico por la orina. Transcurridos 6 meses se
hace patente el retraso del desarrollo mental. La mayor parte de los pacientes son
deficientes graves o profundos y en ocasiones se alcanza la deficiencia media.
El portador de esta anomalía, que nace tras un embarazo normal y sin complicaciones, se
desarrolla durante los primeros meses casi siempre sin mostrar anormalidad ninguna. Sin
embargo Partington encontró, casi en la mitad de los lactantes, la existencia de vómitos en
los primeros meses de vida y en un tercio de ellos una irritabilidad inacostumbrada. En una
proporción similar de casos, a los padres ya les había llamado la atención un desagradable
olor del cuerpo del niño. Una parte de ellos mostró dermatosis eczematiformes durante el
primer trimestre, 7 de 36 ya había tenido en el primer año de vida ataques convulsivos. A
los 9 meses llama la atención el retraso en el desarrollo psicomotor.

Datos físicos

El desarrollo corporal cursa casi con normalidad. No obstante puede comprobarse cierta
tendencia al enanismo, aunque también se han descrito casos con tallas superior a la
frecuente. La dentición suele retrasarse hasta después del undécimo mes.

La gran mayoría de los enfermos muestran una piel clara, ojos azules, y color claro del pelo.
Alrededor del 10% poseen cabellos oscuros. La pobreza de pigmentos llama más la atención
en los pueblos de cabellos oscuros. La piel de los portadores además de ser clara es muy
suave aterciopelada y muy sensible. En algunos enfermos se han observado eflorescencias
papulosas en las caras de extensión de las extremidades y en la faz. En ciertos pacientes
se puede encontrar también una tendencia a la acrocianosis.

Datos conductuales

Características clínicas raras

• Cifosis.
• Pies planos.
• Espina bifida.
• Sindactilia en los dedos de los pies.
• Bloqueo cardiaco intraventricular.
• Hipogenitalismo.
• Dermografismo.
• Sensibilidad a la luz.
• Hipersegmentación de las células neutrófilas de la sangre.
• Disminución de la tolerancia a la galactosa.
• Metabolismo basal ligeramente elevado.

Las etapas del desarrollo habitual, la edad en la que el niño se sienta y habla, a veces, se
alcanzan a la edad normal, pero, de ordinario, se retrasa. En la edad límite en que debe
esperarse que el niño normalmente realice estos actos, el 35% no puede andar y el 63% no
puede hablar.

Estos niños, en general, tienen un peso y talla promedio por debajo del correspondiente a
su edad. En la mitad de los casos tiene microcefalia y prominencia del maxilar.

Sus movimientos son lentos y patosos y a menudo suelen adoptar la posición de sastre. Las
anomalías del tono muscular que contribuyen a estos cambios son de origen neurológico.2 de
cada 3 pacientes tienen hiperreflexia tediciosa e hipercinesia sobreañadida estos últimos
son voluntarios y muy variados.
Referencias:

1. Scriver CR, Kaufman S. 2001. Hyperphenylalaninemia: phenylalanine hydroxylase


deficiency. In Scriver CR, Beaudet AL, Valle D, Sly WS (eds). The Metabolic and Molecular
bases of Inherited Disease. McGraw-Hill., pp . 1667-1724.
2. Martínez-Pardo M, Marchante C. et al: 1998. ”Protocolo de diagnóstico, tratamiento y
seguimiento de las hiperfenilalaninemias”. An. Esp. Pediatría suplemento 114: 3-8.
3. Belanger-Quintana A, Morais A, Mena MA, Martínez- Pardo M. 2004. Niveles de
neurotransmisores dopaminérgicos y serotoninérgicos en líquido cefalorraquídeo en niños.
An Esp Pediatr 60:82.

11.- Un niño de 6 años presenta edema generalizado y proteinuria de 8 gramos al día, sin
hematuria, hipertensión ni disminución de la función renal. La actitud más adecuada en este
caso sería:

a) Confirmar la existencia de proteinuria.


b) Realizar biopsia renal.
c) Estudiar a fondo su estado inmunológico.
d) Administrar esteroides.

Este es un caso de síndrome nefrótico (proteinuria > 3.5g al día). La causa más frecuente
en los niños es la Glomerulonefritis de cambios mínimos (70% de los casos); si se tratara
de un adulto, deberíamos pensar que de tratarse de una glomerulonefritis membranosa (la
de cambios mínimos ocasiona sólo un 20% d los síndromes nefróticos de los adultos).
Es más frecuente en los varones, existiendo con frecuencia antecedentes de infección de
la vía respiratoria alta.
Clínicamente se manifiestan con aparición de proteinuria, con edemas en la región
periorbitaria, derrames, ascitis llegando incluso a la anasarca. No suele existir
hipertensión. En la analítica predomina una tendencia a la hipercolesterolemia y a la
hiperlipidemia.
En el estudio histológico, lo más característico es la ausencia de alteraciones en la
microscopía óptica (únicamente se encuentra un aumento de la matriz mesangial y acúmulo
de gotas lipídicas como consecuencia de la excesiva reabsorción en el túbulo). La
inmunofluorescencia es negativa, mientras que en la microscopía electrónica se demuestra
la ausencia o fusión de los podocitos (dato característico de la nefrosis lipoidea).
En general, ante un síndrome nefrótico en un niño debemos iniciar el tratamiento sin
necesidad de la biopsia, ya que en la mayoría de los casos es consecuencia de una
glomerulonefritis de cambios mínimos. Sin embargo, en un adulto, es necesario realizar
previamente una biopsia antes de aplicar el tratamiento.

Referencia:
Masson, Tratado de Medicina Interna., págs. 2368-72.
12.-Un niño capaz de correr, subir escaleras solo, sin alternar los pies, hacer una torre de
seis cubos y elaborar frases de tres palabras, presenta un desarrollo psicomotor propio de
la siguiente edad:

a) 15 meses.
b) 18 meses.
c) 24 meses.
d) 36 meses.

Durante el desarrollo neurológico lo esperable es que a los 15 meses camine solo, gatee
escaleras, hace torres de 3 cubos y nombra objetos familiares.
A los 18 meses sube escaleras cogido de una mano, hace torres de 4 cubos y nombra partes
de su cuerpo.
A los 24 meses sube y baja escaleras escaleras, escaló a escalón, hace torres de 7 cubos.
Frases de 3 palabras (sujeto, verbo y complemento).

Referencias:

1. Bender L.: "Test Guestáltico Visomotor de Bender". Ediciones Paidós - Barcelona


(2006)
2. Brazelton, T.B. y Nugent, J.K. "Escala para la evaluación del comportamiento
neonatal". Paidós. Barcelona (1997).
3. Broderick P.: "Pediatric vision screnning for the Family Physycian". American Family
Psysician Journal. Vol. 58 No 3 (1998)

13.- Masculino de 9 años que acude al servicio refiriendo dolor inguinal y escrotal intenso,
que comenzó de forma brusca posterior a realizar actividad física. El niño está afectado y
se observa tumefacción y dolor en el hemiescroto derecho, no se detecta reflejo
cremastérico y el testículo derecho está elevado y rotado. ¿Cuál de los siguientes
tratamientos considera usted, que es el adecuado para este paciente?

a) Analgésicos y antibióticos por vía endovenosa.


b) Tratamiento analgésico ambulatorio.
c) Exploración quirúrgica inmediata.
d) Ingreso hospitalario para observación clínica.

Torsión testicular
Aunque la torsión testicular puede ocurrir a cualquier edad hay dos picos de incidencia:
• En el período neonatal: suele tratarse de un edema escrotal asintomático, ya que la
torsión sucede intraútero y sólo veremos los cambios posteriores: aumento del tamaño del
teste, que se ve más oscuro, e hidrocele reactivo.
• En las primeras fases de la pubertad (edad media 14 años). Se debe a la fijación anormal
de los testículos a la cubierta escrotal, pudiendo rotar libremente dentro de la serosa,
sufriendo una torsión, con la isquemia vascular consiguiente.
Clínica
• Dolor testicular súbito e intenso, acudiendo al pediatra habitualmente en las primeras 12
horas. Con frecuencia se irradia al abdomen.
• Suele asociar naúseas o vómitos.
• En ocasiones está precedida de un traumatismo leve.
Exploración física
• El teste se suele ver más elevado y horizontalizado.
• Reflejo cremastérico. Servirá de comparación el obtenido previamente en el lado sano.
Para ello se roza suavemente la cara interna del muslo superior, constatando la existencia
de un movimiento de ascenso del teste dentro de la bolsa escrotal. En la torsión suele
estar ausente, aunque no es patognomónico: también puede faltar en las hernias inguinales,
hidroceles, orquitis o tumores. Sin embargo, su presencia casi descarta dicho diagnóstico
(aunque puede verse en las torsiones parciales o en torsión de poco tiempo de evolución).

Se debe palpar el teste y el epidídimo normal, y pasar después al afecto. El dolor testicular
es muy intenso, en todo el teste, aumentando al elevarlo. En ocasiones puede haber dolor
abdominal
infraumbilical referido. El hemiescroto, sólo en un tercio de los casos, puede estar
edematoso o hiperémico.
• Transiluminación: se observa escaso líquido libre, con el teste aumentado de tamaño y de
densidad respecto al contralateral.
• Se ha comunicado la existencia de torsión en testes no descendidos, por lo que esta
entidad debe ser sospechada en aquellos dolores abdominales bajos de inicio súbito y de
intensidad elevada.
• Siempre explorar el abdomen. En ocasiones, el dolor testicular puede ser un dolor
referido de otras localizaciones (apendicitis, cólico nefrítico, etc.).
Diferencial
• Otros procesos: hidrocele (poco doloroso, transiluminación demostrando líquido libre);
hernias inguinoescrotales (presencia de bultoma inguinoescrotal reducible); traumatismos;
púrpura de Schölein-
Henoch, varicocele, etc.
Exploraciones complementarias
• Diagnóstico clínico claro: exploración quirúrgica inmediata.
• Diagnóstico clínico no claro: sobre todo si han transcurrido más de
12 horas, está indicada la realización de ecoDoppler-color (sensibilidad
100% y especificidad del 97%): disminución o ausencia de flujo arterial en los casos de
torsión testicular comparado con el teste sano. Aunque en prepúberes el flujo testicular es
pequeño y puede no verse claramente en la ecoDoppler, por lo que en ausencia de flujo
visible en el testículo no doloroso, no podemos confirmar ni descartar la torsión. En estos
casos, y sobre todo si la clínica es muy sugestiva, se hará una exploración quirúrgica.
En torsiones intermitentes o si se ha destorsionado espontáneamente, la ecografía puede
mostrar un flujo aumentado, normal o disminuido.
En las torsiones evolucionadas se puede ver el signo del «halo» (aumento de la
vascularización y flujo alrededor del testículo).
• Otra prueba complementaria útil es la gammagrafía con Tc-99, pero no es superior a la
ecoDoppler.
Tratamiento
Es una urgencia quirúrgica, por cuanto las posibilidades de supervivencia del teste afectado
son del 90% en las primeras 6 horas, disminuyendo rápidamente con posterioridad (menos
del 20% tras 12 horas de evolución). Se procederá a la extirpación u orquidopexia, según la
viabilidad del teste. Es esencial la exploración y fijación del escroto contralateral, ya que la
afectación es bilateral hasta en el 40%. Tras un episodio de torsión testicular, la función
endocrina testicular (formación de testosterona) no se ve afectada. Sin embargo sí se han
encontrado alteraciones en la función exocrina (espermatogénesis) del teste contralateral.

BIBLIOGRAFIA:

1. Dogra V, Gottlieb R, Oka M, Rubens D. Sonography of the scrotum. Radiology


2003; 227: 18-36.
2. Jefferson RH, Perez LM, Joseph DB. Critical analysis of the clinical presentation of
acute scrotum: a 9 year experience at a single institution. J Uro 1997; 158: 1198-1200.
3. Kadish HA, Bolte RG. Revisión retrospectiva de pacientes pediátricos con epididimitis,
torsión testicular y torsión de las hidátides. Pediatrics (ed esp) 1998; vol 46 (1):35-39.
4. Kass EJ, Lundak B. In: Pediatric Urology. The acute scrotum. Pediatrics Clinics of North
America. October 1997, Vol 44 (5): 1251-66.
5. Nussbaum Blask A, Bulas D, Shalaby-Rana E, Rushton G, Shao C, Madj M. Color Doppler
sonography and scintigraphy of the testis: a prospective, comparative analysis in children
with acute scrotal pain. Ped Emerg Care 2002; 18(2): 67-71.
6. Perron C. Pain scrotal. Synopsis of Pediatric Emergency Medicine. Fleisher G, Ludhig S,
Silverman B. 4th ed. 2002; 187-191.

14.- ¿Cual de las medidas siguientes es más adecuada para combatir la hiperbilirrubinemia
(l0mg/dl) de un lactante de 3 semanas, con un desarrollo y crecimiento normales, que
recibe lactancia materna?

a) Fototerapia.
b) Exanguineotransfusión.
c) Fenobarbital.
d) Esperar un par de días y repetir la prueba.

La fototerapia es el empleo de luz visible para el tratamiento de hyperbilirubinemia en

el recién nacido (RN). Esta terapia relativamente común baja el nivel de bilirrubina en

el suero por transformación de la bilirrubina en isómeros solubles en agua que pueden

ser eliminados sin la conjugación en el hígado


La fototerapia convierte la bilirrubina que está presente en los capilares superficiales y

espacio intersticial a isómeros solubles en agua que son excretables sin pasar por el

metabolismo del hígado (Fig. 4). Maisels, un notable experto en bilirrubina, sugiere que

la fototerapia se parece mucho a una droga percutanea. Cuando la fototerapia ilumina la

piel, una infusión de fotones de energía, como moléculas de una medicina, es absorbida

por la bilirrubina de la misma manera que una molécula de medicina se une a un receptor.

Las moléculas de bilirrubina en la piel expuestas a la luz sufren las reacciones

fotoquímicas relativamente rápido, configurational isomerization, isomerización

estructural, y la forma de fotooxidación no tóxica, isómeros excretables.

Estos isómeros de bilirrubina tienen formas diferentes del isómero natal, son más

polares, y pueden ser excretados del hígado en la bilis sin sufrir la conjugación o

requerir transporte especial para su excreción. La eliminación urinaria y gastrointestinal

son ambas importantes en reducir la carga de bilirrubina.

Figura 4. El mecanismo de fototerapia. Cuando las moléculas de bilirrubina absorben la

luz, 2 reacciones fotoquímicas principales ocurren: el natural 4Z, 15Z-bilirubin se


convierte a 4Z, 15E bilirubin (también conocido como photobilirrubina) y a lumirrubina.

A diferencia de 4Z, 15Z la bilirrubina, photobilirrubina puede ser excretado vía

hepática sin la conjugación, pero su clearance es muy lento, y su conversión es

reversible. En el intestino (lejos de la luz), photobilirrubina es convertida atrás a

bilirubina natal.

La lumirrubina no es reversible. Aunque mucho menos lumirrubina que photobilirrubina

es formado, lumirrubina es eliminado del suero mucho más rápidamente, y es probable

que la formación de lumirrubina es principalmente responsable de la disminución en el

suero de la bilirrubina. Las pequeñas cantidades de bilirrubina natal también son

oxidadas a monopyrroles y dipyrroles que pueden ser excretados en la orina. Esto es un

proceso lento y sólo un contribuidor menor a la eliminación de bilirrubina durante la

fototerapia. Cortesía de diagrama de María Puchalski.

El objetivo de la fototerapia es disminuir la bilirrubina sérica y prevenir su acumulación

tóxica en el cerebro, donde puede causar serias complicaciones neurológicas

permanente conocido como kernicterus. La fototerapia ha reducido enormemente la

necesidad de exanguíneo transfusión para tratar la hiperbilirrubinemia.

La fototerapia es usada de 2 modos principales: profiláctica y terapéuticamente.

• En RN prematuros o aquellos con un conocido proceso hemolítico, a menudo

es usado profilacticamente, para prevenir un rápido aumento de la

bilirrubina sérica.

• En pretérminos pequeños o RN de término, es administrada en dosis

terapéuticas para reducir niveles de bilirrubina excesivos y evitar el

desarrollo de kernicterus.

La fotoisomerización de bilirrubina comienza casi al instante cuando la piel es expuesta

a la luz. A diferencia de la bilirrubina no conjugada, los fotoproductos de estos procesos

no son neurotóxicos. Por lo tanto, ante una hiperbilirrubinemia severa del RN, es

importante comenzar la fototerapia sin retraso.


Referencias
1. Stokowski LA. Early recognition of jaundice and kernicterus. Adv Neonatal Care

2002;2:101-114.

2. Maisels MJ. A primer on phototherapy for the jaundiced newborn. Contemp

Pediatr. 2005; 22(6): passim. (OR, 38, 40, 44, 47, 48, 53, 54,57).
3. McDonagh AF. Phototherapy: from ancient Egypt to the new millenium. J

Perinatol 2001;21:S7-S12.
4. Maisels MJ. Phototherapy-traditional and nontraditional. J Perinatol 2001;

21(Suppl 1):S93-S97.

15.- Masculino que presenta datos de microoftalmía, microcefalia, coriorretinitis,


antecedentes de una erupción petequial neonatal, que sufre retraso mental probablemente
padece la siguiente patología:

a) Síndrome cromosómico.
b) Infección por TORCH.
c) Síndrome de alcoholemia fetal.
d) Galactosemia.

Definición:

El síndrome de TORCH es una infección materna que afecta al feto en gestación. El

síndrome corresponde a un conjunto de síntomas y signos que presenta tanto el feto

cono el RN afectado por la infección congénita y que es producida por una serie de

agentes infecciosos: virales, parásitos y otros, que se han agrupado bajo la sigla TORCH.

Agentes etiológicos del síndrome de TORCH

Citomegalovirus, rubéola, herpes simple, hepatitis B


Virus y C, parvovirus 19, VIH, Enterovirus, varicela
zoster
Parásitos Toxoplasma gondii, Tripanosoma cruzi
Treponema pallidum, Mycobacterium tuberculosis,
Otros
Ureaplasma urealyticum , Micoplasma hominis, etc
Con todos estos agentes la madre puede tener una infección clínica o subclínica (más

frecuente la última situación). La madre durante el embarazo puede presentar una

promoinfección, una reactivación, reinfección o una infección crónica latente; cada una de

estas situaciones puede ser infectante para el recién nacido, lo que varía de un agente

etiológico a otro.

La infección puede ser transmitida vía transplacentaria, en el momento del parto o por

leche materna.

Dependiendo del agente causal y de las semanas de gestación al momento de la infección, el

impacto de ésta se puede traducir en aborto, mortinato, mortineonato, RN con

malformaciones, RN con infección subclínica, clínica o aparentemente sano. Las secuelas

afectan principalmente al SNC.

Tabla 2. TORCH: síntomas y signos

• Retardo del crecimiento intrauterino

• Hidrops no inmune

• Prematurez

• RN pequeño para la edad gestacional

• Hepatomegalia

• Esplenomegalia

• Púrpura

• Ictericia

• Anemia

• Microcefalia

• Hidrocefalia

• Calcificaciones cerebrales

• Coriorretinitis

• Neumonitis

• Alteraciones músculoesqueléticas

El diagnóstico se establece, en general, en el período postnatal y se confirma con la

detección de anticuerpos o de antígenos. Idealmente el diagnóstico debiera ser prenatal,

lo que permitiría en algunos casos, disminuir la transmisión materno –fetal.


Cifras de IgM total sobre 20 mg/dl en el RN, apoyan el diagnóstico de infección congénita

pero valores normales no lo descartan y no excluyen el estudio específico. La detección de

IgM específica debe ser complementada con otras técnicas por que da falsos positivos y

falsos negativos. La presencia de IgG en el RN puede estar dada por traspaso de Ac

maternos. Por lo tanto, se debe realizar una curva serológica, con 2 mediciones separadas

por 21 días, considerando de valor el alza en los títulos en por lo menos 4 veces el valor

basal. El estudio específico se debe efectuar en el binomio madre-hijo e incluye los

exámenes enumerados en la tabla 3:

El estudio complementario comprende un conjunto de exámenes a efectuar en el RN,

entre los que se cuentan: fondo de ojo, Rx de cráneo y huesos largos, pruebas hepáticas,

hemograma completo y eco cerbral.

Bibliografía:

1. Red Book. 2000. Report of the Committee on Infectious Disease 25ª Ed.

2. Villarroel J. y Reyes A. Síndrome de TORCH. Enfermedades Infecciosas en

Pediatría. Banfi. A. Ledermann W., Cofré J., Cohen J., Santolaya M.E. 3ª ed.

2004. 312-324.

Ilabaca G. Sindrome de TORCH, en Neonatología. Nazer J. Ramirez R. Hospital Clínico de la


U. de Chile. 2002. Cap. 26, 301-331.

16.- Femenino de 66 años que cursa con retinopatía diabética, en este momento presenta
hemorragias subhialoideas, usted determina que ésta paciente se encuentra en el siguiente
estadío:

a) Retinopatía diabética no proliferativa moderada.


b) Retinopatía diabética no proliferativa severa.
c) Retinopatía diabética con edema macular.
d) Retinopatía diabética proliferativa.

Clasificación

La Retinopatía Diabética se puede clasificar desde etapas más iniciales o leves, a otras más
avanzadas o graves de acuerdo a su aspecto oftalmoscópico, es decir, de acuerdo al
aspecto que se pueda apreciar en el examen de Fondo de Ojo. Para realizar este examen es
de cardinal importancia dilatar al paciente. Según el ETDRS la Retinopatía Diabética se
puede clasificar en una etapa temprana o Retinopatía Diabética No Proliferativa (RDNP) y
una más avanzada o Retinopatía Diabética Proliferativa (RDP). La RDNP se subdivide a su
vez en leve, moderada, severa y muy severa. La RDP se subdivide en temprana, de alto
riesgo y avanzada. El Edema Macular es un evento que puede suceder en cualquier momento
de la progresión de la Retinopatía Diabética.

Retinopatía Diabética No Proliferativa (RDNP)

Los cambios que se producen en la RDNP están limitados a la retina y no pasan más allá de
la membrana limitante interna de esta. Los elementos característicos que se pueden
apreciar en el examen oftalmoscópico comprenden microaneurismas, hemorragias
intraretinales en forma de manchas (dot-and-blot), edema retinal, exudados céreos o
lipídicos (hard exudates), dilataciones venosas y rosarios venosos (venous beading),
anormalidades intraretinales microvasculares (IRMA), manchas algodonosas (cotton-wool o
soft), anormalidades arteriolares y áreas de cierre capilar. De estas alteraciones, las
hemorragias intraretinales, los exudados céreos, las manchas algodonosas y las dilataciones
venosas, pueden ser vistas por el médico internista o médicos no oftalmólogos, usando un
oftalmoscopio directo y, como ya dijimos, con dilatación pupilar.

No detallaremos las características oftalmoscópicas específicas de cada una de las


subdivisiones de la RDNP. Sin embargo, es importante mencionar que las alteraciones más
importantes son: las hemorragias intraretinales, las dilataciones venosas y las
anormalidades intraretinales microvasculares. A mayor número estas, la RDNP aumenta en
su severidad y empeora en su pronóstico. Según el ETDRS, los pacientes con RDNP severa
tienen un 15% de posibilidades de progresar a RDP de alto riesgo en un año y los que
padecen RDNP muy severa tienen un 45% de posibilidades de progresar a RDP de alto
riesgo en un año.

Retinopatía Diabética Proliferativa (RDP)

La isquemia progresiva que se produce en la Retinopatía Diabética, debido al cierre capilar,


tiene como consecuencia la formación de vasos retinales de neoformación o Neovasos, los
cuales junto a un tejido fibroso que los acompaña, proliferan más allá de la retina. Es lo que
se denomina proliferación extraretinal. Estos neovasos son histológicamente diferentes a
los vasos retinales normales. No conservan la barrera hematoretinal, sangran con mayor
facilidad, y como mencionamos recientemente, crecen sustentados en un tejido fibroso, el
que tiene capacidad contráctil. Estas características recién mencionadas, son las
responsables de la Filtración Extravascular, las Hemorragias Preretinales o Vítreas y los
Desprendimientos de Retina Traccionales, respectivamente. En etapas avanzadas los
neovasos pueden crecen en la superficie iridiana y en el ángulo iridocorneal. El crecimiento
del tejido fibrovascular sobre el ángulo, lo hace impermeable a la salida de humor acuoso,
originando aumento de la presión intraocular y glaucoma secundario. Este glaucoma se
conoce como Glaucoma Neovascular, el cual es de muy mal pronóstico. La aparición de estos
neovasos es lo que define a la Retinopatía Diabética Proliferativa. Los neovasos se observan
con mayor frecuencia en el nervio óptico o cercanos a las arcadas vasculares, pero se
pueden encontrar en cualquier parte del fondo de ojo. Es posible ver los neovasos usando un
oftalmoscopio directo y dilatación pupilar.
La RDP evoluciona en tres etapas de menor a mayor severidad: temprana, de alto riesgo y
avanzada. Esto está dado por la ubicación y extensión de los neovasos, la presencia o
ausencia de hemorragia vítrea y la presencia o ausencia de desprendimiento de retina con
compromiso foveal.

Bibliografía

1. AAO, Basic and Clinical Sciences Course, Retina 2000-2001


2. Verdaguer TJ, Retinopatía Diabética. Clasificación, Normas para Pesquisa y
Tratamiento
3. Freeman WR, Practical Atlas of Retinal Disease and Therapy, Second Edition
4. Alvarez NR, Diploma de Actualización en Medicina 1999, Modulo de Diabetes,
Diagnostico y Tratamiento de la Retinopatía Diabética.

17.- Mujer de 24 años acude al servicio de ginecología, por referir ciclos opso-menorreicos
desde el inicio de su menarquia, en los últimos 7 días ha incrementado 15 Kg. de lo que
pesaba habitualmente, se aprecia una gran cantidad de acne, pero además refiere
depilarse el área del bigote cada semana, y cree que esto le sensibiliza la piel para que
aumente el acne.
En el caso de ovario poliquístico el dato clínico que con más frecuencia les acompaña es:

a) Anovulación y esterilidad.
b) Hirsutismo.
c) Amenorrea.
d) Obesidad.

El síndrome de ovarios poliquísticos (SOPQ) afecta aproximadamente a un 4% de mujeres


en edad reproductiva y se caracteriza por anovulación crónica e hiperandrogenismo. Es la
causa más común de infertilidad en mujeres.
Se caracteriza clínicamente por acné, alopecia, hirsutismo, irregularidades menstruales e
infertilidad.
Los hallazgos de laboratorio más frecuentes son: aumento de la hormona luteinizante (LH),
aumento de la relación LH/FSH (hormona folículoestimulante), aumento de andrógenos
(tanto ováricos como adrenales) y de estrógenos circulantes. Otros hallazgos de
laboratorio habituales son una prueba tolerancia oral a la glucosa anormal y alteraciones en
el perfil lipídico.
Todo esto junto con las imágenes ecocardiográficas características define al síndrome.
La terapéutica permite dos grandes enfoques que pueden superponerse: la corrección de
las manifestaciones de hiperandrogenismo y el tratamiento de las alteraciones del eje
reproductivo (anovulación, esterilidad). Los antiandrógenos están fundamentalmente
indicados para tratar los síntomas virilizantes.
Las alternativas para inducir la ovulación son numerosas: al citrato de clomifeno y a la
antigua resección en cuña se agregan las gonadotrofinas humanas, pulsos de GnRH (hormona
liberadora de gonadotrofinas), medidas o fármacos para modificar los niveles de insulina, y
finalmente técnicas quirúrgicas endoscópicas para reducir la masa ovárica.

BIBLIOGRAFIA

1. Guzick D.Polycystic ovary syndrome: Symptomatology, pathophysiology, and


epidemiology. Am J Ostetric Gynecol 1998; 179 (6): 89-93.
2. Stephen Franks. Polycystic ovary syndrome. N Engl J Med 1995; 333(13): 853-861.
3. Gori J.R., Larusso A. Ginecología de Gori. 2ª Edición. Buenos Aires, Argentina.
Editorial El Ateneo. 2001.
4. Adams J., Polson D. W., Franks S. Prevalence of polycystic ovaries in women with
anovulation and idiopathic hirsutism. Br Med J 1986; 293: 355-9.
5. Copeland L. J . Ginecología. Buenos Aires, Argentina. Editorial Panamericana. 1ª
Edición. 1994.
6. Ehrmann D.A., Rosenfield R.L., Barnes R.B., Brigell D.F., Sheikh Z. Detection of
functional ovarian hyperandrogenism in women with androgen excess. N Engl J Med
1992; 327:157-162.
7. Kahasar-Miller M., Conway Myers B., Boots L., Azziz R. Steroidogenic acute
regulatory protein (StAR) in the ovaries of healthy women and those with
polycystic ovary syndrome. Am J Obstet Gynecol 2001; 185(6): 1381-7.
8. Pérez Sánchez A. Ginecología. Santiago de Chile. Publicaciones Técnicas
Mediterráneo. 3ª Edición. 1995.
9. Velázquez E., Mendoza S., Hamer T., Sosa F., Glucck C. Metformin therapy in
women with polycistic ovary syndrome reduces hiperinsulinemia, insulin resistence,
hyperandrogenemia, and systolic blood pressure, while facilitating menstrual
regularity and pregnancy. Metabolism 1994 ; 43: 647-655.

18.- Masculino de 44 años que cursa con infección aguda por VIH, éste paciente
clínicamente se presenta con:

a) Fiebre, adenopatias, faringitis y rash.


b) Cefalea, alteraciones gastrointestinales y visceromegalias.
c) Adenopatías, cefalea y diarrea.
d) Rash, prurito y cefalea.

Manifestaciones clínicas en la fase aguda VIH.

Los principales síntomas y signos son:

• Fiebre y/o sudoración, 97%


• Adenopatías, 77%
• Odinofagia (dolor al tragar), 73%
• Erupción cutánea, 70%
• Artralgias y mialgias (dolor de articulaciones y músculos), 58%
• Trombopenia, 51%
• Leucopenia, 38%
• Diarrea, 33%
• Cefalea (dolor de cabeza), 30%
• Elevación de las transaminasas, 23%
• Anorexia, náuseas o vómitos, 20%
• Hepato y/o esplenomegalia (aumento tamaño de hígado o bazo), 17%

Referencias:

• Ioannidis JP, Contopoulos-Ioannidis DG. Chemokine receptor genotypes and HIV


disease progression: A preliminary meta-analysis [Abstract 13311]. 12 Conferencia
Mundial del SIDA, Ginebra, Suiza.
• Jesper Eugen-Olsen J, Iversen AKN, Benfield TL, et al. Chemokine receptor CCR2b
64I polymorphism and its relation to CD4+ T cell counts and disease progression in
a Danish cohort of HIV infected individuals [Abstract 13327]. 12 Conferencia
Mundial del SIDA, Ginebra, Suiza.
• Kupfer B, Kaiser R, Matz B, Brackmann HH, Schneweis KE: The homozygous defect
in the CCR5 gene gives a high degree of protection against HIV infection also when
inoculated parenterally [Abstract 23255]. 12 Conferencia Mundial del SIDA,
Ginebra, Suiza.
• Kaiser R, Kupfer B, Rockstroh JK et al: CCR5-genotype and CC-chemokine levels
are not significantly different in infected and uninfected recipients of a HIV-1
contaminated clotting factor concentrate. [Abstract 13380]. 12 Conferencia
Mundial del SIDA, Ginebra, Suiza.

19. - A 71 years-old man complains of occasional lower back pain. His blood pressures
150/85 mmHg and his pulse is 80/min. Cardiac examination reveals an S4 gallop. Abdominal
examination reveals a pulsatile mass approximately 5.0 cm. in diameter palpable in the
epigastric area. Peripheral pulses are normal. Which of the following is the most likely
diagnosis?

a) Abdomninal aortic aneurysm.


b) Cancer of the proximal colon.
c) Chronic pancreatitis.
d) Lipoma of the abdominal wall.

Epidemiología:
Los aneurismas abdominales, afectan al 2-5% de los mayores de 60 años y más en
hombres que en mujeres, en una proporción 4/1 (1).

La mayoría son de etiología arterioesclerótica y de ubicación infrarrenal (90%); y pueden


extenderse a una o ambas arterias ilíacas (70% de los casos) (1). Según la forma, los
aneurismas son fusiformes (75%) cuando afecta toda la circunsferencia del vaso, o
saculares (con cuello) cuando solo está englobado una porción de dicha circunsferencia.
Estos últimos son infrecuentes de observar en la aorta abdominal, y por lo general se
ubican proximales al origen de las arterias renales.

Clínica:
El 75% de los pacientes son asintomáticos, y se descubren por hallazgos en exámenes de
rutina o por una masa pulsátil en determinadas posiciones corporales. Ocasionalmente el
diagnóstico se hace por laparotomía por otras patologías (1).

Los pacientes sintomáticos consultan por:

• Dolor epigástrico o lumbar.


• Masa pulsátil dolorosa a la palpación.
• Síntomas gastrointestinales como nauseas, vómitos y pérdida de peso, en
aneurismas de gran tamaño.
• Asociado a episodios isquémicos en miembro inferior (raro).

Lo más frecuente es el hallazgo de una masa pulsátil umbilical o supraumbilical (se palpa
cuando su diámetro es > a 4,5 cm).

BIBLIOGRAFIA

1. Ferraina P, Oria A. Cirugía de Michans. Buenos Aires: El Ateneo, 2000: 949-955


2. Farreras R. Medicina Interna. Madrid: Mosby/ Doyma Libros,1995: Vol 2: 650-651
3. Frutos Ortiz E, Moirano J, Fassi J. Cirugía. Buenos Aires: El Ateneo, 1993: 67.11
4. Chang JB, Stein TA, Liu JP, Dunne ME. Risk factors associated with rapid growt of
small abdominal aortic aneuryms. Surgery 1997, 121: 117-122
5. Matsushita M, Nishikimi N, Sakurai T, Nimura Y. Relationship betwen aortic
calcification and atherosclerotic disease in patients with abdominal aortic aneurym:
Int Angiol 2000, 19: 276- 279.
6. Du Toit DF, Louwrens H, Klompje J, Grenewald JH. Ruptured abdominal aortic
aneurym and horseshoe kidney. A Afr Med J 1983, 64: 750-751.

20.- Un jardinero de 60 años de edad presenta cambios de personalidad. Su familia indica


que varias semanas antes se quejó de dolor en la muñeca y hace una semana tuvo asimetría
facial transitoria. ¿Cuál de los siguientes sería un tema especialmente importante a
considerar?

a) Exposición a toxinas.
b) Hábitos sexuales.
c) Picadura de garrapata.
d) Picadura de mosquito.

Allen R. M. MMS Medicina Interna. 5ª. Edición. National Medical Series. Mc. Graw Hill.
2006. (capítulo 8 VII F 3 b, c). La enfermedad de Lyme se transmite por picadura de
garrapata. Las complicaciones neurológicas son neuropatía craneal, radiculopatía y
encefalopatía. Las actividades en el exterior de este paciente lo ponen en riesgo de
exposición a enfermedad de Lyme si vive en un área endémica y la sugerencia de artritis de
la muñeca concuerda con las manifestaciones sistémicas de aquélla. La exposición a toxinas
puede causar confusión, pero no produce artritis o debilidad facial. Una conducta sexual de
alto riesgo predispone a infección por virus de la inmunodeficiencia humana (VIH) que
puede causar parálisis de nervio facial y encefalopatía. Sin embargo, dados los
antecedentes y la presentación de este enfermo, es más probable la enfermedad de Lyme.
Una picadura de mosquito puede transmitir encefalitis vírica. Sin embargo, no hay
antecedente de fiebre y la artritis de la muñeca y la debilidad facial no apoyan un
diagnóstico de encefalitis vírica temprana. El consumo excesivo de vitamina no produce
artritis, neuropatía de pares craneales ni cambios de la personalidad.

21.- Paciente femenino de 3 meses de edad que hace tres días inicia con rinorrea hialina,
tos y estornudos. El día de ayer presenta taquipnea con 55 rpm, tiraje intercostal con
aleteo nasal. A su ingreso se observan distrés respiratorio con sibilancias inspiratorias y
espiratorias y algunos crepitantes bilaterales. Una Rx de tórax muestra hiperinsuflacción
bilateral con una atelectasia laminar y corazón pequeño. Presenta: pH 7,24; pCO2: 58 mmg;
CO3H: 21 mEq/L. ¿Cuál es el diagnóstico más probable?

a) Bronconeumonía bilateral.
b) Crisis asmática de origen infeccioso.
c) Bronquiolitis.
d) Neumonitis intersticial.

La bronquiolitis es la enfermedad del tracto respiratorio inferior más frecuente en los dos
primeros años de vida. Aunque su mortalidad es baja, su elevada morbilidad origina una gran
demanda asistencial y causa un importante número de ingresos hospitalarios. Se presenta
de forma epidémica durante el invierno y principio de la primavera. Su etiología es viral, y el
virus respiratorio sincitial (VRS) es el agente que con más frecuencia se aísla.
El diagnóstico es eminentemente clínico. En la mayoría de los casos, cursa de forma benigna
y autolimitada; las formas más graves se presentan en lactantes más pequeños y en los que
tienen algún factor de riesgo asociado, en estos casos la tasa de hospitalización es alta.

El cuadro comienza como un catarro de vías altas, con estornudo, tos, rinorrea y en
ocasiones fiebre; y en el transcurso de 2 ó 3 días se intensifica la tos, aparecen los
síntomas de obstrucción de la vía respiratoria respiratoria inferior con aumento del
trabajo respiratorio, taquipnea e irritabilidad.
En los casos más graves, la dificultad respiratoria es marcada y presenta rechazo de las
tomas de alimento y postración.
La apnea puede ser la primera manifestación de bronquiolitis en lactantes pequeños.
Dada la evolución del curso clínico de la bronquiolitis, se recomienda reevaluar a las 24-48
horas a todos los lactantes menores de 12 meses que, en ambiente epidémico consulten por
un catarro de vías altas. Además, es conveniente advertir a los padres de la necesidad de
consultar ante la presencia de dificultad respiratoria, agitación, mal color, vómitos o
rechazo de la alimentación.
A la exploración física, podemos encontrar retracciones costales (tiraje subcostal e
intercostal) y la auscultación pulmonar puede ser normal, aunque en la mayoría de los casos
presenta espiración alargada, roncus, sibilantes y crepitantes bilaterales. Los sibilantes
pueden escucharse con el oído desnudo, sin ayuda del fonendoscopio.
No existe un patrón radiológico típico en la bronquiolitis. Los hallazgos radiológicos más
comunes son: el atrapamiento aéreo, engrosamientos peribronquiales, infiltrados
intersticiales y atelectasias laminares o segmentarias.
El hemograma es inespecífico y sólo estaría indicado realizarlo en aquellos casos en los que
sospechemos alguna complicación.
Para el diagnóstico de la hipoxemia se recomienda la pulsioximetría transcutánea, ya que
tiene muy buena correlación con la PaO2 sanguínea. Se considera hipoxia leve del 96-98%;
moderada, del
93-95%, y grave, menos de 93%.

Referencia:
González Caballero D, González Pérez- Yarza E. Bronquiolitis aguda: bases para un
protocolo racional. An Esp Pediatr
2001; 55 (4): 355-64.

22.- Se trata de un recién nacido al que se decide colocar una sonda orogástrica, al darnos
cuenta de que dicha sonda no avanza y apreciar en una radiografía toracoabdomial la
ausencia de aire en el intestino, ¿Sospechamos de?

a) Atresia de esófago tipo II y V.


b) Atresia de esófago tipo III y IV.
c) Hernia diafragmática congénita.
d) Atresia de esófago tipo I y II.

Consiste en la interrupción de la luz esofágica con o sin comunicación con la vía aérea. No
existe aún una definición precisa del término "long gap". De acuerdo a nuestra experiencia
se trata de aquellas AE, independientemente del tipo, caracterizadas por una distancia
entre cabos esofágicos lo suficientemente grande como para imposibilitar la realización de
una anastomosis primaria término-terminal.

De acuerdo a nuestro criterio, basado en la clasificación de Ladd, las atresias de esófago


pueden clasificarse en los tipos I, II, III, IV y V
1). La Tipo I es la atresia de esófago aislada sin fístula traqueoesofágica o “atresia pura”.
No tiene comunicación con la vía aérea. Representa aproximadamente el 5% de los casos
observados y es la segunda en frecuencia. Puede diagnosticarse con más facilidad en el
embarazo, que cursa típicamente con polihidramnios y ausencia de imagen gástrica en las
ecografías prenatales. Al nacer, los neonatos presentan el abdomen excavado por falta de
pasaje de aire al intestino. En todos los casos ambos cabos esofágicos se encuentran muy
separados entre sí.
La Tipo II es una forma muy rara de atresia de esófago que representa del 1 al
3% de los casos. Existe una fístula traqueoesofágica desde el cabo superior del esófago a
la tráquea cervical. Al igual que en las Tipo I, no hay pasaje de aire al intestino distal y
ambos extremos se encuentran muy alejados entre sí.
La Tipo III es la forma observada en el 90% de los pacientes. Comúnmente se la conoce
como sinónimo de atresia de esófago. Se caracteriza por una bolsa esofágica comunicación
entre el extremo distal esofágico y la tráquea (fístula tráqueoesofagica al cabo inferior),
que puede nacer a cualquier altura de la tráquea y excepcionalmente, también en los
bronquios. En estos casos el embarazo no cursa típicamente con polihidramnios y suele
verse una imagen gástrica normal en las ecografías prenatales. Al nacer los neonatos suelen
presentar el abdomen distendido por el gran pasaje de aire que existe desde la vía aérea
hacia el estómago e intestino. La distancia entre ambos cabos es por lo general inferior a
dos o tres vértebras. La coexistencia de atresia de esófago Tipo III con atresia duodenal,
intestinal o malformaciones anorrectales constituye un cuadro de urgencia; el gran flujo de
aire desviado hacia el sistema digestivo a través de la fístula traqueoesofágica inferior
provoca distensión abdominal severa y eventualmente, incluso, perforación intestinal.
La Tipo IV se caracteriza por una fístula tráqueoesofagica en ambos cabos esofágicos. Su
observación es poco frecuente. Representa menos del 2% de los casos. La fístula superior
suele encontrarse por accidente cuando se efectúa la corrección quirúrgica inicial. En
ocasiones pasa inadvertida por el cirujano y su diagnóstico es efectuado en forma tardía.
La Tipo V no es en realidad una atresia de esófago. El esófago se encuentra permeable y es
de buen calibre.
En estos pacientes lo que se observa es una fístula traqueoesofágicaaislada, generalmente
de ubicación cervical o torácica alta. El diagnóstico se realiza excepcionalmente en el
periodo neonatal ya que los síntomas suelen aparecer en la infancia.
Bibliografía:

Urgencias en Pediatría, Interamericana.McGraw – Hill. Capítulo: Urgencias


Médico Quirúrgicas, Sección XXIII, pág. 774-778.
Operative Pediatric Surgery. Moritz M. Ziegler. International Edition, pág. 349- 354.
Cirugía Pediátrica, Ashcraft - Holder Interamericana.McGraw – Hill pág. 257 a 277.

23.- Se trata de un recién nacido que presenta ausencia de lanugo y vérmix gaseosa
sospechamos de:

a) Prematuros.
b) Termino.
c) Postmaduros.
d) Hipotrófico.

Definición:
Se define como recién nacido postérmino a cualquier niño nacido más allá de 42 semanas de
gestación, cualquiera que sea su peso. El recién nacido postmaduro es el producto de una
gestación postérmino o gestación prolongada que presenta signos de postmadurez.

Signos más comunes:

Peso normal o disminuido y talla aumentada.


Piel seca y descamada, puede ser apergaminada.
Uñas largas y quebradizas.
Manos tumefactas con descamación precoz e intensa.
Abundante cabello.
Lanugo y vérmix caseoso ausentes.
Depósitos de grasos disminuidos.
Coloración verde, marrón o amarilla de la piel y uñas por impregnación de meconio.
Actitud vivas y facies despierta.

BIBLIOGRAFÍA

· García-Monroy L., Embarazo Prolongado en: Nuñez-Maciel E. et al. Tratado de Ginecología


y Obstetricia, Volumen II Cap.50.México, Ed. Cuéllar, S.A de C.V; 2001. pp. 687-694.

· Matute GMM. Inicio del trabajo de parto. En: Nuñez-Maciel E. et al. Tratado de
Ginecología y Obstetricia, Volumen I Cap.10. México, Ed. Cuéllar, S.A de C.V; 2001. pp. 187-
194.
· Clinicas Obstétricas y Ginecologicas Vol 1:2002. McGraw-Interamericana.

· Álvarez-Ponce VA., Lugo-Sánchez AM., Álvarez-Sánchez AZ., Muñiz-Rizo ME.


Comportamiento del embarazo prolongado en el servicio de cuidados perinatales. Rev
Cubana Obstet Ginecol 2004.

24.- Masculino de 15 años de edad, con antecedentes de cuadro gripal de tres días de
evolución, inicia hace 12 hrs. con dolor en la bolsa escrotal y aumento de volumen, a la
exploración física se encuentra edema y enrojecimiento del escroto y no se puede palpar el
testículo derecho, ¿El diagnóstico más probable es?

a) Torsión testicular.
b) Orquiepididimitis.
c) Hidrocele agudo.
d) Hernia inguinal estrangulada.

El síndrome escrotal agudo es una urgencia urológica cuyo principal síntoma es el dolor
intenso del contenido escrotal, su importancia radica en el diagnóstico precoz de la torsión
testicular, cuadro de emergencia quirúrgica, por lo que se ha establecido un síndrome en el
que se engloban diversas afecciones intraescrotales cuyo denominador común es el dolor
testicular de aparición brusca que puede ir acompañado de otros síntomas dependiendo de
su etiología, como tumefacción escrotal, náuseas, vómitos, síndrome miccional... que nos
orientarán sobre el diagnóstico.

RECUERDO ANATOMICO

El escroto es la bolsa que aloja al testículo, éste es una glándula de forma tubular cuya
capa externa es la túnica albugínea. Del polo superior del testículo sale el epidídimo que
tras descender por la cara posterior de éste, se continúa con el cordón espermático. El
cordón espermático está formado por el conducto deferente, los vasos sanguíneos (arterias
espermáticas y deferencial y venas espermáticas), linfáticos y nervios que llegan a la
glándula. El cordón espermático está envuelto por el músculo cremaster.

La irrigación del testículo procede de la arteria espermática interna, rama de la aorta


abdominal y el drenaje venoso se realiza a través de la vena espermática interna que
desemboca en la vena cava inferior en el lado derecho y en la vena renal en el lado izquierdo.

Las funciones del testículo son endocrina, las células de Leydig sintetizan y liberan la
testosterona y exocrina que es la espermatogénesis.
ORQUITIS Y EPIDIDIMITIS

La inflamación del contenido escrotal plantea un problema de diagnóstico en el escroto


agudo principalmente con la torsión funicular. La infección aislada del testículo es rara,
siendo más frecuente la presentación en forma de orquiepididimitis. Predomina en hombres
jóvenes sexualmente activos y en ancianos con infección urinaria pero se puede producir a
cualquier edad.

La causa más frecuente de la inflamación del contenido escrotal es la infección. Así en los
jóvenes prepuberales los gérmenes más frecuentes son las enterobacterias aunque sobre
factores predisponentes como son las malformaciones congénitas (válvulas uretrales,
abombamiento ectópico de un uréter a vesícula seminal). En el adolescente es causa
frecuente la orquitis en el transcurso de una parotiditis. En los varones jóvenes menores de
40 años es la causa infecciosa con mucho lo más frecuente, sobre todo enterobacterias (E.
coli, Proteus Mirabilis, Klebsiella pneumoniae) y gérmenes de transmisión sexual como
Neisseria gonorrheae, Mycoplasma, pero sobre todo la Chlamydia tracomatis, y cocos gram
positivos como estafilococos aureus. Por encima de los 40 años a la causa infecciosa se
suele añadir una obstrucción urinaria distal, tuberculosis, reflujo urinario posterior a la
prostatectomía transuretral, y la reacción granulomatosa por el tratamiento del cáncer
vesical superficial con bacilo de Calmette Guerin (BCG).

También puede aparecer orquiepididimitis en el curso de la varicela, fiebre tifoidea,


mononucleosis infecciosa, rickettsiosis, brucelosis filariasis, actinomicosis, sinusitis,
osteomielitis, endocarditis y ser secundaria a cuadros septicémicos por E. coli, Klebsiella,
pseudomona, estreptococo y estafilococo. Una causa no infecciosa es el tratamiento con
amioradona.

Clínicamente aparece dolor escrotal intenso, irradiado a lo largo del cordón espermático
por lo que irradia a ingle e incluso a abdomen con tumefacción escrotal no distinguiéndose
el epidídimo del testículo, con induración de la piel escrotal que puede producir hidrocele
reactivo con síntomas de infección urinaria concomitantes y de secreción uretral, además
de afectación del estado general con fiebre que puede llegar a ser de hasta 40 °C, con
escalofríos, acompañado de náuseas y vómitos.

En la exploración física, el escroto se observa edematoso, tenso y enrojecido,


el cordón se palpa engrosado y doloroso, siendo muy dolorosa la palpación, y difícil de
distinguir el límite entre escroto y epidídimo en casos evolucionados. El signo de Prehn, al
elevar el testículo afecto produce mejoría del dolor, es positivo. La transiluminación del
escroto es negativa a no ser que exista un hidrocele reactivo, en tal caso será positiva.
En cuanto a los estudios complementarios habrá que realizar analítica de orina, cultivo de
orina, y de semen y de secreción uretral si la hubiere, cultivo de Lowenstein y tinción de
Ziehl-Nielsen tanto en orina como en semen e inmunofluorescencia para Chlamydia, además
de antibiograma. También es necesario realizar hemograma, hemocultivos seriados, una
bioquímica sanguínea que incluya urea, creatinina e iones.
En el sedimento de orina nos mostrará piuria en la mayoría de las ocasiones, discreta piuria
sin bacteriemia en las ETS y piuria franca con urocultivo positivo en caso de gérmenes
gram negativos, aunque también podremos encontrar a pacientes con sedimento urinario
negativo. En el hemograma es frecuente leucocitosis con desviación a la izquierda.
La ecografía nos mostrará engrosamiento y edema de las túnicas escrotales, pudiendo
haber o no hidrocele, con epidídimo engrosado (patrón heterogénico), y un testículo
aumentado de tamaño (patrón hipoecogénico global) o como una zona hipoecogénica
perihiliar.
La eco-dopopler está indicada cuando hay dudas sobre el diagnóstico diferencial con la
torsión funicular, siendo en estos casos muy útil, y en la orquiepididimitis encontraremos
una hipervascularización.

La gammagrafía está indicada para el diagnóstico diferencial con procesos isquémicos y


neoplásicos, es un método de gran sensibilidad y especificidad,
pero que no se tiene siempre disponible en el área de urgencias.

Se observará un aumento de la captación a nivel testicular debido a la hiperemia.

Si después de todas las pruebas complementarias existen dudas sobre el diagnóstico, será
necesaria la cirugía para llegar al diagnóstico definitivo.

Referencias
1. Young Y., Miller R Incidence of malignant tumours in U.S. children. J Pediatr 1975; 86:
254-258.
2. Reiter A., Schrappe M. Chemotherapy in 998 unselected childhood acute lymphoblastic
leukemia patients. Results and conclusions of the multicenter trial ALL-BFM 86. Blood
1994; 84: 3122-3133.

3. Schorin M., Blattner S., Gelber R., et al.


Treatment of childhood acute lymphoblastic leukemia: Results of Dana Farber Cancer
Institute/Children s Hospital acute lymphoblastic leukemia consortium protocol 85-01. J
Clin Oncol 1994; 12: 740-747.

25.- Paciente de 25 días de nacido al cual se desprendió el cordón umbilical a los 9 días y
cuyo ombligo no ha cicatrizado. Presenta en este nivel una tumoración roja, prominente,
circular y que segrega un contenido alcalino. Refiere la madre que a veces nota en esta
tumoración ruido de gases. ¿El diagnóstico más probable será?

a) Fístula vésico-umbilical.
b) Granuloma umbilical.
c) Persistencia del conducto onfalomesentérico.
d) Onfalocele.
Durante la vida fetal el conducto onfalomesentérico une al saco vitelino con el intestino
medio y se cierra normalmente para desaparecer por completo. Se encuentra conectado
con el intestino primitivo en el saco amniótico. En el desarrollo embriológico normal, el
conducto onfalomesentérico involuciona entre las 5a y 7a semanas de vida intrauterina. Un
fracaso en la regresión produce varias anormalidades, en dependencia del lugar donde se
localice este fallo: en el lado umbilical o en el intestinal.

Los vestigios del conducto onfalomesentérico (vitelino) pueden presentarse como anomalías
relacionadas con la pared abdominal. Sin embargo, puede ocurrir que todo o parte del
conducto fetal se mantenga y entonces se produzca sintomatología clínica. También puede
persistir como una estructura permeable en toda su longitud o mantenerse como un
divertículo o quiste cuando persiste en su periferia, parte central o media; o quizás quede
representado simplemente por un resto de epitelio intestinal ectópico a nivel umbilical o
como cordón fibroso.

La explicación embriológica de la persistencia del conducto onfalomesentérico puede


explicarse diciendo que el conducto onfaloentérico o vitelino que representa una
comunicación entre el vértice del asa umbilical fetal y el saco vitelino se oblitera y
desaparece junto con sus vasos acompañantes arteria y vena onfalomesentéricas hacíanla
séptima semana-fetal. Sin embargo, cuando este proceso no ocurre, da lugar a la patología
conocida de las estructuras remanentes del conducto onfalomesentérico.

Después de los cuidados iniciales en la edad del recién nacido que se han dado al cordón
umbilical y una vez que éste se ha desprendido, esta zona recibe poca atención en los días
subsiguientes; sin embargo, puede ser asiento de muchas lesiones, tanto congénitas como
adquiridas, con lascuales el médico debe estar familiarizado para poder diagnosticarlas.
El diagnóstico se hace mediante la observación, cuando el recién nacido realiza esfuerzos
como el provocado por el llanto, pues se produce la salida de material fecal o gases por el
ombligo.
REFERENCIAS

1. Pomeranz A. Anomalies, abnormalities, and care of the umbilicus. Pediatr Clin N Am 51


(2004) 819.

2. Rowe M, Chapter 48: Disorders of the umbilicus. Essentials of Pediatric Surgery, 1995.

3. Care of the umbilical cord, a review of the evidence. OMS. 1998.

4. Cushing A. Omphalitis: a review. Pediatr Infect Dis J 1985, 4(3); 282-285.

5. Janssen P. To dye or not to Dye: A randomized, Clinical Trial of a Triple Dye/alcohol


Regime Versus Dry cord care. Pediatrics 2003; 111, 15-20.

6. Zupan J. Topical umbilical cord care at birth The Cochrane Database of Systematic
Reviews, 2004. Issue 3.

7. Sawardekar K. Changing spectrum of neonatal omphalitis. Pediatr Infect Dis J,


2004;23:22-6.

8. Soto G. Anomalías uracales: Un diagnóstico rara vez planteado. Pediatría al día


2001;17(5) 331)

9. Cilento BG Jr. Urachal anomalies: defining the best diagnostic modality. Urology 1998,
52 (1) 120-2.

10. Ossandón F. Malformaciones congénitas: patología umbilical. Pediatría al día 1995; 11(2)

11. Lotan G. Double ligature: A treatment for pedunculated umbilical granulomas in


children. Am Fam Physician 2002; 65:2067-8.

12. Novoa A. El pediatra ante un lactante con caída tardía del cordón umbilical. Arch arg
Pediatr 2004; 102(3).

26.- Al realizar en un paciente de 7 años de edad el diagnóstico de escarlatina, usted elige


el siguiente fármaco por ser el medicamento de elección en esta patología.

a) Penicilina benzatínica.
a) Cefuroxime.
b) Amoxicilina.
c) Eritromicina.

Streptococcus pyogenes (hemolítico del grupo A) es el agente etiológico de la escarlatina,


no ofrece resistencia a la penicilina benzatínica y con dosis única ofrece un buen índice de
curación, erradicación y reducción de complicaciones.

González-Saldaña N, Infectología Clínica Pediátrica, 7ª edición, páginas 403-407.


27.- Recién nacido de 38 semanas de gestación en su segundo día de vida, que presenta una
dermatosis diseminada a cara y tronco caracterizada por vesículas y pústulas, que respeta
palmas y plantas. ¿Cuál es el diagnóstico?

a) Eritema Tóxico.
b) Melanosis pustulosa.
c) Dermatitis atópica.
d) Rubéola congênita.

El eritema tóxico es una entidad sin significado patológico que se manifiesta con vesículas
y pústulas sobre una base eritematosa, respeta palmas y plantas. Aparece entre el primero
y tercer día de vida. Están formadas por un infiltrado de eosinófilos, el cultivo es estéril y
desaparece en la primera semana. En cambio, la melanosis pustulosa se presenta al
nacimiento, se localiza en palmas y plantas, las lesiones están formadas por PMN, el cultivo
es estéril y desaparecen en varias semanas.

Bibliografía

1. Askin DF. Complications in the transition from fetal to neonatal life. J Obstet
Gynecol Neonatal Nurs 2002 May-Jun;31(3):318-27.
2. Bondas-Salonen T. New mothers' experiences of postpartum care--a
phenomenological follow-up study.:J Clin Nurs. 1998 Mar;7(2):165-74.
3. Desmond M. Franklin R, Vallbona C. et al: The clinical behavior of the newly born.
Infant. The term baby. J Pediatr 1965; 62:307.

Desmond M , Rudilph A, Phitaksphraiwan P.: The transitional Care Nursery. 1966 Pediatr
Clin North Am 13:651

28.- Masculino de 2 años y medio inicia con fiebre elevada de 4 días de evolución, con
afectación del estado general, y ligera irritabilidad. A la exploración destaca la presencia
de un exantema maculopapuloso discreto en tronco e hiperemia conjuntival bilateral sin
secreción. Presenta además enrojecimiento bucal con lengua aframbuesada e hiperemia
faríngea sin exudados amigdalares, además de adenopatías laterocervicales rodaderas de
unos 1,5 cm de tamaño. Ha recibido 3 dosis de Azitromicina. ¿Cuál, entre los siguientes, es
el diagnóstico más probable?

a) Mononucleosis infecciosa.
b) Rubeola.
c) Escarlatina.
d) Síndrome de Kawasaki.

Definición: Vasculitis generalizada


Descrita por el Dr. Tomisaku Kawasaki en 1967. Ese año él publica 50 casos, presentados
entre 1961 y 1967, con el nombre de Síndrome Linfomucocutáneo. Al principio se pensó que
era una enfermedad benigna, pero ya en 1970 se habían publicado 10 casos de muerte por
esta enfermedad.
Recientemente se examinó en Londres el corazón de un niño de seis años que había muerto
en 1870, con diagnóstico de “escarlatina”, encontrándose tres aneurismas coronarios
trombosados. Antes de la publicación de Kawasaki, los casos de esta enfermedad
identificados post mortem fueron denominados periarteritis nodosa infantil.

Etiología
Continúa siendo desconocida, pero se acepta que tiene etiología infecciosa, por el cuadro
clínico y la epidemiología. Existiría, además, una predisposición genética e inmunológica con
formación de un super antígeno, el que desencadenaría la enfermedad. Las células T serían
las receptoras del super antígeno.

Criterios diagnósticos
No existen exámenes patognomónicos, ya que la etiología continúa siendo desconocida, por
lo que el diagnóstico se basa en hecho clínicos.
Fiebre de más de cinco días, sin causa conocida, con al menos cuatro de los signos
siguientes:
— Inyección conjuntival bilateral, no exudativa.
— Cambios en la mucosa bucal, como: labios rojos, secos y fisurados; eritema de la faringe
y lengua de frutilla.
— Cambios en las manos y pies: enrojecimiento y edema en la etapa aguda y en la etapa
subaguda, descamación periungueal
— Linfoadenopatías cervicales conmás de 1,5 cm de diámetro cada una, indoloros y móviles.
Si existe compromiso coronario, sólo se exige fiebre de más de cinco días de evolución y
tres de los otros criterios clínicos.
Un hecho que llama la atención es la gran irritabilidad que presentan estos enfermos.
Como hechos asociados están: compromiso cardiaco, meningitis aséptica, artralgia o artri
tis, disfunción hepática leve, hídrops vesicular, diarrea, neumonitis radiológica, otitis media
e inflamación de la vacuna BCG.
Desgraciadamente, no todos los pacientes llenan los criterios para el diagnóstico. Existe el
síndrome de Kawasaki atípico o incompleto, con menos signos que los expuestos y con
compromiso coronario. Por esto el síndrome de Kawasaki debe estar presente en el
diagnóstico diferencial de todo cuadro febril prolongado sin causa etiológica demostrable,
en niños.
BIBLIOGRAFÍA Apuy. JJ. Enfermedad de Kawasaki. 1. 2003; 564:109-113. Brogan P, et al.
Kawasaki disease: 2. an evidence based approach to diagnosis, treatment, and proposal for
future research. Arch Dis Child 2002;86:286-292.Burns JC, et al. Kawasaki Disease: a 3.
Brief history. Pediatrics 2000; 106(2): 27.Burns JC, Glodé M. kawasaki syndrome. 4. The
Lancet 2004; 364: 533-44.

29.- Se realiza diagnóstico de roséola un paciente pediátrico, ¿Qué características


presenta el exantema?

a) Máculopapular violáceo, que no desaparece a la dígito presión.


b) Máculopapular eritemotoso, con descamación.
c) Máculopapular eritemotoso, de inicio súbito al desaparecer la fiebre.
d) Máculas, vesículas y costras, pruriginoso.

Exantema súbito

Manifestaciones clínicas: conocida también como "roséola infantum" o "fiebre de los tres
días", comienza con fiebre alta - en ocasiones hasta de 40,5°C - siendo característico en
estos niños que conservan un relativo buen estado general, pese a la fiebre. No es
infrecuente que consulten en el servicio de urgencia por convulsiones febriles. Actualmente
se sabe que el virus tiene tropismo hacia el sistema nervioso central y las convulsiones
pueden representar un efecto directo del virus sobre el tejido cerebral. Después de 3 a 4
días de fiebre, aparece el exantema que es macular o papular y se palpa granuloso; es de
color rosado pálido y no es pruriginoso. Se observa primero en el tronco, cuello y en la
región retroauricular; su generalización es infrecuente. El exantema dura alrededor de 2 a
3 días.

Agente etiológico: virus herpes tipo 6 (HHV6), virus ADN, miembro de la familia
Herpesviridae.

Epidemiología: afecta a los lactantes entre los 6 a 18 meses de vida, y su mecanismo de


transmisión se presume por la vía respiratoria y contacto directo con algún huésped que
esté excretando el virus. El período de incubación es de 9 a 10 días.

Diagnóstico: se hace por el cuadro clínico y la exclusión de otras enfermedades febriles


agudas. Existen estudios serológicos específicos aún en evaluación para diagnóstico de
infección aguda.

Aislamiento del paciente hospitalizado: precauciones estándar.

Tratamiento: manejo sintomático de la fiebre y eventualmente de episodios convulsivos.

Bibliografía:
• González-Saldaña N, Macías-Parra M. Vacunas en pediatría. 1er ed. México. McGraw-
Hill Interamericana; 1998.
• Comittee on Infectious Diseases 1997-2000. Red Book 2000, 25 ed. American
Academy of pediatrics; 2000.

30.- Un preescolar de 4 años de edad presenta un peso y talla en el percentil 3, se refiere


por presentar anorexia, irritabilidad, apatía al juego, se le observa con palidez, el
laboratorio reporta una hemoglobina de 7.3g/dl, hierro sérico de 36mcg/dl y saturación de
transferían de 12%, ¿El tipo de anemia más probable es?

a) Ferropénica
b) Hemolítica
c) Perniciosa
d) Drepanocítica

Es el tipo de anemia más frecuente en la población general. Se trata de una anemia


caracterizada por la disminución o ausencia de los depósitos de hierro, baja concentración
de hierro sérico, baja saturación de transferrina y baja concentración de la Hb o del
hematocrito.

La ferropenia se manifiesta con un síndrome característico:

SNC: irritabilidad, labilidad emocional, disminución de la concentración y la memoria,


cefalea, ataxia, parestesias, trastornos del sueño y la alimentación (PICA: apetencia
desmedida por sustancias poco habituales. Es muy característica la pica de hielo o
pagofagia), aumento de la sensibilidad al frío, síndrome de piernas inquietas…

Piel y mucosas: coloración azulada de la esclerótica, fragilidad de uñas y cabello, coiloniquia


o aspecto cóncavo de las uñas, glositis, queilitis angular o rágades, atrofia gástrica, en
casos graves producción de membranas esofágicas (síndrome de Plummer-Vinso).

Etiología:

La anemia ferropénica puede deberse a:

Aumento de la utilización: gestación, lactancia, crecimiento corporal rápido en la


infancia y adolescencia.
Pérdidas fisiológicas: menstruación
Pérdidas patológicas:
Hemorragia digestiva. El sangrado digestivo crónico es la causa más
frecuente en este grupo: hemorragias gástricas por medicamentos (AAS,
AINES, corticoides asociados a AINES…), hernia hiatal, diverticulosis,
hemorroides. En pacientes mayores de 60 años debe considerarse la
posibilidad de neoplasia, sobre todo de colon y la angiodisplasia intestinal.
Genito-urinarias
Aparato respiratorio
Hemólisis intravascular
Alteraciones en la absorción:
Dietas insuficientes: Debe interrogarse siempre acerca de los hábitos
dietéticos. El hierro presente en alimentos de origen vegetal como
cereales y legumbres tiene baja biodisponibilidad (algunos alimentos como
las lentejas tienen excesiva fama de riqueza en hierro). Incluso en carnes
y pescados el contenido en Fe es relativamente bajo. En gestantes, niños,
mujeres en edad fértil, la dieta puede ser un factor ostmenopáusi o
causal de deficiencia de hierro. En mujeres ostmenopáusicas y en el varón
la dieta sería causa excepcional de ferropenia.
Absorción defectuosa: gastrectomías parciales o totales, enfermedad
inflamatoria intestinal, enfermedad celíaca (causa relativamente frecuente
de anemia ferropénica); en algunos casos podría estar indicada la detección
de anticuerpos antiendomisio y antigliadina para descartar una enfermedad
celíaca no diagnosticada.

Diagnóstico:
El nivel de ferritina sérico bajo es indicativo de una situación de depleción de hierro y es la
prueba definitiva de la existencia de anemia por déficit de hierro (AF). Los demás
parámetros no permiten diferenciar con seguridad la AF de la anemia por enfermedad
crónica. Generalmente aparece microcitosis, aunque hasta en el 30% de las anemias
ferropénicas, en algunas poblaciones, el VCM puede ser >80; asimismo en el 10% de las
microcitosis los parámetros relacionados con el hierro son normales. Otros datos a tener
en cuenta:

El frotis periférico habitualmente muestra anisocitosis y poiquilocitosis.

Cuando la microcitosis se asocia a aumento de glóbulos rojos, sobre todo si es mantenida,


pensar en talasemia.

Puede haber trombocitosis reactiva.

Bibliografía:

1. Abramson S, Abramson N. Common' Uncommon Anemias. Am Fam Physc 1999; 59


(4). 851-8.
2. Adamson JW. Anemia and Polycytemia. En: Kaasper DL, Braunwald E (Edit.).
Harrison´s principles of internal medicine. 16th. ed. Nwe Cork: McGraw-Hill; 2005.
p.329-336.
3. Hernández Nieto L, Hernández García MT, Juncá Piera J, Vives-Corrons JL,
Martín-Vega C. Enfermedades del sistema eritrocitario: anemias. En: Farreras
Valentí P, Rozman C. (Dir.). Medicina Interna. Barcelona: Elsevier; 2004. p. 1644-
1669.
4. Glader B. Anemia: General Considerations. En: Wintrobe´s Clinical Hematology.
11th. Philadelphia: Lippincott Williams & Wilkins; 2004. p.947-978.

31.- Recién nacido a término, obtenido por cesárea. Presenta dificultad respiratoria
durante las primeras horas de vida, con frecuencias respiratorias de 90 por minuto y
cianosis que mejora con el oxígeno. Los campos pulmonares están bien ventilados. La
radiografía de tórax muestra líneas de líquido en las cisuras, y marcas vasculares
prominentes. ¿Cuál es su diagnóstico?

a) Aspiración de meconio.
b) Taquipnea transitoria del recién nacido.
c) Cardiopatía congénita cianógena.
d) Enfermedad de membrana hialina.

La Taquipnea Transitoria del Recién Nacido (TTRN), se define como su nombre lo dice,
como un aumento de la frecuencia respiratoria en el neonato de manera transitoria, es una
enfermedad benigna y autolimitada que afecta principalmente el neonato a termino, aunque
puede afectar a los neonatos pretérmino limite nacidos por cesárea.

CLÍNICA
El cuadro clínico se inicia desde el nacimiento, se caracteriza por taquicardia, frecuencia
respiratoria aumentada (60-160 rpm), retracciones esternales y subcostales, quejido
espiratorio y cianosis leve que mejora al administrar cantidades mínimas de oxigeno. La
clínica puede agravarse en las primeras 6-8 horas, para estabilizarse posteriormente y, a
partir de las 12-14 horas, experimentar una rápida mejoría de todos los síntomas, la
totalidad de los signos y síntomas suelen autolimitarse a los 3-4 días después del
nacimiento.
Los gases sanguíneos pueden revelar acidosis respiratoria que se resuelve en las siguientes
8 a 24 horas.
Al examen físico no se suelen encontrar ruidos pulmonares patológicos, no estertores ni
roncus. Los Rx de tórax pueden revelar hiperinsuflación, una trama vascular pulmonar
prominente, fisuras interlobares aumentadas de tamaño por la presencia de liquido,
aplanamiento y depresión del diafragma, aumento de la sombra cardiotímica (cardiomegalia)
y en ocasiones derrame pleural.
El diagnóstico diferencial incluye enfermedad de la membrana hialina, la cual se diferencia
de la TTRN por que ésta ultima presenta una brusca recuperación del recién nacido y tiene
ausente un patrón reticulogranular con broncograma aéreo en la Rx de tórax; otros son:
neumotórax, aspiración meconial, insuficiencia cardiaca congestiva, neumonías bacterianas o
víricas, trastornos metabólicos, policitemia e hiperviscosidad y hernia diafragmática
asociada a hipoplasia pulmonar; las características radiológicas y la mayor gravedad de la
dificultad respiratoria permiten diferenciar estos trastornos del TTRN.
El diagnóstico se confirma por la evolución favorable de los Rx a las 24-48 horas en
asociación con el mejoramiento del cuadro clínico.
En el manejo de la TTRN hay controversia, algunos textos (2) recomiendan, debido a que
los síntomas son inespecíficos y compatibles con sepsis neonatal o neumonía, tomar los
exámenes (PCR, hemograma y cultivos) necesarios para confirmar su etiología y tratar con
antibióticos de amplio espectro hasta establecer el diagnostico definitivo; de otra parte
hay literatura(3) que teniendo en cuenta el carácter autolimitado de la enfermedad solo
recomienda administración de oxigeno con el objetivo de mantener la presión parcial de
oxigeno normal.
No se recomienda el uso de diuréticos (furosemida) en el edema pulmonar ya que no se ha
demostrado su utilidad y pueden llevar a un desequilibrio hidroelectrolítico.
En cuanto al manejo nutricional del neonato con TTRN se recomienda tener en cuenta la
frecuencia respiratoria (FR) del neonato para decidir si dar vía oral o parenteral, es así
como: si la FR es menor de 60 rpm se permite vía oral, si la FR esta entre 60-80 rpm la
alimentación debe ser por sonda nasogástrica y si la FR es mayor de 80 rpm se recomienda
dar nutrición parenteral(4).

Bibliografía:

1. W. Taeusch, R. Ballard, C. Gleason: Avery's Diseases of the Newborn. Octava Edición.


Elsevier Saunders. 2005. pag
697-699.
2. G. Avery, M. Fletcher, M. MacDonald: Neonatología, fisiopatología y manejo del recién
nacido. Quinta Edición. Editorial
Médica Panamericana. 2001. Pagina 507.
3. A. Rudoklpph. R. Collin: Pediatría de Rudolph. Mc Graw Hill Interamericana. Edición 21.
España 2004.

32.- Se trata de lactante que es traído al servicio con datos de desnutrición severa,
anemia clínica, manifestaciones purpúricas y como antecedentes llama la atención procesos
infecciosos de difícil control. La posibilidad diagnóstica es una deficiencia de:

a) Hierro.
b) B 12.
c) Folatos.
d) Piridoxina.

La anemia megaloblástica es un tipo de anemia en donde existe una disminución de la


síntesis del ADN con detención de la maduración que compromete las tres líneas celulares
de la médula ósea (glóbulos rojos, glóbulos blancos y plaquetas). Las causas que la producen
son numerosas, pero aproximadamente el 95% de los casos es consecuencia de una
deficiencia de vitamina B 12 y/o de ácido fólico. Las manifestaciones clínicas y
hematológicas son similares en ambos casos, pero las manifestaciones neurológicas se
presentan sólo en los casos de deficiencia de vitamina B12. El tratamiento está ligado a la
causa que la produce.
ETIOLOGÍA

La anemia megaloblástica en el niño casi siempre se debe a la carencia de folatos, con


menos frecuencia a la de vitamina B12 y más raramente a defectos hereditarios de la
absorción, transporte o metabolismo de estas vitaminas; en más del 95% de los casos se
debe a carencia de estas vitaminas.

La deficiencia de folatos es relativamente frecuente en madres que presentan desnutrición


y escaso aporte de vegetales en la dieta; en estos casos con frecuencia se asocia a anemia
por deficiencia de hierro. En estas circunstancias no es posible para la madre proporcionar
un aporte suficiente de folatos al feto en rápido crecimiento o al niño durante la lactancia.

En niños mayores, la desnutrición continúa siendo la principal causa de deficiencia de


folatos. Otras causas incluyen los defectos de absorción en síndromes de absorción
intestinal deficiente, aumento de los requerimientos (especialmente en pacientes con
anemia de células falciformes y talasemia) y administración de medicamentos que funcionan
como antagonistas al inhibir la dihidrofolatoreductasa (triamtereno, sufisoxasol,
trimetoprim).

El cuadro clínico es similar tanto en la deficiencia de folatos como en la de vitamina B12. El


comienzo es insidioso y los primeros síntomas que se presentan son palidez, irritabilidad,
apatía, anorexia y debilidad.4

La deficiencia de la vitamina B12 es rara antes de los dos años de edad; algunos pacientes
presentan signos que permiten sospechar esta deficiencia: la lengua es dolorosa y adquiere
un color rojo intenso, presenta atrofia papilar y signos de glositis recidivante; con
frecuencia existe fiebre moderada y diarrea crónica o episódica. El bazo es palpable en la
mitad de los pacientes severamente anémicos.

La complicación neurológica es común en adultos y muy rara en los niños y depende


principalmente de deficiencia de la vitamina B12. Se inicia con parestesias en pies y dedos
de las manos a consecuencia de neuropatía periférica, asociadas a alteraciones sensitivas;
sin tratamiento el cuadro puede evolucionar a ataxia espástica. Es importante destacar que
la anemia megaloblástica por deficiencia de folatos no se acompaña de las alteraciones
neurológicas observadas en la deficiencia de vitamina B12.

ANEMIA MEGALOBLASTICA: Revisión bibliográfica


Jorge Gustavo Romero Valdez, Carlos Eduardo Sandoval Benetti, César Luis Sánchez
Dra. Laura Adriana Acosta
33.- Recién nacido de término que presenta reflejo de moro asimétrico a expensas de
inmovilidad del brazo derecho; el resto de los reflejos no presentan alteraciones. La
madre refiere que durante el parto presentó distocia de hombros. ¿Cuál es el diagnóstico?

a) Parálisis de Déjerine – Klumpke.


b) Parálisis de Erb – Duchenne.
c) Hemiparesia derecha.
d) Lesión de plexo braquial.

La parálisis de Erb – Duchenne se presenta en recién nacidos que nacen por parto con
distocia de hombros, provocando una hiperextensión lateral del cuello, lesionando las raíces
de C5 y C6; por lo que se manifiesta con adducción y rotación interna del brazo ipsilateral,
el reflejo del Moro es asimétrico. En caso de afectarse la raíz de C4 se agregará parálisis
frénica. En cambio, la parálisis de Déjerine - klumpke se presenta en las distocias de
nalgas, con hiperextensión del brazo, se lesionan las raíces de C7 y C8, manifestándose con
mano caída y ausencia del reflejo de prensión palmar del lado afectado; puede involucrar la
raíz de T1 originando síndrome de Horner.

Bibliografía:

1. Allieu Y, Cenac, P. Neurotization via spinal accessory nerve in complete paralisis due to
multiple avulsion injuries of the brachial plexus. Clin Orthop 1988, 237: 67-74.
2. Bennet GC, Harrrold, AJ. Prognosis and early management of birth injuries to the
brachial plexus. Br Med J 1976, I: 1520-1521.
3. Bentolilla V, Nizard R, Bizot P, Sedel L. Complete traumatic brachial plexus palsy. J Bone
Joint Surg 1999, 81A: 20-28.
4. Birch R, Bonney G, y Wynn Parry, CB. Surgical disorders of the peripheral nerves.
Edinburgh: Churchill Livingstone, 1998.
5. Birch, R. Infraclavicular lesions. En: Boome RS, ed. The brachial plexus. New York:
Churchill Livingstone, 1997, 79-88.
6. Bonnard C, Slooff ACJ. Brachial plexus lesions. Drawings of explorations and
reconstructions by Algimantas Otonas Narakas. Berlín: Springer, 1999.
34.- Una herramienta importante para realizar el siguiente diagnóstico es la triada de
Gregg. ¿A cual de las siguientes infecciones nos referimos?

a) Citomegalovirus.
b) Toxoplasmosis.
c) Rubéola.
d) Sífilis congénita.

Greeg, en 1941, fue el primero que describió la tríada característica del SRC: cardiopatía
congénita, cataratas y sordera. Posteriormente estudios en animales corroboraron los
mismos hallazgos encontrados en los fetos humanos; defectos cardiacos, oculares,
esqueléticos, SNC (caso índice) restricción del crecimiento fetal y óbito, siendo mayor el
riesgo de teratogénesis cuando la infección ocurre en el periodo de la organogénesis

La tríada de Gregg que consiste en hipoacusia neurosensorial, malfiormaciones congénitas


(PCA o estenosis pulmonar) y anomalías oculares (catarata, glaucoma, retinitis en sal y
pimienta) se presenta en la rubéola; otras manifestaciones son: corioretinitis, púrpura
trombocitopénica y microcefalia. En la infección por citomegalovirus también se presenta
microcefalia y coriorretinitis, así como calcificaciones periventriculares.

BIBLIOGRAFÍA

1.-De Santis M. Cavalliere A, Straface G, Caruso A. Rubella infection in pregnancy.


Reproductive Toxicology 2006;21:390-8.

35.- Masculino de 10 años con los siguientes antecedentes: a los 6 años y en varios análisis
se detecta hematuria microscópica, con niveles IgA normales y normocalciuria. A la edad de
9 años persistía la hematuria en los análisis e incluso habían observado algún episodio
recortado de hematuria macroscópica. Un año más tarde se detectó proteinuria moderada
de 1250 mg/24 horas. En el momento de la consulta persisten las alteraciones en el
sedimiento, pero la proteinuria es de rango nefrótico, con creatinina sérica de 1,3 mg/dl.
Existen antecedentes familiares de nefropatía evolutiva con desarrollo de insuficiencia
renal y de miopía familiar por “lenticonus”. ¿El diagnóstico más probable es?

a) Glomerulonefritis mesangial (enfermedad de Berger).


b) Enfermedad poliquística autosómica dominante.
c) Nefritis intersticial por hipersensibilidad.
d) Enfermedad de Alport.

En medicina, la enfermedad de Alport es una enfermedad genética ligada al cromosoma X,


en la que una mutación en la síntesis del colágeno afecta los riñones, oídos, y ojos causando
sordera y trastornos de la vista, incluyendo distrofia de la córnea y cataratas. Fue
inicialmente identificado por el médico británico Cecil A. Alport en 1927, que describió una
familia británica en la que muchos miembros desarrollaban enfermedades renales. Él
describió que los hombres afectados en la familia morían a causa de enfermedades renales,
mientras que las mujeres estaban menos afectadas.

Se conoce ahora que muchos casos del síndrome de Alport son causados por una mutación
en el gen del colágeno COL4A5, entre otras. Este gen codifica la cadena alfa-5 del colágeno
tipo IV y está localizado en el cromosoma X. En ciertas familias se ven plaquetas de gran
tamaño circulantes en la sangre, trombocitopenia e inclusiones leucocitarias que se parecen
a las encontradas en la anomalía de May-Hegglin

El síndrome de Alport se caracteriza por tener afección renal, coclear y ocular. La principal
señal de este síndrome es la hematuria microscópica (microhematuria). Los hombres con el
síndrome Alport ligado al cromosoma X (XLAS) padecen microhematuria desde una edad
muy temprana. Alrededor del 90% de mujeres con XLAS también la tienen. Hay 2 métodos
para el diagnóstico clínico: secuenciación y análisis de deleción/duplicación. El análisis de
secuenciación de COL4A5 identifica cerca del 80% de las mutaciones de individuos
afectados con antecedentes familiares en herencia ligada al X. El análisis de
deleción/duplicación del gen COL4A5 identifica deleciones (típicamente multiexónicas)
cercanas al 10% de individuos afectados con antecedentes familiares ligada al X.

Referencias
Behrman RE, Kliegman RM, Jenson HB. Nelson Textbook of Pediatrics. 17th ed.
Philadelphia, Pa: Saunders; 2003.

36.- Femenino que cursa con diagnóstico de Salmonella Typhy. ¿Cuál de los siguientes
cultivos permite aislar con mayor frecuencia éste agente causal?

a) Hemocultivo.
b) Coprocultivo.
c) Mielocultivo.
d) Urocultivo.

DIAGNOSTICO DE LABORATORIO

Dadas las variadas manifestaciones clínicas de las salmonelosis, la confirmación del


diagnóstico de estas infecciones, requiere de métodos microbiológicos que permitan el
aislamiento o identificación del agente causal o de pruebas serológicas que facilitan
reconocer anticuerpos específicos presentes en el suero de los pacientes.

Hemocultivo: Es el procedimiento de elección, cuando se realiza apropiadamente y en


medios selectivos a base de bilis. Coincidiendo con la fisiopatología de la infección, son
positivos especialmente durante la primera semana de la infección; se calcula que al final de
la tercera semana de positividad solamente alcanza un 50%.

Mielocultivo: El cultivo del aspirado de médula ósea se considera como el mejor método
para el aislamiento de salmonella en los pacientes con fiebre tifoidea y paratifoidea.
Aunque el procedimiento produce una molestia transitoria, en general es bien tolerado y los
cultivos son más rápidamente positivos. Se recomienda sea practicado por personal con
experiencia. Pueden ser positivos aún cuando los hemocultivos sean negativos.

Coprocultivo: Puede ser positivo desde el comienzo de la infección, aunque su máxima


positividad en la infección aguda, se observa durante la tercera semana. Es particularmente
útil para el control postratamiento de los pacientes y para detectar los portadores
crónicos.

Cultivo de bilis duodenal: Obtenido por aspiración o utilizando la técnica que lleva un
dispositivo en cápsulas de gelatina. No es superior al hemocultivo y con certeza no supera a
la asociación del hemocultivo con el coprocultivo.

Urocultivo: Su valor diagnóstico es muy limitado pues la bacteriuria no es continua. Su


máxima positividad está en la tercera semana. La Salmonella también puede ser aislada de
otros productos como las manchas rosadas o reoseolas tíficas, de la secreción bronquial,
del líquido articular, etc.

DIAGNOSTICO SEROLOGICO

Reacción de seroaglutinación (Widal): es de poco valor como prueba diagnóstica. En la


infección no tratada sólo cerca del 50% de los pacientes pueden tener un aumento
significativo de las aglutininas contra el antígeno "O", en algún momento de la enfermedad.
Las aglutininas contra el antígeno "H" no tienen valor diagnóstico aunque puedan observarse
títulos elevados de ellas.

En muchos casos de fiebre tifoidea no hay elevación de los títulos de aglutininas durante el
curso de la infección y en ocasiones se pueden observar elevaciones no específicas, debido
a reacciones cruzadas.

Diagnóstico inmunoenzimático: la detección de anticuerpos IgM e IgG contra el


lipopolisacarido por técnica ELISA aún no está disponible para uso rutinario.

Con fines de investigación se han utilizado otras pruebas dentro de las cuales están la
reacción de polimerasa en cadena (PCR), las pruebas de fagotipificación, las de
susceptibilidad antimicrobiana y la investigación del perfil plasmídico de algunas cepas. En
los estudios epidemiológicos se usan las pruebas de fagotipificación, de susceptibilidad
contra los antimicrobianos y el perfil plasmídico, las cuales han demostrado ser útiles y
complementarias para el estudio de cepas aisladas de alimentos, o de aguas contaminadas, y
en brotes de salmonelosis en los cuales se requiere establecer una fuente común de
infección.
LECTURAS RECOMENDADAS

1.-Borrego JJ, Castro D, Jiménez Notorio M, Luque A, Rodríguez Avial C, Picazo JJ.
Comparison of epidemiological markers of salmonella strains isolated from differents
sources in Spain. J Clin Microbiol 30: 3058, 1992

2.-De Los Rios O, Restrepo J, Carvajal CD. Salmonelosis: Revición de conceptos.


Comportamiento epidemiológico en Antioquia. Bol Epidenmol Antioquia 7:19

3.- Edelman R, Levine MM. Sumary of an international workshop on typhoid fever. Rev Inf
Dis 8:329, 1996.

4.-Prada G. Infecciones por especies de Salmonella. En: Medicina Interna. Chalem,


Escandón, Campos, Esguerra editores. Fundación Instituto de Reumatología e Inmunología.
Editorial Presencia. Santafé de Bogotá.

Tacket C. Molecular epidemiology of salmonella. Epidemiologic Reviews 11:99.

37.- Masculino de 47 años de edad que cursa con un ataque agudo de gota, la articulación
que se afecta con mayor frecuencia en éste caso es:

a) La rodilla.
b) La muñeca.
c) La primera metatarso-falángica.
d) El codo.

El American College of Rheumatology tiene 11 criterios, y la presencia más o menos de seis


sugiere la presencia de gota. Los 11 criterios son:

1. Más de un ataque de artritis activa


2. Inflamación máxima desarrollada a lo largo de un día.
3. Ataque de oligoartritis.
4. Enrojecimiento observado en la articulación.
5. Primera articulación metatarsofalángica dolorosa o inflamada
6. Ataque unilateral de la primera articulación metatarsofalángica
7. Ataque unilateral de la articulación del tarso
8. Tofo (probado o sospechado)
9. Hiperuricemia
10. Hinchazón asimétrica en la radiografía dentro de una articulación
11. Terminación completa de un ataque.

Bibliografía:
• Coll JM, Blanch J. Hiperuricemia y gota. Aspectos terapéuticos. Jano
1997;1201:47-52.
• González Barber A, Carlavilla AB. Guía en Hiperuricemia y Gota. Madrid:
• EDIMSA, 2003.
• Nadal A. Farmacovigilancia. Alopurinol: aumento en la incidencia de episodios agudos
de gota. Offarm 2000;19(1):128.
• Pérez Ruiz F, Calabozo Raluy M, Ugalde Espiñeria J, Herrero Beites AM.
• Artropatías microcristalinas I. Hiperuricemia y gota. Medicine, 2001;8(34):1765-
72.
• Sánchez Pozo A, Faus MJ. Hiperuricemia y gota. Pharm Care Esp 2003; 5:105-9.
• Sancho Bueso T, Bernardino de la Serna I, García Puig J. Consulta diaria.
• ¿Qué haría usted ante un paciente con hiperuricemia? Medicina Integral
2000;3(35):100-11.
• Sancho Bueso T, García Puig J. Criterios terapéuticos ante la hiperuricemia.
• Revista Clínica Española 2001;2(201):85-7.

38.- Femenino de 32 años tratado actualmente por padecer artritis por fiebre reumática,
la articulación que se afecta con mayor frecuencia en este caso es:

a) El codo.
b) La muñeca.
c) La cadera.
d) La rodilla.

La artritis es generalmente la manifestación inicial de la fiebre reumática, aunque se puede


presentar antes una carditis asintomática. Una o ambas rodillas se afectan en el 76% de
los casos aproximadamente y uno o ambos tobillos en el 50% de los casos.

Gibofsky A, Zabriskie J. Acute rheumatic fever and poststreptococcal arthritis. Harris ED


Jr, Budd RC, Genovese MC, Firestein GS, Sergent JS, Sledge CB, Ruddy S, editors. Kelley’s
Textbook of Rheumatology. Philadelphia: Elsevier Saunders; 2005. p. 1684-96.

39. - Characteristic to be met by subjects in a cohort study.

a) Having the disease under study.


b) Have a family history of the disease under study.
c) They are adults with multiple illnesses to study.
d) Not having the disease under study.

El estudio de cohorte es el diseño ideal para buscar la causa de una enfermedad por lo que
compara a un grupo de sujetos, libres de la enfermedad en estudio expuestos a los
factores de sospecha con otro grupo de sujetos también libres de la enfermedad, que no
estén expuestos a los factores sospechosos.

Ruiz M. A. Epidemiología Clínica, 1ª. Ed. 2004; Pág.: 279,280

40.- El signo clínico que indica irritación del nervio ciático como componente de una
lumbalgia se denomina:

a) signo de Lassegue.
b) signo de Galeazzi.
c) signo de McMurray.
d) signo de Filkestein.

La lumbalgia es una patología muy frecuente en nuestro medio, con una enorme
repercusión sanitaria y sociolaboral. Con la edad, se inicia una cascada degenerativa en la
columna lumbar, que comienza en el disco intervertebral, continuando por las facetas
articulares y demás elementos vertebrales. Esta degeneración del raquis forma parte del
envejecimiento normal del individuo, aunque en ocasiones puede causar dolor y/o
alteraciones neurológicas. Para comprender la fisiopatología del dolor lumbar, será
preciso conocer que la inervación de la columna lumbar se hace fundamentalmente por
tres ramos nerviosos: ramo dorsal de los nervios espinales lumbares (o rami dorsal),
nervio sinuvertebral de Luschka y ramos ventrales de la cadena simpática. Existen dos
tipos de patrones de dolor en la columna lumbar: el dolor irradiado y el dolor referido. La
cascada de la degeneración consta en tres estadios. El primer estadio sería la disfunción.
El anillo fibroso se fisura y pierde la capacidad de contener al núcleo pulposo. Esto
ocasiona primero el síndrome de disrupción discal y, si el núcleo supera el contorno del
annulus, las hernias discales. El segundo estadio de Kirkaldy-Willis es el de inestabilidad.
En este estadio la movilidad en el segmento móvil aumenta de forma patológica.

Maniobra de Lassègue: Es la más importante, es casi patognomónica.


Con el paciente en decúbito supino se levanta la pierna extendida. Se considera positiva
cuando aparece un dolor intenso a nivel lumbar, en la pierna o en ambos niveles, entre los
30º-75º de flexión en la cadera, debido al estiramiento del nervio ciático. Pasados los 70º
puede aparecer un falso positivo (dolor también en sujetos sanos) por la distensión de los
isquiotibales. Para descartarlo realizaremos otras maniobras diagnósticas.

Bibliografía:

1. Andersson GBJ. Epidemiologic features of chronic low-back pain. Lancet. 1999;354:581-


5.
2. Estudio EPISER. Sociedad Española de Reumatología. 2000.
3. Herrera Rodríguez A, Rodríguez Vela J. Estenosis de canal lumbar. Rev Ortop
Traumatol. 2002;4:351-72.
4. MacGregor AJ, Andrew T, Sambrook PN, Spector TD. Structural, psychological, and
genetic influences on low back and neck pain: a study of adult female twins. Arthritis
Rheum. 2004;51:160-7.

5. Kim KS, Yoon ST, Park JS, Li J, Park MS, Hutton WC. Inhibition of proteoglycan and
type II collagen synthesis of disc nucleus cells by nicotine. J Neurosurg Spine.
2003;99:291-7.
6. Fujiwara A, Tamai K, Yamato M, An HS, Yoshida H, Saotome K, et al. The relationship
between facet joint osteoarthritis and disc degeneration of the lumbar spine: an MRI
study. Eur Spine J. 1999;8:396-401.
7. Eyre DR, Muir H. Types I and II collagens in intervertebral disc. Interchanging
radialdistributions in annulus fibrosus. Biochem J. 1976;157:267-70.56.-5.

41.- Es invierno y es enviado de la guardería, lactante de 6 meses que inicia cuadro hace 48
hrs, con febrícula y algún vómito ocasional, hoy inicia con evacuaciones blandas al principio
y más tarde se hacen líquidas, incrementándose los vómitos. Refiere la madre que en la
guardería ha habido otros niños con un cuadro similar. ¿Cuál es la etiología más probable en
este caso?:

a) Yersinia enterocolítica.
b) Campylobacter yeyuni.
c) Rotavirus.
d) Adenovirus.

Rotavirus es un género de virus perteneciente a la familia Reoviridae. Se han identificado


siete grupos, tres de los cuales (Grupo A, B y C) infectan a los humanos. El grupo A es el
más común y el más esparcido, causando el 90% de las infecciones.

Estos causan vómito y diarrea y son los más comúnmente causantes de diarrea severa en los
infantes, ocasiona aproximadamente 55,000 hospitalizaciones cada año en los Estados
Unidos y mata a cerca de 600,000 niños cada año en países en vías de desarrollo. Nuevas
vacunas han mostrado ser efectivas y seguras en el 2006. Virtualmente todos los niños
menores de cinco años han sido infectados por algún rotavirus. Rotavirus es la causa más
frecuente de gastroenteritis en niños de guardería y de gastroenteritis nosocomial en
niños pequeños. En líneas generales es el motivo más frecuente de gastroenteritis en niños
menores de dos años (4 a 24 meses de vida) y de deshidratación y muerte en los países en
vías de desarrollo

Se transmiten por la ruta fecal oral, infectando células del intestino delgado y produciendo
una enterotoxina, provocando una gastroenteritis que puede llevar a una diarrea e incluso
deshidratación. Aunque fueron descubiertos en 1973 y son responsables de más del 50% de
los ingresos hospitalarios de niños con diarrea severa, siguen siendo subestimados por la
comunidad médica, sobre todo en los países en vías de desarrollo.

Los rotavirus propician gastroenteritis aguda y fuerte dolor abdominal. "Diarrea infantil",
"diarrea invernal", "infección no bacterial aguda" y "gastroenteritis viral aguda" son los
otros nombres con los que se denomina a este padecimiento. La dosis infectante se
presume que es de 10-100 partículas virales infecciosas, ya que una persona con rotavirus
frecuentemente excreta una gran cantidad de partículas virales: en el orden de (108-1010
particulas infecciosas /ml de heces). La vía de contagio se da a través del contacto con
manos, objetos o utensilios contaminados. El período de incubación de la enfermedad por
rotavirus es de aproximadamente 2 días. La enfermedad está caracterizada por vómito y
diarrea acuosa de 3 a 8 días, y fiebre con dolor abdominal ocurre con frecuencia. La
inmunidad se produce después de la infección. Infecciones posteriores tienden a ser menos
severas que la infección original.

Una micrografía electrónica de Rotavirus: obsérvese el aspecto de «rueda» del virión

BIBLIOGRAFÍA RECOMENDADA:

American Academy of Pediatrics, Committee on Infectious Diseases. Prevention of


rotavirus disease: guidelines for use of rotavirus vaccine. Pediatrics 1998; 102: 1483-91.
Bernstein D. J Infect Dis 2002; 186: 1487-1489.

Centers for Diseases Control and Prevention. Rotavirus vaccine for the prevention of
rotavirus gastroenteritis among children: recommendations of the Advisory Committee on
Immunization Practices (ACIP). MMWR 1999; 48: 1-23.

Chang EJ, Zangwill KM, Lee H, Ward JI. Lack of association between rotavirus infection
and intussusception: implications for use of attenuated rotavirus vaccines. Pediatr Infect
Dis J 2002; 21: 97-102.

42.- Recién nacido con sospecha de menigoencefalitis por clínica y un líquido


cefalorraquídeo con aumento de leucocitos, aumento de proteínas e hipoglucorraquia en el
que no se tiene aún un germen aislado. El tratamiento de elección es:

a) Dicloxacilina y amikacina.
b) Ampicilina y amikacina.
c) Penicilina sódica cristalina.
d) Vancomicina.

La meningoencefalitis es definida como la inflamación de las meninges y el encéfalo. Dentro


de las causas de la meningoencefalitis purulenta la más frecuente es la bacteriana.
La etiología varía dependiendo del grupo de edad, en menores de 2 meses el más común es
Streptococcus del grupo b, después de esta edad el más frecuente es el S. pneumoniae. El
cuadro clínico se caracteriza por 4 síndromes: infeccioso, de hipertensión endocraneana,
meníngeo y de daño neuronal. Las complicaciones pueden ser agudas, subagudas y crónicas.
El diagnóstico se hace con el estudio citoquímico y cultivo del LCR. Las medidas
terapéuticas irán encaminadas al manejo de los signos y síntomas componentes de los 4
síndromes. El tratamiento específico se iniciará en forma empírica y se modificará de
acuerdo a los resultados del cultivo.

En la meningitis bacteriana el aspecto macroscópico LCR es turbio o incluso purulento


debido al mayor contenido de células y proteínas. En la mayoría de los casos de
meningoencefalitis bacteriana el número de leucocitos es superior a 500 x mm3; y el
predominio de células polimorfonucleares es mayor a 50%.
La presencia de hipoglucorraquia (< 50% de la glicemia central o glucorraquia < 40 mg/dL) e
hiperproteinorraquia moderada (entre 200 y 500 mg/dL) sugiere etiología bacteriana. Sin
embargo, en casos con cuadros neurológicos de evolución más prolongada, mayor de 10 días,
el diagnóstico diferencial debe establecerse con meningoencefalitis de etiología
tuberculosa.
Enfermedades infecciosas y microbiología
Número number 1 enero-marzo
January-march 2002 volumen volume 22
Meningoencefalitis bacteriana
Fortino solórzano santos,* maría guadalupe miranda novales,**
Rita d díaz ramos*

43.- Un niño de 12 años en Yautepec Morelos, se encuentra dormido sobre una toalla a la
orilla de una alberca, súbitamente presenta dolor intenso en muslo derecho y se observa
una pequeña pápula eritematosa El niño refiere sensación de ardor intenso en el muslo, su
madre aplica una pomada con antihistamínico y le administra un antihistamínico oral.
Dos horas después el paciente inicia parestesias, nausea y vómito, comienza a presentar
dolor abdominal intenso, en episodios, síntomas que en una hora se hacen más intensos, la
causa más probable de este cuadro es:

a) Picadura de viuda negra.


b) Picadura de alacrán.
c) Picadura de cara de niño.
d) Picadura de nauyaca

El veneno que inocula es 15 veces más potente que el veneno de una serpiente de cascabel y
puede condicionar efectos sistémicos graves e incluso la muerte.

El primer síntoma generalmente es un dolor similar a una punción con un alfiler y la


sensación se experimenta cuando realmente se ha efectuado la picadura de la araña. Sin
embargo, es posible que algunas personas no lo sientan. Puede haber hinchazón y
enrojecimiento leve y una lesión en forma de diana.

De 15 minutos a una hora más tarde, un dolor muscular sordo se irradia desde el área de la
picadura a todo el cuerpo.

Síntomas
Náuseas
Ataque al estado general
Diaforesis
Contracturas musculares
Dolor muscular
Retención urinaria
Estreñimiento
Taquicardia
Insuficiencia cardiaca
Hipertensión arterial
Inquietud
Ansiedad
Sensación de muerte
Inminente

Current Pediatric Diagnosis and Treatment 17 Ed Mc Graw Hill. Pag 346 . 2005

44.- La indicación quirúrgica es obligada en un paciente afectado de enterocolitis


necrotizante cuando presente uno de los siguientes signos clínicos:

a) Heces mucosanguinolentas.
b) Vómitos biliosos.
c) Distensión abdominal.
d) Neumoperitoneo.

Enfermedad adquirida que afecta principalmente a RN prematuros o patológicos y que se


caracteriza por necrosis de la mucosa o de capas incluso más profundas del intestino, sobre
todo en el íleon terminal y con menos frecuencia del colon y del intestino delgado proximal.
Síntomas, signos y diagnóstico

La enfermedad puede iniciarse con un íleo que se manifiesta con distensión abdominal,
residuos gástricos biliosos (tras las tomas) que pueden progresar a vómitos de bilis o
presencia de sangre macroscópica o microscópica en las heces. La sepsis puede ponerse de
manifiesto con letargia, inestabilidad térmica, aumento de las crisis de apnea y acidosis
metabólica.

La detección sistemática de sangre oculta o de sustancias reductoras en las heces de los


prematuros (que han recibido alimentación oral o enteral) puede ayudar a diagnosticar la
ECN. Las radiografías iniciales pueden ser inespecíficas o mostrar sólo el íleo. Sin embargo,
un asa intestinal fija y dilatada que no cambia en las radiografías posteriores indica una
ECN. Las radiografías diagnósticas son las que muestran neumatosis intestinal y gas en el
territorio de la vena porta. El neumoperitoneo es un signo de perforación intestinal e indica
la necesidad urgente de una intervención quirúrgica.

BOL PEDIATR 2006; 46(SUPL. 1): 172-178


Protocolos de Neonatología
Enterocolitis necrotizante neonatal
I. FERNÁNDEZ JIMÉNEZ1, I. DE LAS CUEVAS TERÁN2

45.- Femenino de año y medio de edad refiere la madre que inició con tos seca, rinorrea
hialina, temperatura axilar de 37.8°C. Por la tarde muestra tos intensa y en accesos, dolor
subesternal al toser, estridor inspiratorio; al explorarle se le observa pálido, con aleteo
nasal, hundimiento intercostal bilateral a la inspiración y taquicardia. Faringe hiperémica
roja; en tórax se escuchan algunos estertores gruesos diseminados en ambos hemitorax y
disminución del murmullo vesicular. Usted sospecha de:

a) Bronquiolitis.
b) Asma bronquial.
c) Neumonía bilateral.
d) Laringotraqueobronquitis.

La laringotraqueobronquitis también conocida como laringitis subglótica o crup vírico, es


una enfermedad infecciosa aguda de etiología viral que afecta fundamentalmente a la
laringe y a las estructuras que se localizan por debajo de las cuerdas vocales, en donde los
tejidos blandos subglóticos inflamados producen obstrucción de la vía aérea en grado
variable. Este padecimiento afecta con mayor frecuencia al grupo de edad comprendido
entre seis meses y tres años, con un pico de incidencia a los dos años de edad; no hay
predominio de sexo, y se presenta más frecuentemente en otoño e invierno.

CUADRO CLÍNICO

El cuadro inicial es el de una rinofaringitis e incluye irritación nasal, coriza, fiebre


generalmente menor de 39 °C, tos seca y odinofagia, con poca o nula afección del estado
general; 24 a 48 horas después aparece disfonía, la tos se hace "crupal" (traqueal, perruna
o en ladrido) y se presentan en forma gradual estridor inspiratorio creciente (estridor
laríngeo) y signos de dificultad respiratoria de intensidad variable. En la exploración física
son evidentes la disfonía, el estridor laríngeo que en ocasiones es audible a distancia, la
disminución del murmullo vesicular y datos de dificultad respiratoria.1-6

Forbes describe la progresión del cuadro clínico de la si guiente manera:

Etapa 1: Fiebre, ronquera, tos crupal y estridor inspiratorio al molestar al paciente.

Etapa 2: Estridor respiratorio continuo, participación de músculos accesorios de la


respiración con retracción de costillas inferiores y de tejidos blandos del cuello.

Etapa 3: Signos de hipoxia e hipercapnia, inquietud, ansiedad, palidez, diaforesis y


taquipnea.

Etapa 4: Cianosis intermitente, cianosis permanente, paro respiratorio.

Por lo general la enfermedad es leve, en pocas ocasiones progresa más allá de la etapa 1,
más del 95% reciben tratamiento ambulatorio; del 5% que requiere hospitalización sólo del
1 al 1.5% requieren medidas de apoyo ventilatorio (intubación endotraqueal o
traqueostomía).

La duración del padecimiento es muy variable, desde tres a siete días en casos leves, hasta
siete a catorce días en casos graves

PAC Infecciones de Vías Respiratorias en Niños Parte B Libro 2.


46. - In a male patient in determining non-pathological short stature, the most common
cause is:

a) Hypothyroidism.
b) Malnutrition.
c) Nephropathy.
d) Family.

La talla constituye un indicador muy útil del estado de salud de un niño, por lo cual una
desviación significativa del canal de crecimiento normal puede ser la primera manifestación
de una alteración clínica de importancia. Junto con la talla de un niño, es importante
evaluar su velocidad de crecimiento, parámetro más sensible ya que permite detectar
cambios en el canal de crecimiento que habitualmente preceden a los cambios en la talla
absoluta. La velocidad de crecimiento debe establecerse sobre períodos no menores a 6
meses para ser confiable, restando particular atención a las desviaciones significativas y
no a aquellas menores, que pueden observarse en niños normales.

Se define como talla baja aquella ubicada bajo el percentil 3 o bajo 22 desviaciones
estándar de las curvas de NCHS. Estas curvas, de origen norteamericano, tienen utilidad
para pacientes pertenecientes a estratos socioeconómicos medios y altos, pero tienen
menos utilidad para evaluar a niños que pertenecen a estratos socioeconómicos bajos en
nuestro país. En general ameritan estudio aquellos pacientes cuya talla se encuentra bajo
el percentil 3, o cuya velocidad de crecimiento se ha deteriorado en forma significativa
incluso antes de ubicarse 22 desviaciones estándar bajo la media.

Las causas de talla baja se clasifican en:

1. Variantes normales: talla baja familiar y retraso constitucional.

2. Trastornos primarios del crecimiento como displasias esqueléticas, trastornos del


desarrollo intrauterino y anormalidades cromosómicas.

3. Alteraciones del crecimiento secundarias a nutrición inadecuada, enfermedades crónicas


(como síndrome de malabsorción, insuficiencia renal, alteraciones pulmonares o cardíacas),
y enfermedades endocrinológícas (como hipotiroidismo, alteraciones del eje somatotráfico,
síndrome de Cushing, o raquitismo).

Causas de talla baja

· Familiar
·Retardo constitucional
·Retardo del crecimiento intrauterino
· Alteraciones nutricionales
· Enfermedades sistémicas
· Alteraciones cromosómicas
· Displasias óseas
· Deprivación emocional - maltrato infantil
· Trastornos endocrinos

REFERENCIAS

1. Mahoney CP: Evaluating the child with short stature. Pediatr Clin North Am
1987; 34: 825-48.

2. Mericq V, y Cassorla F: Sistema hormona del crecimiento-efector y su rol en


el crecimiento infantil. Rev Chil Pediatr 1997; 68: 27-37.

3. Rosenfeld R, Albertsson- Wikland K, Cassorla F, et al: Diagnostic controversy,


the diagnosis of childhood growth hormone deficiency revisited. J Clin End

4. Codner E, Mericq V, Ugarte F, et al: Utilidad de la determinación del factor de


crecimiento insulino símil tipo 1 y de su proteína ligante tipo 3 en el diagnóstico de la
deficiencia de hormona de crecimiento en niños. Rev Méd Chile 1999; 127: 807-13.

47.- La hemólisis en la isoinmunización materno- fetal al Rh esta dada por:

a) IgA
b) IgE
c) IgG
d) IgM

Hemólisis por isoinmunización: Etiología

La exposición materna (por transfusión de sangre, hemorragia feto-materna, amniocentesis


o aborto) a antígenos extraños de la sangre fetal lleva a la producción y paso
transplacentario de anticuerpos IgG específicos que pueden destruir los glóbulos rojos
fetales.

La Ig G materna atraviesa la placenta. Si los anticuerpos están dirigidos contra antígenos


presentes en el eritrocito fetal, se fijan a la membrana eritrocitaria y transforman a los
hematíes en células blanco que son eliminadas por los macrófagos tisulares.

Desde que se inició la profilaxis con gammaglobulina anti-D para prevenir la isoinmunización
anti-D la causa mas frecuente de enfermedad hemolítica en el recién nacido es la
isoinmunización ABO.

Osorio A. IsoinmunizaciÛn Rh: Manejo de enfermedad hemolÌtica fetal severa. Rev Chil
Ultrasonog. Volumen 2006: 9: 130-137.

48.- Lactante de un año que inició con un cuadro diarréico prolongado, recibió tratamiento
con antibióticos, se reportó con mejoría clínica, posteriormente reinició con el cuadro
enteral ¿El agente causal más probable en la recaída es?

a) Escherichia Coli
b) Clostridium Difficile
c) Shigela Flexneri
d) Clostridium Perfirgens.

Clostrirum difficile es un bacilo anaerobio gram positivo, responsable de la colitis asociada


a antibióticos. Coloniza el tracto digestivo después de que su flora habitual haya sido
alterada por el tratamiento antibiótico.
Tras la colonización, produce dos exotoxinas (toxina A y toxina B) que se unen al los
receptores de las células epiteliales intestinales, conduciendo a la producción de una
diarrea secretora.
Es una de las infecciones nosocomiales más comunes, con una elevada morbimortalidad.
La transmisión es fecal-oral y ha producido importantes epidemias dentro del ambiente
hospitalario.
No todas la cepas de Clostridium son productoras de enterotoxinas, en ocasiones se
comportan como colonizantes del tracto digestivo sin producir enfermedad.

Factores de riesgo:
• Uso de antibioterapia: prácticamente todos las antibióticos pueden predisponer a la
colonización por Clostridium difficile.
• Asociación frecuente: Ampicilina, Amoxicilina, Cefalosporinas y Clindamicina
• Asociación ocasional: otras Penicilinas, Sulfamidas, Eritromicina, Trimetropim y
Quinolonas.
• Raramente o nunca asociados: Aminoglucósidos, Tetraciclinas, Cloramfenicol,
Metronidazol y Vancomicina.
• Pacientes con edad avanzada y debilitados.
• Intervención quirúrgica sobre tracto gastrointestinal.
• Contacto previo con paciente colonizado.
• Alimentación enteral.

Manifestaciones clínicas:
El espectro clínico abarca desde portadores asintomáticos hasta enfermedad fulminante
con megacolon tóxico. Los síntomas más frecuentes son diarrea acuosa, dolor abdominal y
fiebre.
Las manifestaciones clínicas se inician entre 5-10 días después de la administración de
antibióticos, aunque pueden aparecer el primer día o hasta 10 días después de haber cesado
el tratamiento antibiótico.
Colitis:
• Diarrea profusa (más de 10 deposiciones al día)
• Presencia de leucocitos en heces.
• Fiebre, nauseas, anorexia, leucocitosis con desviación a la izquierda.
• Dolor y distensión abdominal.
• Colitis parcheada difusa en la sigmoidoscopia.

Colitis pseudomembranosa:
• Mayor número de deposiciones.
• Presencia de leucocitosis y sangre en heces.
• Fiebre, nauseas, anorexia, leucocitosis con desviación a la izquierda.
• Más marcada distensión abdominal.
• Placas adheridas, amarillentas de 2-10 mm en la sigmoidoscopia

BIBLIOGRAFIA:
• Bartlett JG, Moon N, Chang TW, et al. The role of Clostridium difficile in antibiotic-
associated pseudomembranous colitis. Gastroenterology 1978; 75:778.
• Thomas C, Stevenson M, Riley TV. Antibiotics and hospital-acquired Clostridium difficile-
associated diarrhea: a systematic review. J Antimicrob Chemother 2003; 51:1339.
• Tedesco FJ. Pseudomembranous colitis: pathogenesis and therapy.
Med Clin North Am 1982; 66:655.
• Fekety R, Shah AB. Diagnosis and treatment of Clostridium difficile colitis. JAMA 1993;
269:71.

49.- Se trata de paciente de tres meses de edad el cual desde hace aproximadamente 3
semanas presenta episodios intermitentes de distensión abdominal, dolores de tipo cólico y
algunos vómitos. Tendencia al estreñimiento. Entre sus antecedentes personales hay que
destacar que fue prematuro, pesó 900gr. al nacimiento y tuvo dificultad respiratoria
importante que precisó ventilación asistida durante 15 días. El diagnóstico más probable en
es paciente es:

a) Megacolon congénito.
b) Estenosis cólica secundaria a Enterocolitis necrotizante.
c) Enteritis crónica por rotavirus.
d) Vólvulo intestinal intermitente.

La enterocolitis necrotizante (ECN) es una enfermedad grave que afecta a recién nacidos,
en especial prematuros, con una incidencia y morbimortalidad elevados. Constituye la
urgencia gastrointestinal más frecuente en las UCI neonatales.
Se presenta como un síndrome gastrointestinal y sistémico que comprende síntomas
variados y variables, como distensión e hipersensibilidad abdominal, sangre en heces,
intolerancia a la alimentación, apnea, letargia, y en casos avanzados acidosis, sepsis, CID y
shock.

El síndrome clínico ha sido clasificado en estadios por Bell y col. (1978) y modificado
por Walsh y Klegman (1986) para incluir hallazgos sistémicos, intestinales y
radiológicos.
• A. Estadio I : sospecha de enterocolitis necrotizante
• Los hallazgos sistémicos son inespecíficos.
• Los hallazgos intestinales incluye el residuo gástrico y heces guayaco –positivas.
• Los hallazgos radiológicos son normales e inespecíficos.
• B. Estadio II A: enterocolitis necrotizante leve
• Los hallazgos sistémicos son similares al estadio I.
• Los hallazgos intestinales incluyen distensión abdominal prominente con
hipersensibilidad a la palpación o sin ella, ruidos hidroaéreos ausentes, sangre
macroscópica en materia fecal.
• Los hallazgos radiológicos, íleo con asas dilatadas con áreas focales de neumatosis
intestinal.

• C. Estadio II B: enterocolitis necrotizante moderada


• Los hallazgos sistémicos incluyen acidosis leve y trombocitopenia
• Los hallazgos intestinales incluyen edema de la pared abdominal e hipersensibilidad
a la palpación con una masa palpable o sin ella.
• Los hallazgos radiológicos incluyen neumatosis extensa y ascitis temprana.
• Puede haber gas en la vena porta intrahepática.
• D. Estadio IIIA: enterocolitis necrotizante avanzada:
• Los hallazgos sistémicos incluyen acidosis respiratoria y metabólica, ventilación
asistida por apnea, hipotensión arterial, oliguria, neutropenia y coagulación intravascular
diseminada.
• Los hallazgos intestinales incluyen edema que disemina, eritema e induración del
abdomen.
• Los hallazgos radiológicos incluyen ascitis prominente y asa centinela persistente
sin perforación.
• E. Estadio IIIB: enterocolitis necrotizante avanzada:
• Los hallazgos sistémicos revelan signos vitales e índices de laboratorio en
deterioro, síndrome de shock y desequilibrio electrolítico.
• Los hallazgos intestinales y radiológicos muestran evidencias de perforación.

Hallazgos radiológicos en la radiografía de abdomen


• Distensión abdominal generalizada
• Íleo paralítico
• Neumatosis intestinal quistoide (patognomónico)
La neumatosis intestinal quistoide se la puede visualizar de varias maneras, cúmulos de gas
lineales, curvilíneos, esponjosos y espumosos.
Este último debe distinguirse de materia fecal o meconio mezclado con aire.
Cualquier lactante con sospecha de enterocolitis necrotizante en el que se encuentren
radiográficamente colecciones de aire lineales, curvilíneas esponjosas o espumosas
debe considerarse que tiene neumatosis intestinal quistoide hasta que se demuestre lo
contrario.
La neumatosis intestinal quistoide suele verse con mayor frecuencia en el colon, pero puede
ocurrir desde estomago hasta recto.

Se observa neonato con marcada distensión abdominal, con acidosis metabólica,


letargo y edema de la pared abdominal (a)

1. Hartmann G. E., Drugas G. T., Shochat S. J. Post-necrotizing enterocolitis


strictures presenting with sepsis of perforation: risk of clinical observation. J.
Pediatr. Surg. 1988; 23: 562-6.
2. Kosloske A. M., Burstein J., Bartow S. A. Intestinal obstruction due to colonic
stricture following neonatal necrotizing enterocolitis. Ann Surg. 1980 Aug;192 (2):
202-7.
3. Schwartz M. Z., Hayden C. K., Richardson C. J., Tyson K. R., Lobe T. E. A
prospective evaluation of intestinal stenosis following necrotizing enterocolitis. J.
Pediatr. Surg. 1982 Dec; 17 (6): 764-70.
4. Bell M. J., Ternberg J. L., Askin F. B. Intestinal stricture in necroting enterocolitis.
J. Pediatr. Surg. 1976; 11: 319-27.
5. Pokorny W. J., Harr V. L., McGill, C. W., et al; Intestinal stenosis resulting from
necrotizing enterocolitis. Am J. Surg 1981 42: 721-724.
6. Schimpl G., Hollwarth M. E., Fotter R., Becker H. Late intestinal strictures
following successful treatment of necrotizing enterocolitis. Acta Paediatr. Suppl.
1994; 396: 80-3.
7. Bütter A., Flageole H., Laberge J. M. The Changing face of Surgical Indication for
Necrotizing Enterocolitis J. Pediatr. Surg. 2002; 37: 469- 499.
8. Gobet R. , Sacher P. , Schwobel M. G. Surgical procedures in colonic strictures
after necrotizing enterocolitis. Acta Paediatr. Suppl. 1994;396:77-9.
50.- ¿Qué eventos adversos podemos esperar por la aplicación de Difteria
PertusisTetanos?

a) Adenitis supuras y encefalitis.


b) Parálisis flácida aguda.
c) Episodio hipotónico e hiporreactivo y llanto incontrolable y persistente por más
de 3 horas.
d) Exantema maculopapular y fiebre.

Los sistémicos se presentan dentro de las 48 horas después de la vacunación, se han


notificado (fiebre en el 40% de los vacunados, en el 5 % llanto persistente e incontrolable
por más de tres horas, somnolencia, irritabilidad y malestar general. En menos del 3%
cefalea, convulsiones, calosfrío, mialgias y artralgias.

Committee on Infectious Diseases 2006. Red Book 2006, 27th Ed.


American Academy of Pediatrics; 2006.

51.- Realiza usted el diagnóstico de de raquitismo grave asociado a alopecia, este dato le
hace pensar en:

a) Déficit funcional del receptor de vitamina D.


b) Tubulopatía primaria asociada.
c) Déficit de 25 hidroxilasa hepática.
d) Déficit de 1 alfa hidroxilasa renal.

El raquitismo resistente a la vitamina D, también conocido como Tipo I, se hereda en


forma autosómica recesiva y corresponde a un defecto congénito de la hidroxilación de la
vitamina D, hay déficit de la 1,25(OH) 2D3 que conduce a una disminución de la absorción
intestinal de calcio, hipocalcemia e hipofosfatemia y por lo tanto a un hiperparatiroidismo
secundario. Se observan los típicos signos clínicos y radiológicos y generalmente responde
a dosis altas de calcitriol (es una prueba diagnóstica y terapéutica).

El raquitismo calcitriol resistente o de tipo II, se hereda en forma autosómica recesiva y


constituye una auténtica forma de resistencia periférica a acciones de 1,25(OH) 2D3,
probablemente por una anormalidad del receptor. Existen signos clínicos y radiológicos de
raquitismo severo y se asocia a alopecia total y a síndrome hipotónico. Requiere megadosis
parenterales de calcio y de vitamina D2.D3.

Milliani Z, Valieri M. Actualización diagnóstico- terapéutica de raquitismo. Rev


Venez Endocrinol Metabol; 2 (2): 2-9.

52.-Masculino de 3 y medio años, con buen estado general y hemihipertrofia corporal, se le


descubre repentinamente una masa abdominal; el estudio urográfico i.v. muestra una masa
voluminosa sin calcificación en el riñón izquierdo que distorsiona el sistema pielocalicial, y la
ecografía abdominal determina el carácter sólido de la masa, así como la existencia de
trombosis en la vena renal. El diagnóstico mas probable es:

a) Neuroblastoma.
b) Nefroma mesoblástico congénito.
c) Carcinoma renal.
d) Tumor de Wilms.

TUMOR DE WILMS
Etiología

• Varias regiones cromosómicas se han asociado al Tumor de Wilms


– Banda 11p13 (contiene el gen supresor del tumor de Wilms WT1)
– Banda 11p15(sitio donde se coloca el gen WT2 del tumor de Wilms)
– Brazo cromosómico 17q (contiene el locus familiar FWT1)
– Brazo cromosómico 19q (contiene el locus familiar FWT2)
• p53
– Gen supresor de tumor
– Se asocia a anaplasia, estadios avanzados de la enfermedad
– Recurrencia

• Otros
– Bcl-2, TrkB

Cuadro clínico

• Masa abdominal
– Localizada en el flanco
– No se mueve con respiración

• Distensión abdominal
• Dolor abdominal
• Hematuria macroscópica
• Fiebre
• Anemia (asociada a hemorragia subcapsular repentina)

• Hipertensión (por aumento de la actividad de la renina): 25%

Bibliografía:

1. Grundy P, Breslow N, Green DM, et al.: Prognostic factors for children with
recurrent Wilms' tumor: results from the second and third National Wilms' Tumor
Study. Journal of Clinical Oncology 7(5): 638-647, 1989.
2. Shamberger RC, Guthrie KA, Ritchey ML, et al.: Surgery-related factors and local
recurrence of Wilms tumor in National Wilms Study 4. Annals of Surgery 229(2):
292-297, 1999.
3. Abu-Ghosh A, Goldman S, Krailo M, et al.: Excellent response rate (91%) to
ifosfamide, carboplatin, and etoposide (ICE) in children with advanced and/or
relapsed Wilms' tumor. Proceedings of the American Society of Clinical Oncology
A2157, 559a, 1999.
4. Kung FH, Bernstein ML, Camitta BM, et al.: Ifosfamide/carboplatin/etoposide
(ICE) in the treatment of advanced, recurrent Wilms tumor. Proceedings of the
American Society of Clinical Oncology A2156, 559a, 1999.
5. Garaventa A, Hartmann O, Bernard JL, et al.: Autologous bone marrow
transplantation for pediatric Wilms' tumor: the experience of the European Bone
Marrow Transplantation Solid Tumor Registry. Medical and Pediatric Oncology
22(1): 11-14, 1994.
6. Pein F, Michon J, Valteau-Couanet D, et al.: High-dose melphalan, etoposide, and
carboplatin followed by autologous stem-cell rescue in pediatric high-risk
recurrent Wilms' Tumor: a French Society of Pediatric Oncology study. Journal of
Clinical Oncology 16(10): 3295-3301, 1998.
7. Tannous R, Giller R, Holmes E, et al.: Intensive therapy for high risk (HR) relapsed
Wilms' tumor (WT): a CCG-4921/POG-9445 study report. Proceedings of the
American Society of Clinical Oncology 19: A2315, 2000.

53.- Una niña de 12 años es sometida a una prueba cutánea de Mantoux como parte de una
exploración física de control. No ha sufrido exposiciones conocidas a tuberculosis ni tiene
factores de riesgo. A las 48 horas aparece una zona de eritema de 20mm y un área de
induración de 7mm ¿Cómo pueden interpretarse estos hallazgos?

a) Es un resultado positivo de exposición a tuberculosis.


b) Es un resultado negativo de exposición a tuberculosis.
c) Conviene hacer una radiografía de tórax.
d) Deben obtenerse lavados gástricos de Mycobacterium tuberculosis.
Para determinar si el hallazgo de una prueba cutánea de Mantoux es positiva o no, se usa la
zona de induración, y no la de eritema. En ausencia de factores de riesgo conocidos o de
exposición una zona de induración menor a 10 mm, es un resultado de Mantoux negativo.
No están indicados ni radiografía de tórax ni lavado gástrico.

Bibliografía:
1. Grupo de trabajo de Tuberculosis de la Sociedad Española de Infectología Pediátrica.
Interpretación de la prueba de tuberculina.An Pediatr (Barc) 2003;59(6):582-5.
2. López-Hermosa, P. Papel actual del Mantoux en la población Infantil. Rev.Ped. Atención
Primaria.Vol I. Nº5, Jul/Sep 1999.
3. Alcaide Megías J et al. Epidemiologia de la tuberculosis. An Esp Pediatr 2000;53: 449-
457.
4. Alonso Moreno FJ et al. Prevalencia de la infección tuberculosa en las personas
inmigrantes del Área de salud de Toledo. Rev Esp Salud pública 2004; 78: 593-600.
5. Rodríguez Valvín E. Situación actual de la tuberculosis en España: incidencia y
mortalidad desde 1995.Características de los casos de tuberculosis y meningitis
tuberculosa declarados en 2000. SEMERGEN 2002; 28(7):395-400.
6. Rodrigo T, Caylà JA. Efectividad de los programas de control de la tuberculosis en
España. Med Clin (Barc) 2003;121(10):375-7.
7. De Paula F et al. Prevalencia de la infección tuberculosa entre los inmigrantes
magrebies. Med Clin (Barc) 2000;114(7):245-9.
8. Fernández Sanfrancisco MT et al. Prevalencia de la infección tuberculosa en la
población de inmigrantes de Ceuta, España. Rev Esp Salud Pública 2001; 75:551-8.

54.- Madre con embarazo normoevolutivo, RN termino con dificultad respiratoria severa
inmediatamente después de su nacimiento, ruidos respiratorios disminuidos, abdomen
plano. Rx de tórax con masa multiquística izquierda que desplaza mediastino a la derecha.
El diagnóstico más probable es:

a) Hernia diafragmática.
b) Fístula traqueo-esofágica.
c) Neumonía congénita complicada.
d) Atelectasia izquierda.

La hernia diafragmática congénita se produce por la falta del cierre de la membrana


pleuroperitoneal en la 8ª semana de gestación con defecto de la porción posterolateral del
diafragme (Hernia de Bochdalek) 80% es izquierda. Puede asociarse a otras anomalías
(atresia esofágica, anomalías del tubo neural, cardiopatías, hipoplasia pulmonar,
malrotación y alteraciones cromosómicas como trisomía 18, 21, 13.
Tiene una mortalidad que va del 25 al 80% y una frecuencia de 1/2000 –5000 recién
nacidos, más frecuente en varones que mujeres.

El cuadro clínico puede presentarse como:


• Insuficiencia Respiratoria Severa desde el nacimiento, con polipnea, quejido, aleteo nasal
y retracción costal.
• Disminución o ausencia de murmullo vesicular.
• Ruidos hidroaéreos (RHA) en tórax.
• Desplazamiento de ruidos cardiacos al lado contralateral.
Diagnóstico Prenatal: Ecografía rutina.
Diagnóstico Postnatal: Radiografía (Rx) tórax AP y cuadro clínico (Imágenes radiográficas
donde se aprecia consolidación en hemitorax izquierdo e imágenes hidroaéreas con
desplazamiento mediastino al lado contralateral)
Diagnóstico Diferencial: Quistes congénitos de pulmón – neumotórax.

Referencias:
1. Hernia Diafragmática Congénita 2000 – 2006 Dra. Nálit Almuna Fernández.
2. Hernia Diafragmática Congénita – Avances en tratamiento Prenatal Clínicas de
Perinatología, ISSN 0186-0208, Nº. 3, 2003 (Ejemplar dedicado a: Cirugía maternofetal),
ISBN 8448603508, pags. 459-473.
3. Hernia Diafragmática. Manual de Diagnóstico y Terapéutica en Pediatría Hospital
Infantil La paz Universidad Autónoma de Madrid Pag 320 ,2009.
4. Hernia Diafragmática Urgencias y tratamiento del Niño Grave J Casado Ana Serrano pag
627 2000.

55.- Paciente masculino de 3 meses de edad, residente de campo agrícola; con antecedente
de anemia, el cual acude porque la mamá le ha administrado gotas para los cólicos y
actualmente lo encuentra irritable, caliente, no quiere comer, no fija la mirada, como que no
ha orinado bien, presenta taquicardia, mucosas secas. La primera sospecha de intoxicación
que usted tendría sería por:
a) Hierro.
b) Ácido acetil salicílico.
c) Órgano fosforado.
d) Anticolinérgicos.

En el mundo existen alrededor de 13 millones de químicos naturales y sintéticos, y menos


de 3000 causan el 95% de las intoxicaciones. Un veneno (tóxico) es una substancia capaz de
producir efectos adversos en un organismo viviente. Existen distintos tipos, aquellos de uso
humano (comidas y sus aditivos, medicamentos y cosméticos) y aquellos que no lo son
(productos de limpieza, industriales, químicos, plantas y hongos no comestibles). Una
sobredosis implica exposición a cantidades excesivas de los primeros y a cualquier cantidad
de los últimos.

DIAGNÓSTICO

Para diagnosticar una intoxicación es vital tener una historia clínica y examen físico lo más
detallado posible dentro del contexto de urgencia de cada paciente.

Historia

1. Si es posible, nombre y cantidad de cada substancia.


2. Tiempo, ruta, duración y circunstancias de la exposición.
3. Tiempo de inicio, naturaleza y severidad de los síntomas
4. Medidas de ayuda administradas.
5. Historia médica y psiquiátrica, incluyendo medicamentos que el paciente
habitualmente.

Examen físico

1. Signos vitales, signos de estimulación o depresión, Glasgow modificado para


niños.
2. Examen físico, buscando lugar de entrada del tóxico (Ej: punciones venosas,
quemaduras por ácidos o cáusticos) o signos de intoxicación crónica (Ej:
líneas de Mees en el lecho ungueal, en la intoxicación por arsénico).
3. La presencia de ciertos síntomas y signos pueden clasificarse en síndromes
tóxicos, dentro de los cuales los principales están:
o Anticolinérgico: Midriasis, fiebre, íleo, taquicardia, rubor, mucosas
secas, visión borrosa, retención urinaria, mioclonus, psicosis tóxica,
agitación, convulsiones y coma.
Causado por: Atropina, antihistamínicos, fenotiazinas,
antidepresivos tricíclicos, floripondio (planta alucinógena).
Acercamiento terapéutico: Fisostigmina sólo en casos graves con
riesgo vital.
o Colinérgico: Miosis, salivación, epífora, defecación, emesis,
bradicardia, broncoconstricción.
Causado por: Insecticidas organofosforados y carbamatos,
pilocarpina.
Acercamiento terapéutico: Atropina, pralidoxima en intoxicación
por organofosforados.
o Extrapiramidal: Coreoatetosis, hiperreflexia, trismus, opistótonos,
rigidez y temblor.
Causado por: Haloperidol, fenotiazinas.
Acercamiento terapéutico: Difenhidramina y benztropina
o Alucinógeno: Alucinaciones, despersonalización, desrealización.
Causado por: Anfetaminas, canabinoides, cocaína, fenciclidina,
alcaloides indol.
Acercamiento terapéutico: Benzodiazepinas.

Bibliografía:

1. Beers, Mark; Berkow, Robert. The Merck Manual, 17th Ed., 1999. Section 23:
Poisoning.
2. Isselbacher, Kurt. Harrison´s Principles of Internal Medicine, 13th Ed., 1994.
Chapter 395: Acute poison and drug overdosage.
3. Goldfrank, Lewis et al.; Goldfrank´s Toxicologic Emergencies, 6th Ed., 1998.
Chapter 9: Identifying the Nontoxic Exposure.

Leikin, Jerrold; Paloucek, Frank. Poisoning & Toxicology Compendium 1st Ed., 1998.
Approach to Toxicology.

56.-Masculino de 44 años diagnosticado por trastorno obsesivo-compulsivo. Los actos


obsesivos más frecuentes en este trastorno son:

a) Recuentos mentales.
b) Evitar pisar las cruces de las baldosas.
c) Comprobaciones y rituales de limpieza.
d) Acumulación y colección de objetos.

Síntomas del TOC

Obsesiones

Las obsesiones son ideas o impulsos no deseados que aparecen repetidamente en la mente
de la persona que padece TOC. Los pacientes suelen tener miedo a sufrir daño ellos mismos,
o alguien al que quieren, se preocupan irracionalmente por no contaminarse, o tienen una
necesidad excesiva de hacer las cosas correctamente o con perfección. Una y otra vez, la
persona piensa algo que le inquieta, como por ejemplo, "Mis manos pueden estar
contaminadas--debo lavarlas" o "Puedo haber dejado el gas abierto" o "Estoy haciéndole
daño a mi hijo." Estos pensamientos angustiosos se inmiscuyen con los otros pensamientos
del paciente, y causan ansiedad. A veces, las obsesiones son de carácter violento o sexual, o
tienen que ver con enfermedades.

Compulsiones:

En respuesta a sus obsesiones, la mayoría de las personas con TOC recurren a


comportamientos repetitivos llamados compulsiones. Los más frecuentes son los rituales de
limpieza y las comprobaciones. Otros comportamientos compulsivos incluyen recuentos (a
menudo al mismo tiempo que se realizan otras acciones compulsivas, tales como lavarse las
manos), hacer repeticiones, acaparamiento, y recolocaciones sin fin de objetos en un
esfuerzo para mantenerlos perfectamente alineados. También son corrientes los problemas
mentales, tales como repetir frases mentalmente y hacer listas. Estos comportamientos, en
general, tienen por objeto proteger de peligros a la persona que padece TOC, o a los otros.

Algunas personas con TOC tienen rituales establecidos; otros tienen rituales que son
complejos y cambiantes.

El ejecutar estos rituales sólo proporciona alivio temporal de la ansiedad, pero no hacerlos
incrementa la ansiedad de la persona.

57.- La secreción excesiva de moco traqueobronquial suficiente para producir tos y


expectoración durante 3 meses como mínimo en 2 años consecutivos, se traduce como:

a) Bronquitis crónica.
b) Enfisema.
c) Asma.
d) Asbestosis.

Bronquitis crónica en toda persona que tenga tos y expectoración persistente durante tres
meses al año durante dos años consecutivos como mínimo. En la bronquitis crónica simple,
los pacientes no tienen signos de obstrucción al paso del aire. Algunos de estos pacientes
pueden tener unas vías respiratorias hiperreactivas, que producen broncoespasmo y
sibilancias de forma intermitente. Es lo que se llama bronquitis asmática crónica.

BIBLIOGRAFÍA
1. Guías para el diagnóstico y el tratamiento de la Enfermedad Pulmonar Obstructiva
crónica
Edición Especial 2003 INER
2. Pawles RA. Global Strategy for the diagnosis, management and prevention of chronic
obstructive pulmonary disease. Initiative for Chronic Obstructive Lung Disease (GOLD).
Am
J Respir Crit Care Med 163. 2001;1256-1276
3. Fabri LM. Global strategy for the diagnosis, management and prevention of COPD;2003
Update. Eur Respir S. 2003;22:
4. Sin, Don D. Contemporary Management of Chronic Obstructive Pulmonary Disease:
Scientific Review. JAMA . 2003;290:2301-2312
5. Man S. Contemporary Management of Chronic Obstructive Pulmonary Disease: Clinical
Applications. JAMA.2003;290:2313-2316.

6. Sutherland ER. Current Concepts Management of Chronic Obstructive Pulmonary


Disease.
N Engl J Med.2004;350:2689-2697
7. Hurst JR. Chronic obstructive pulmonary disease: the clinical management of an acute
exacerbation. Postgrad Med J;80:497-505

58.- Masculino de 17 años, que presenta hemartrosis. Tiene TPT alargado, TP y TT


normales, por lo que tiene una alteración de la vía:

a) Del Complemento.

b) Intrínseca.

c) Extrínseca.

d) Colinérgica.

Manual CTO 7° edición, p. 723.

La vía intrínseca está constituida por la activación secuencial de los factores XII, XI, IX,.
VIII y V. el tiempo de tromboplastina parcial activada mide la actividad de la coagulación
intrínseca y sirve para monitorizar el tratamiento con la heparina no fraccionada.

59. - The Virchow triad of thrombus in the etiology of pulmonary embolism is constituted
by:

a) Venous stasis, hypercoagulability, and endothelial damage.


b) Venous stasis, endothelial injury and hypoxemia.
c) Hypoxemia, hypercoagulability and cough.
d) Dry Cough.
Tromboembolismo pulmonar

Etiología:

El 90% de los casos de tromboembolismo pulmonar tienen su origen en venas de las


extremidades. Es favorecida por la triada de Virchow: éstasis venosa, daño de la íntima,
aumento de la coagulabilidad (puerperio, cirugía mayor, cáncer, uso de anticonceptivos,
policitemia vera, síndrome de hipercoagulabilidad (deficiencia de PC, PS, ATIII, resistencia
a la proteína C).

Fisiopatología.

Efectos respiratorios: aumento del espacio muerto alveolar, broncoconstricción,


taquipnea e hipoxemia, alteración ventilación/perfusión (V/Q) por redistribución de flujo.

Efectos hemodinámicos: La reducción mecánica leve o moderada no produce


aumento de la resistencia en forma significativa. Sobre el 50% de los casos presentan
incremento brusco de la resistencia y presión. Al efecto del émbolo se suman las aminas
liberadas por las plaquetas. En el 60-70% se desencadena cor pulmonale agudo con
disminución brusca del gasto cardiaco.

Manifestaciones clínicas del tromboembolismo pulmonar:

Disnea, Taquipnea, Dolor pleurítico, Crepitantes, Tos, Taquicardia, Hemoptisis,


broncoespasmo, cianosis, síncope, palpitaciones. Síndromes: disnea aguda de causa
desconocida, hemoptisis y/o dolor pleurítico, shock cardiogénico.

Bibliografía:

1.-British Thoracic Society, Standards of Care Committee. Suspected acute pulmonary


embolism: A practical approach. Thorax 1.997; 52 (suppl 4): S1-S23.

2.-The PIOPED Investigators. Value of the ventilation/perfusion scan in acute


pulmonary embolism: results of the Prospective Investigation of Pulmonary
Embolism (PIOPED). JAMA 1.990; 263: 2.753-9.

3.-Goodman PC. Spiral CT for pulmonary embolism Sem Resp Crit Care Med 2.000;
21(6): 503-10.
60.- Un cuadro de diarrea con una duración de más de dos semanas, pero que generalmente
no se extiende por más allá de cuatro se define como:

a) Diarrea aguda.
b) Diarrea persistente.
c) Diarrea crónica.
d) Diarrea acuosa.

La clasificación de la diarrea de acuerdo al tiempo de duración de este síntoma la divide en:


aguda, con duración de menos de 14 días, crónica con duración de más de 4 semanas, y
persistente ha la que tiene una duración menor del mes, y mayor a las dos semanas.

Kasper DL, Braunwald E, Fauci AS, Hauser SL, Longo DL, Jameson JL. Harrison´s
Principles of Internal Medicine. McGraw Hill. 16 Ed. 225 p.

61.- Infant, 33 weeks of gestation presenting in the first day of life, tachypnea, xiphoid
retraction, intercostal retractions, nasal flaring, deep moan and cyanosis. On physical
examination, crackles are audible bibasilar. The chest radiograph shows reticulonodular
infiltrates with air bronchogram. What is the most likely diagnosis?

a) Pneumonia.
b) hyaline membrane disease.
c) Transient tachypnea of the newborn.
d) meconium aspiration syndrome.

La enfermedad de membrana hialina se debe a un déficit de surfactante, afecta a recién


nacidos prematuros, su frecuencia aumenta en hijos de madres diabéticas y en embarazos
múltiples. El cuadro clínico se caracteriza por datos de dificultad respiratoria de inicio
precoz como: taquipnea, quejido intenso, aleteo nasal, retracciones xifoideas e inter y
subcostales y cianosis parcialmente refractaria al oxígeno. Los síntomas progresan hasta
alcanzar un máximo al tercer día. En la auscultación aparecen crepitantes en ambas bases.
En la radiografía de tórax se presenta un infiltrado reticulonodular con broncograma aéreo
en unos pulmones poco ventilados, se pueden presentar atelectasias. En la gasometría se
observa hipoxemia, hipercapnia y acidosis respiratoria. El tratamiento consiste en soporte
respiratorio, administración endotraqueal de surfactante y antibióticos. Por otro lado, la
taquipnea transitoria del recién nacido se presenta en recién nacidos de término, que nacen
por cesárea o parto vaginal rápido, debido a un retraso en la absoprción del líquido;
manifestándose con un distrés respiratorio de inicio precoz, la auscultación es normal.

Manual CTO pediatría, 7° edición p. 1302.


Guía clínica SDR neonatalMarzo 2006

62.- Lactante masculino de 8 meses de edad, con 8 Kg. de peso que presenta cólico intenso,
rechazo de alimentos, vómitos y evacuaciones mucosanguinolentas, la primera posibilidad
diagnóstica es:

a) Apendicitis aguda.
b) Divertículo de Meckel.
c) Invaginación intestinal.
d) Malrotación intestinal.

Invaginación intestinal

Obstrucción intestinal aguda provocada por la introducción del tubo digestivo dentro de sí
mismo.

Clínica: lactante sano y bien nutrido, que de forma súbita, cada 10-15 minutos, durante unos
minutos, presenta crisis de llanto, acompañadas de síntomas vagales (palidez, sudoración,
decaimiento), irritabilidad y rechazo del alimento. Inicialmente permanece asintomático
entre los episodios de llanto. De forma progresiva aparecen vómitos, anorexia, decaimiento,
sangre roja en heces, e incluso colapso vascular y shock.

Exploración física:

• Fosa iliaca derecha vacía. Signo de Dance (13%).

• Palpacion de masa en hipondrio derecho y colon transverso “signo de morcilla” (24 a


90%)

• Tacto rectal, “hocico de tenca” (0.5 al 3%)

Puede presentar sangre en el tacto rectal, aumento de ruidos hidroaéreos y


excepcionalmente prolapso rectal de la invaginación.
Rx. Simple:

• Patrón anormal de aire


• Opacidad en CID 25-60%
• Datos de oclusión

• Distensión de asas

• Niveles hidroaéreos

• Ausencia de aire en recto

• Tratamiento:

• Ayuno.

• Terapia hídrica.

• Esquema Antimicrobiano. Ampicilina-Amikacina.

• Sonda a derivación

• Catéter Venoso.

• Sonda Urinaria.

Bibliografía:

1.- O’NEILL PEDIATRIC SURGERY 1998 URG. PED. HIM 5 ED 2002


OLDHAM SURGERY OF INFANTS AND CHILDREN. 1997
63.- La conducta más adecuada a seguir en un paciente masculino de 14 años con
diagnóstico de orquiepididimitis es:

a) Administración de antibióticos y de antiinflamatorios.


b) Exploración quirúrgica de inmediato.
c) Vigilancia y exploración quirúrgica en 24 hrs.
d) Administración de AINES.

El tratamiento consistirá de una orquiepididimitis consiste en:


— Medidas generales: reposo en cama durante al menos 5 días o hasta que ceden los
síntomas y utilización de un suspensorio.
— Antiinflamatorios y analgésicos a dosis habituales.
— Antibióticos:
Empíricamente si no conocemos germen causal, se asocian:
• Netilmicina 150 mg/12 h/vía IM/5 días.
• Ciprofloxacino 500 mg/12 h/oral/10 días.
Si conocemos germen:
— Enterobacterias y gonococos:
Ceftriaxona F 1 gr/24 h/I.M.
Amoxicilina-clavulánico 500 mg/8 h/oral.
Quinolonas a dosis habituales.
— Cocos gram positivos:
Amoxicilina-clavulánico 500 mgr/8 h.
Ciprofloxacino 500 mgr/12 h.
— Pseudomonas:
Aztreonam 2 gr/24 h.
Ciprofloxacino 500 mgr/12 h/oral.
— Chlamydia Teacomatis:
Doxicilina 200 mgr/12 h/oral.
Referencias:

1. Young Y., Miller R Incidence of malignant tumours in U.S. children. J Pediatr 1975; 86:
254-258.
2. Reiter A., Schrappe M. Chemotherapy in 998 unselected childhood acute lymphoblastic
leukemia patients. Results and conclusions of the multicenter trial ALL-BFM 86. Blood
1994; 84: 3122-3133.
3. Schorin M., Blattner S., Gelber R., et al.
Treatment of childhood acute lymphoblastic leukemia: Results of Dana Farber Cancer
Institute/Children s Hospital acute lymphoblastic leukemia consortium protocol 85-01. J
Clin , Oncol 1994; 12: 740-747.

64.- Un niño de 4 años de edad previamente sano aunque no vacunado presenta inicio súbito
de fiebre alta estridor inspiratorio, rechazo para beber agua y se aprecia posición tripoide.
De las siguientes acausas de estridor inspiratorio ¿cual se ajusta mejor a este cuadro
clínico?

a) Epiglotitis.
b) Anillo vascular.
c) Aspiración de cuerpo extraño.
d) Tumor laríngeo.

La epiglotitis aguda (EA) es una inflamación de la epiglotis y las estructuras adyacentes de


instauración brusca y rápidamente progresiva, que se produce sobre todo en niños
pequeños.
Su consecuencia más importante es la capacidad de provocar una obstrucción severa e
incluso total de la vía aérea superior, pudiendo causar la muerte.
La introducción de la vacuna contra el Haemophilus influenzae tipo B (Hib), el principal
patógeno implicado en la EA, ha reducido notablemente su incidencia, especialmente en los
niños menores de 5 años.

Clínica
El caso típico de EA es el de un varón de 2 a 4 años de edad que presenta en cualquier
momento del año una historia de 6 a 12 horas de fiebre elevada y disfagia. La odinofagia,
más frecuente en niños mayores y adultos, se observa en el 85% de éstos3. De forma
relativamente rápida, se instaura dificultad respiratoria, que hace que el paciente adopte
una postura hacia adelante, con el cuello en hiperextensión y la boca abierta con la lengua
fuera, presentando una postura clásicamente llamada "en trípode". Se muestra ansioso e
inquieto. Es típico el babeo, aunque no constante, pudiendo faltar en la mitad de los casos.
Al contrario que en las laringitis agudas, el estridor inspiratorio no es tan ruidoso, sino de
tono bajo y húmedo, y es rara la tos.

BIBLIOGRAFIA:

1. AGUDELO, Bertha. Tratamiento Del Croup Basado En La Evidencia. Revista


Colombiana De Neumología. Volumen 12 Nº 4, Noviembre De 2000.
2. REYES, Aristizábal, Leal. Neumologia Pediatica: Infeccion, Alergia Y Enfermedad
Respiratoria Del Niño. 4° Edición, Bogotá: Editorial Medica Panamericana, 2001.

65.- Adolescente de 12 años es atendido en consulta por dolor mesogástrico y diarrea de


dos semanas de evolución, cuatro evacuaciones al día alternadas con estreñimiento, las
evacuaciones son semilíquidas, fétidas y de color verdoso grasoso que flotan en el sanitario,
sin sangre. E.F. Abdomen distendido, timpánico y con peristalsis aumentada.
El agente causal más probable en este caso es:

a) Entamoeba hystolítica.
b) Giardia lamblia.
c) Trichuris trichiura.
d) Salmonella tiphy.

La infección en el hombre tiene una evolución clínica variable, que va desde la infección
asintomática, la mayoría de las veces, hasta la diarrea severa. Esto parece estar
relacionado tanto con factores del hospedero como del agente biológico. El período de
incubación después de la ingestión de quistes es variable y puede ser tan corto como una o
dos semanas. (Hill, 1993; Farthing, 1996; Ortega y Adam, 1997; Procop, 2001).
Los signos y síntomas de la enfermedad son difíciles de distinguir de los de otras
enfermedades gastrointestinales. La infección no siempre produce diarrea. De hecho, otros
síntomas digestivos como los dolores abdominales y los cólicos pueden ocurrir más
frecuentemente que la diarrea (Beaver et al., 1984; Adam, 1991, Aparicio Tijeras et al.,
2004).
Giardia es reconocida como una causa de rápida pérdida de peso y malabsorción de grasas,
y puede presentarse lo mismo en forma crónica, que en forma aguda. Se ha descrito incluso
que la afección de las funciones pancreáticas pudiera ser una causa de severa malabsorción
en giardiasis (Carroccio et al., 1997). Diferentes estudios han encontrado los niveles de
algunos oligoelementos como el hierro sérico, el zinc (Ertan et al., 2002; Demirci et al.,
2003) y el magnesio (Olivares et al., 2003; Kilic et al., 2004) significativamente más bajos
en pacientes infectados con Giardia que en controles, lo que pudiera deberse a la
mala absorción que se produce en la giardiasis (Demirci et al., 2003; Olivares et al., 2003).

La sintomatología gastrointestinal es la más frecuente y comprende un amplio espectro de


manifestaciones clínicas (tabla 1): a) enteritis aguda (autolimitada), b) diarrea crónica, y c)
malabsorción con esteatorrea y pérdida de peso Las manifestaciones extraintestinales que
con más frecuencia se han asociado a la giardiosis son erupción maculopapular, urticaria,
aftas, poliartiritis, colangitis, asma bronquial, iridociclitis, retinitis, etc. En las formas de
giardiosis crónica los síntomas predominantes son el malestar abdominal acompañado de
dolor epigástrico difuso. La diarrea puede persistir o alternar con estreñimiento y puede
acompañarse de pérdida de peso.

Tabla 1. Manifestaciones clínicas de la giardiosis.

Síntomas %

Diarrea 63
Déficit de absorción de lactosa 60
Estreñimiento 55
Déficit de absorción de B12 / fólico 45
Flatulencia 46
Esteatorrea 44
Dolor / distensión abdominal 32
Fatiga 28
Anorexia / náuseas 20
Pérdida de peso 18
Vómitos 5
Moco en heces 4
Fiebre 3

Se ha reportado la asociación con enfermedad de Whipple lo que pudiera ser posible en


estos casos por un defecto común del sistema inmune, o que la infección con un tipo de
microorganismo pueda predisponer a la infección con el otro (Fenollar et al., 2003).
También se ha señalado, pero con baja frecuencia, la enfermedad biliar (Hill, 1993;
Farthing, 1996).
Este parasitismo se ha visto muy relacionado con trastornos del crecimiento y desarrollo en
los niños; algunos estudios han mostrado que la infección por G. lamblia tiene un impacto
adverso sobre el crecimiento de los niños (Farthing et al., 1986; Valencia et al., 1995;
Muniz-Junqueira y Queiróz, 2002; Sackey et al., 2003) mientras que otro no mostraron
relación entre la infección por G. lamblia y trastornos nutricionales (Adam, 1991; Lunn et
al., 1999; Moya-Camarena, 2002).

66.- Masculino de 3 años, preescolar. Es atendido en consulta. desde los 2 años de edad, al
llorar presenta cianosis labial y peribucal, cuando se golpea o al regañarlo, no pierde el
conocimiento, la cianosis desaparece al ceder el llanto, le ocurre casi a diario, no hay
antecedente familiar de enfermedad neurológica o cardiovascular. Examen físico peso 13.6
kg., talla 93 cm., resto sin datos patológicos. En este paciente el diagnóstico más probable
es:

a) Epilepsia.
b) Espasmo del sollozo.
c) Tetralogia de fallot.
d) Enfermedad por reflujo gastroesofágico.

Espasmo del sollozo


Es la retención de la respiración posterior a un evento que disguste tal como una caída, el
estar frustrado o enojado, o por estar asustado. -El niño da uno o dos gritos largos y
posteriormente retiene su respiración en expiración hasta que sus labios se ponen azules.
-El niño posteriormente se desmaya (algunos llegan a tener algunos espasmos musculares). -
después el niño respira normalmente y permanece completamente alerta en menos de 1
minuto. -El inicio ocurre entre 6 meses y 2 años. Esto sucede solamente cuando el niño está
despierto.

Se define al espasmo del sollozo (Breath-holding spells), como un evento caracterizado por
crisis recurrentes de apnea transitoria, pérdida del conocimiento y cambios en el tono
muscular normal. De inicio súbito, no seconsidera secundario a patología orgánica ni
resultado de una manifestación psiquiátrica.

Entre el 5 y el 7% de la población infantil sana presenta crisis de espasmos de sollozos, sin


embargo, algunos autores coinciden en que esta entidad es mucho más frecuente. Por lo
general dichos eventos inician entre los 6 a 12 meses de edad con un pico de incidencia
entre los 12 y los 24 meses de edad.

Es poco frecuente que se presente en niños mayores de 6 años de edad por lo que su
aparición en niños mayores de 4 años amerita especial atención. El 25% de los niños que los
presentan tiene un familiar directo que lo padeció en la infancia. Se presenta más
frecuentemente en varones.

Son causas frecuentes de síncopes y convulsiones anóxicas secundarias a isquemia o hipoxia,


con la consecuente depresión súbita de la función neuronal, en contraste con las
convulsiones epilépticas, secundaria a una descarga excesiva de las neuronas.

El espasmo del sollozo a sido dividido en el espasmo del sollozo pálido y cianótico,
dependiendo de la coloración de la piel durante el evento. La fisiopatología, en cada caso en
particular, es diferente. Entender las características del espasmo del sollozo y poder
diferenciarlo de otras entidades, puede ayudar al pediatra a tranquilizar a los padres.

Bibliografía:

1- Berman, RE., Kliegman, RM., Jenson, HB: Nelson Textbook of Pediatrics. 16th edition,
Philadelphia, Pennsylvania, W.B. Saunders Company U.S.A.., 2,000; 1829.
2- Anderson JE, Bluestone D: espasmos del sollozo. Contemp Pediatr 2,000;17(1):61-72.
3- Macan H, et al: Espasmo del sollozo en 91 niños y respuesta al tratamiento con hierro.
Arch Dis Child 1999;81:261-262.
4- Breningstall GN: Breath-holding Spells . Pediatr Neurol 1996;14:91-97.
5- DiMario FJ: Breath-holding spell in childhood. Am J Dis Child 1992;146:125-131.
6- Lombroso CT, Lei-man P: Breath-holding spell (cyanotic and pallid in fantile syncope).
Pediatrics 1967;38:563-581.
7- Gauk EW,Kidd L, Prichard JS: Mechanism of seizures associated with breath-holding
spell. N Engl J Med 1963; 268: 1436-1441.

67.- Masculino de 3 años es llevado a consulta por hiporexia. Antecedentes: habita en un


rancho, geofagia positiva, dolor abdominal, cólico desde hace varios meses. Las
evacuaciones en los últimos 4 días son semilíquidas mucosas y con pujo. Exploración física:
mala higiene personal, desnutrido, abdomen globoso blando y dolor a la presión en colon
descendente, peristalsis aumentada. La complicación mas frecuente que se puede presentar
en este paciente es:

a) Prolapso rectal, trichurosis


b) Apendicitis.
c) Obstruccion intestinal.
d) Perforación intestinal.
Introducción.

La trichuriosis es una geohelmintiasis frecuente en zonas tropicales, rurales. Se estima que


se encuentran infectadas unos 100 millones de personas en Latinoamérica y Caribe (Hotez
PJ, et al., 2008). Predomina en niños en edad escolar, en quienes se asocia a colitis crónica
y síndrome disentérico, retardo en el crecimiento y disminución de peso; la deficiencia en
las funciones cognitivas y alteraciones conductuales se han relacionado con anemia
ferropriva, altas cargas parasitarias y desnutrición. Los casos de la parasitosis en adultos
que viven en zonas endémicas han aumentado, pero no se reportan usualmente. (Khuroo M,
et al. 2010).

La ascariosis y la trichuriosis son las infecciones por geohelmintos más frecuentes en


México.

NTDs en LAC: Prevalencia y distribución. Hotez PJ, et al, 2008.

LAC: Latin American and Caribbean Region

NTDs. Neglected Tropical Diseases.

Las lesiones intestinales y el cuadro clinico varían en relación directa al número de


parásitos y factores dependientes del hospedero (edad, estado nutricional, infecciones
concomitantes).
En infecciones leves y moderadas el daño, apenas apreciable, consiste en compresión
mecánica de las células de la mucosa colónica y se asocia a dolor abdominal de tipo cólico y
episodios diarreicos.

En infecciones masivas la mucosa intestinal se encuentra edematosa y friable, con sangrado


fácil; es característica la degeneración y necrosis de las células cercanas a la cabeza del
parásito, con pequeñas hemorragias subepiteliales e inflamación con infiltración difusa de
linfocitos y eosinófilos.
Induce, al igual que los otros geohelmintos, una respuesta de tipo Th2 y respuesta
reguladora Th2/Treg (Jackson JA, et al. 2009).

Las manifestaciones clínicas varían de acuerdo a la masividad de la infección y la presencia


de otros parásitos (poliparasitismo) e incluyen dolor abdominal, cefalea, hiporexia, pérdida
de peso, diarrea crónica, disentería, pujo, tenesmo, prolapso rectal y signos y síntomas
relacionados con anemia hipocrómica microcítica; cada tricocéfalo expolia alrededor de
0.005 ml de sangre/día y restos tisulares. Además, la irritación constante de las
terminaciones nerviosas intramurales redunda en hiperperistaltismo.

Complicaciones.
- Poliparasitismo
- Prolapso rectal
- Anemia
- Apendicitis
- Infección bacteriana 2aria
- Retraso pondoestatural y déficit cognitivo en escolares.

TRICHURIOSIS

Dra. Teresa Uribarren Berrueta


Departamento de Microbiología y Parasitología, Facultad de Medicina, UNAM October,
2010.

Bibliografía:

- Mohammad S. Khuroo, Mehnaaz S. Khuroo, and Naira S. Khuroo. Trichuris dysentery


syndrome: a common cause of chronic iron deficiency anemia in adults in an endemic area
(with videos). Gastrointestinal Endoscopy, Jan 2010; 71(1):200-204.
doi:10.1016/j.gie.2009.08.002.
- Geary TG, Woo K, McCarthy JS, Mackenzie CD, Horton J, Prichard RK, de Silva NR, (...),
Bundy DA. Unresolved issues in anthelmintic pharmacology for helminthiases of humans.
Int J Parasitol 2010;40(1):1-13. doi:10.1016/j.ijpara.2009.11.001 Geohelmintos y otros
nematodos.
- Kyung-Sun Ok, et al. Trichuris trichiura Infection Diagnosed by Colonoscopy: Case
Reports and Review of Literature. Korean J Parasitol. Sept 2009;47(3):275-280
DOI: 10.3347/kjp.2009.47.3.275.

- Hu Y, Xiao S-H, Aroian RV. The new anthelmintic tribendimidine is an L-type (Levamisole
and Pyrantel) nicotinic acetylcholine receptor agonist. PLoS Neglected Tropical Diseases
2009;3(8), art. no. e499.

- Jackson JA, Friberg IM, Little S, Bradley JE. Review series on helminths, immune
modulation and the hygiene hypothesis: Immunity against helminths and immunological
phenomena in modern human populations: Coevolutionary legacies? Immunology 2009;126
(1):18-27. doi:10.1111/j.1365-2567.2008.03010.x.

68.- Recién nacido de 32 semanas de gestación, que presenta de manera súbita hipotensión,
anemia y abombamiento fontanelar. A al EF presenta una FC de 100 x’, una FR de 20 x’ y
comienza a convulsionar ¿Cuál es el diagnóstico más probable?

a) Hemorragia de la matriz germinal.


b) Leucomalacia periventricular.
c) Hidrocefalia.
d) Hipotiroidismo congénito

Manual CTO pediatría, 7° edición p.1295.

La matriz germinal es una estructura del cerebro inmaduro hasta la semana 34 de


gestación. Su lesión produce hemorragia. Se manifiesta con deterioro del estado general,
aparición de pausas de apnea, bradicardia, hipoventilación, convulsiones, disminución del
reflejo del Moro y fontanelas a tensión. Debe sospecharse ante un paciente pretérmino que
presenta de forma súbita anemia, hipotensión y abombamiento fontanelar. Puede derivar en
la aparición de leucomalacia periventricular.

69.- La indicación quirúrgica es obligada en un paciente afectado de enterocolitis


necrotizante cuando presente uno de los siguientes signos clínicos:
a) Heces mucosanguinolentas.
b) Vómitos biliosos.
c) Distensión abdominal.
d) Neumoperitone.

La enterocolitis necrotizante (ECN) es una lesión isquémico-necrótica intestinal que se


sigue habitualmente de sepsis bacteriana a partir del foco digestivo. Se produce en el 1 -
5% de los RN ingresados en UCI, siendo los RN pretérmino con enfermedades graves los
que tienen mayor riesgo.

Factores predecesores:
- Policitemia
- Inicio precoz de alimentación oral
- Alimentación oral con elevados volúmenes y concentraciones de alimento
- Agentes infecciosos: E. coli, C. prefrings, Rotavirus.

Clínica:-
- Distensión abdominal repentina y heces sanguinolentas.
- Evolución insidiosa a sepsis, shock, muerte.

Diagnóstico:
- Rx simple de abdomen con:
o Edema en asas
o Neumatosis intestinal con patrón en “miga de pan”
o Signos perforación: neumoperitoneo.
o Cuadro evolucionado: gas en vena porta.

El tratamiento de la ECN tiene dos vertientes:


- El tratamiento, en principio, debe ser médico, con dieta absoluta, fluidoterapia y
antibióticos (para anaerobios y gramnegativos incluyendo Pseudomonas).
- Se indica el tratamiento quirúrgico, en caso de:
o Perforación: asa fija durante 24h, neumoperitoneo en la Rx o líquido
parduzco en la paracentesis.
o Peritonitis: abdomen duro, brillante, con eritema en la pared abdominal y
líquido libre en la cavidad abdominal (ecográficamente) o palpación de una
masa.
o Sepsis refractaria al tratamiento.

Con respecto al resto de opciones que aparecen en la pregunta:


- El shock hipovolémico (respuesta 5) no forma parte del cuadro clínico habitual de la
ECN, ya que el shock de la ECN es de tipo séptico y su manejo debe ser, en
principio, médico.
- Las heces mucosanguinolentas (respuesta 1), los vómitos biliosos (respuesta 2) y la
distensión abdominal (respuesta 3), forman parte de la clínica típica de la ECN,
pero no incluye ninguna dentro de los criterios que indican la cirugía.

Nelson, Tratado de Pediatría 15ª Ed., págs. 616-617.

70.- ¿Cuáles son las características de una ictericia fisiológica?

a) Inicia entre el segundo y tercer día de vida, tiene una duración menor de 10
días y la concentración máxima es de 12 mg/dl.
b) Inicia el primer día de vida, tiene una duración menor de 10 días y la concentración
máxima es de 20 mg/dl.
c) Inicia entre el segundo y tercer día de vida, tiene una duración mayor de 10 días y
la concentración máxima es de 12 mg/dl.
d) Inicia a la semana de nacimiento y dura una semana más.

Manual CTO pediatría, 7° edición p. 1306.

La ictericia fisiológica del recién nacido inicia entre el segundo y tercer día de vida, tienen
una duración entre 5 y 7 días y alcanza una concentración máxima de 12 mgdl a los 2 ó 4
días. En cambio la ictericia no fisiológica inicia dentro de las primeras 24 horas de vida,
tiene una duración superior de 10 a 15 días, la bilirrubina total en RNT es mayor de 12
mg/dl o mayor de 14 mg/dl en RNPT, el incremento de bilirrubina es mayor de 5 mg/dl en
24 horas y la bilirrubina directa es mayor de 1 mg/dl.

71.- ¿El diagnóstico de hepatitis A en su fase aguda se realiza con?

a) Serología para detectar anticuerpos totales.


b) Serología para detectar anticuerpos IgG totales inespecíficos.
c) Serología para detectar anticuerpos IgG totales específicos.
d) Serología para detectar anticuerpos IgM totales específicos.
La IgM sérica aparece desde el comienzo de la enfermedad y desaparece en término de 4
meses, pero puede persistir 6 meses o más; la presencia de IgM en suero denota infección
actual o reciente.

Committee on Infectious Diseases 2003. Red Book 2003, 26th Ed.


American Academy of Pediatrics; páginas 374-383.

72.- ¿En un paciente recién nacido de 2 días, con vómitos gástricos, evacuaciones normales,
abdomen distendido, y rayos x de abdomen con asas muy distendidas, con niveles,
hidroaéreo el diagnóstico probable es?

a) Atresia de duodeno.
b) Hipertrofia pilórica.
c) Atresia de ileon.
d) Reflujo gastroesofágico.

De las malformaciones intestinales, la atresia de íleon es la más frecuente y la asociación


con otro tipo de malformaciones o cromosomopatías es elevada; en este caso no existió tal
correlación. Hacer un correcto diagnóstico prenatal mediante ultrasonidos nos permite
planificar el momento idóneo del parto, llevándolo a cabo en un centro con cirugía
pediátrica y así mejorar los resultados perinatales. Se discute cuándo finalizar la
gestación, y en principio no hay contraindicación en prolongarla hasta la madurez fetal
siempre que no exista ascitis fetal, que sería un signo de riesgo inminente de perforación
intestinal por íleo meconial; en tal caso finalizar el embarazo es la postura más prudente
por la elevada mortalidad de esta eventualidad; éste no fue nustro caso. Aunque autores
como Schwobel et al propugnan la evacuación mediante punción de la ascitis fetal en el caso
de presentarse con la finalidad de disminuir la presión abdominal fetal y alargar todo lo
posible la duración del embarazo. Nosotros nos vimos abocados a terminar la gestación por
motivos meramente obstétrica.

• Clínica e investigación en ginecología y obstetricia, ISSN 0210-573X, Vol. 32, Nº. 1,


2005 , pags. 36-37
73.- Se trata de paciente masculino, primogénito, que tiene vómitos progresivos, gástricos,
desde la segunda semana de vida; la posibilidad diagnóstica es?

a) Mala técnica en la alimentación.


b) Reflujo gastroesofágico.
c) Estenosis hipertrófica de píloro.
d) Bandas de Ladd.

La estenosis hipertrofia del píloro (EHP) es una emergencia médico-quirúrgica y


representa una de las causas mas frecuentes de cirugía abdominal en las primeras semanas
de vida1'2.La exacta causa de la hipertrofia del músculo pilórico es desconocida1'3'4. Se
caracteriza por obstrucción del vaciamiento gástrico, lo que clínicamente se expresa como
vómitos progresivos, frecuentemente de carácter explosivo que pueden llevar a
deshidratación grave con riesgo vital del paciente1' 3"5.

Bibliografía:

1.-Urgencias en Pediatría, Interamericana.McGraw – Hill. Capítulo: Urgencias Médico


Quirúrgicas, Sección XXIII, pág. 718-721.
2.-Operative Pediatric Surgery. Moritz M. Ziegler. International Edition, pág. 583- 588.
3.- Cirugía Pediátrica, Ashcraft - Holder Interamericana.McGraw – Hill pág. 297– 313

74.- Se trata de un recién nacido que cursa con lesión del plexo braquial que deja el brazo
en “posición de mesero”, en un parto distócico. ¿Esta lesión es provocada a nivel de?

a) Cervicales 5-6- +/- 7.


b) Cervicales -6-7 T1.
c) Cervicales 6-7-8.
d) Cervicales 7-8 T1.

La parálisis del plexo braquial de los neonatos se clasifica de acuerdo con la localización
anatómica y el tipo de lesión. Las más frecuentes son las lesiones de la porción superior del
plexo. A pesar de que se han descrito los factores de riesgo asociados con la lesión, la
incidencia global de este trastorno ha permanecido estable durante las últimas tres
décadas. La verdadera tasa de recuperación completa después de una parálisis del plexo
braquial es aún tema de discusión. El conocimiento de la clasificación de la lesión de un
lactante determinado puede ayudar a predecir su pronóstico a largo plazo. El examen físico
meticuloso es esencial para arribar al diagnóstico, evaluar las enfermedades asociadas,
determinar el plan terapéutico y establecer la necesidad de derivación. En general no están
indicados otros exámenes complementarios. El tratamiento inicial para el lactante con
parálisis del plexo braquial consiste en asesoramiento, fisioterapia o terapia ocupacional y
observación clínica del lactante. Un porcentaje de los niños afectados requiere una
intervención quirúrgica u otros abordajes terapéuticos, como por ejemplo inyecciones de
toxina botulínica.

La lesión que afecta el tronco superior (raíces C5 y C6) y en ocasiones, el tronco medio
(raíz C7). Los grupos musculares afectados por esta lesión son los rotadores externos y los
abductores del hombro, los flexores del codo, los supinadores del antebrazo y a menudo los
extensores de la muñeca. Esto genera el fenotipo clásico conocido como postura en “propina
de mesero”, con aducción y rotación interna del hombro, extensión del codo, pronación del
antebrazo y flexión de la muñeca. En esta lesión, además del compromiso del músculo
bíceps, cuando el músculo tríceps está débil o ausente, el codo puede estar parcialmente
flexionado.

Fenotipo clásico asociado con la lesión superior del plexo braquial.

Referencias bibliográficas:

1. Borrero JL. Obstetrical Brachial Plexus Paralysis. 2nd ed. Lake Mary, Fla: Design and
Print Progressive Communications; 2007.
2. Seddon HJ. Three types of nerve injury. Brain. 1943:238–288.
3. Rouse DJ, Owen J, Goldenberg RL, Cliver SP. The effectiveness and costs of elective
cesarean delivery for fetal macrosomia diagnosed by ultrasound. JAMA. 1996;276:1480–
1486.
4. Pollack RN, Buchman AS, Yaffe H, Divon MY. Obstetrical brachial palsy: pathogenesis,
risk factors, and prevention. Clin Obstet Gynecol. 2000;43:236–246.
5. al-Qattan MM, Clarke HM, Curtis CG. Klumpke’s birth palsy. Does it really exist? J
Hand Surg [Br]. 1995;20:19–23.

75.- Ante la sospecha de una torsión de testículo, ¿cuál es, entre las siguientes, es la
prueba diagnóstica de elección?
a) Ecografía-doppler.
b) Ecografía.
c) Radiografía escrotal.
d) Palpación bimanual.

Ecografía eco-doppler color: Es la prueba diagnóstica de elección al permitir la visualización


de la vascularización testicular pudiendo distinguir si el flujo circulatorio es normal,
ausente o aumentado. En la torsión lo normal es que esté reducido o ausente. Tiene una
especificidad del 100% y una sensibilidad del 80%, aunque tiene sus limitaciones en las
subtorsiones.
Gammagrafía isotópica testicular con TC 99: Es también un método sensible y específico,
pero del que no siempre se puede disponer en urgencias. Se verá un área de captación
disminuida o ausente, rodeado de un área de captación aumentada por la hiperemia
reactiva.

Bibliografía:
1) Sesions AE et al: Testicular torsión: direction, degree, duration and desinformation. J.
Urol
2003, 169: 663-665.
2) Cummings JM el al: Adult testicular torsión. J.Urol. 202, 167: 2109, 167: 2109-2110
3) Bedos F et al: Manual de Urología. Ed Masson SA. Barcelona 1996. 297-335.

76.- Los Factores de riesgo para presentar enterocolitis necrotizante son:

a) Anemia, hipoxia, bajo gasto e infección por Yersinia enterocolítica.


b) Policitemia, alto gasto e infección por Shigella.
c) Policitemia, hipoxia, bajo gasto e infección por Escherichia coli.
d) Prematurez, corioamnioitis, isoinmunización materno- fetal.
Manual CTO pediatría, 7° edición p.1306.

La enterocolitis necrotizante es una lesión isquémico – necrótica que afecta al íleon distal y
colon proximal. Las causas que predisponen a la isquemia son: policitemia, inicio temprano
de la alimentación con elevados volúmenes y concentraciones y situaciones de hipoxia y
bajo gasto. Se han involucrado a los siguientes agentes: E. coli, Clostridium prefringens,
Staphilococcus epidermidis y rotavirus. Se manifiesta con distensión abdominal y
evacuaciones sanguinolentas en la segunda semana de vida, que puede originar sepsis,
choque y muerte. En la radiografía de abdomen, el dato típico es la neumatosis intestinal,
también se presenta edema de asas, patrón en miga de pan, asa fija y gas en la vena porta.
El tratamiento consiste en antibióticos para Gram negativos y anaerobios, ayuno,
descompresión nasogástrica, aporte de líquidos y cirugía en caso necesario.

77.-Masculino diagnosticado con sífilis congénita tardía, una de las manifestaciones de


esta infección es:

a) Hepatomegalia.
b) Rinorrea.
c) Queratitis.
e) Pseudoparálisis de Parrot.

La sífilis congénita tardía se manifiesta después de los 2 años de vida, el síntoma más
frecuente es la queratitis, que es parte de la tríada de Hutchinson (queratitis, hipocausia y
dientes en tonel o Hutchinson), así como deformidades óseas (tibias en sable, frente
olímpica, engrosamiento clavicular), rodilla de clutton, paresia juvenil y tabes dorsal. La
hepatomegalia, rinorrea, roseola sifilítica y la pseudoparálisis de parrot son
manifestaciones de la sífilis precoz que se manifiesta en los primeros dos años de vida.

Bibliografía:

• Evans HE, Frenkel LD: Congenital Syphilis. Clin Perinatol 1994; 21(1)
Honeyman: Manual de Dermatología, 2da ed. 1988; pg. 46
. Lukehart SA, Holmes KK: Sífilis. en: Principios de Medicina Interna. 12 ed. Ed.
Harrison. Cap. 128, pg. 765.

78.- El signo radiográfico de Thurston Holland es característico de las lesiones fisiarias


tipo (Salter y Harris):
a) I
b) II
c) III
d) IV

Clasificación:

Aunque se han propuesto múltiples clasificaciones la publicada por Salter y Harris en 1963
ha sido la más difundida y es la que utilizaremos en resto del capítulo.
Clasificación de Salter y Harris.

Está en parte basada en la clasificación preconizada 65 años antes por J. Poland y, en ella,
se distinguen cinco grandes grupos (Fig. 1):

• Tipo I. Similar al grupo I de Poland y consiste en una separación completa epifiso-


metafisaria, normalmente como resultado de la aplicación de fuerzas de avulsión o
cizallamiento.
• Tipo II. Es el tipo más común de fractura fisaria. Es muy parecido al tipo I pero, en ésta,
el trazo de fractura, después de transcurrir a lo largo de la mayor parte de la fisis se
adentra en la metáfisis, quedando por tanto un pequeño fragmento de la misma,
generalmente triangular, unido a la fisis y epífisis (signo de Thurston Holland).
Generalmente estas fracturas se producen por la combinación de fuerzas de flexión lateral
(varo-valgo) y cizallamiento.

• Tipo III. En estas fracturas, parte del trayecto también es una separación fisaria pura
pero, en un punto determinado, se produce una inflexión y cruza la fisis y la epífisis hasta
llegar a la superficie articular. Están producidas por un mecanismo de cizallamiento
intraarticular.
• Tipo IV. En este tipo, la línea de fractura comienza en la superficie articular y atraviesa,
casi perpendicularmente, la epífisis, la fisis y la metáfisis. Suelen ser consecuencia de
fuerzas de avulsión o cizallamiento.
La gravedad las fracturas tipos III y IV estriba en dos puntos: el trayecto es
intraarticular y la comunicación epifiso-metafisaria que se produce hace posible la
formación de puentes óseos.

• Tipo V. Se trata de un traumatismo fisario por compresión cuya existencia es


controvertida. El diagnóstico es siempre retrospectivo por la apartición de cierres fisarios
prematuros .

Manual CTO 5ª edición, Traumatología y Ortopedia, Pág. 38

79.- Se trata de masculino de 24 años que acude por esterilidad; es azoospérmico, tiene
fenotipo eunucoide, ginecomastia, distribución feminoide del vello púbico, testículos
pequeños y cromatina sexual positiva en 8%. ¿El diagnóstico a considerar es?

a) Síndrome de Noonan
b) Síndrome de Turner
c) Síndrome de Klinefelter
d) Varón XX

SINDROME DE KLINEFELTER (SK)


Es la cromosomopatía más frecuente y la causa más habitual de hipogonadismo
hipergonadotrópico en el varón. Descrito en 1942 como un síndrome caracterizado por
hipogonadismo, testes pequeños y duros, azoospermia y ginecomastia. Se comprobó
posteriormente que el cuadro corresponde a una patología genética, cuya alteración
cromosómica más habitual es la presencia de un cromosoma X adicional, reflejando un
cariotipo 47 XXY, que representa el 80% de los casos de SK, pero se han descrito otras
variantes como mosaicismos: 47XXY/46XY, 47XXY/46XX, 47XXY/46XY/45X, etc. y
formas con más de un cromosoma X ó Y (48XXYY ó 47XXY/46XX/poliX). Esto hace que
estos sujetos presenten una cromatina de Barr con masa presente, siendo esto propio de
las mujeres por la presencia de 2 cromosomas X. La aparición de más de 2 cromosomas X,
ocasiona una patología que se diferencia del cuadro clásico de SK y se denomiona polisomía
X del varón: 48XXY, 49XXXXY.
La fórmula XXY se debe a una no disyunción del cromosoma X en la primera ó segunda
división meiótica, siendo más frecuente la aparición de SK en relación a la edad materna
más avanzada.
Su frecuencia se estima en 1/1000 recién nacidos varones, pero posiblemente sea más
elevada: en torno a 1/500 concepciones masculinas, y en varones con retraso mental
moderado la frecuencia aumenta a 8/1000 aproximadamente.
CLINICA: En general el diagnóstico es tardío, debido a la pobre expresividad clínica de
este síndrome en la infancia.

1) Talla alta: Es propia del síndrome de Klinefelter en la edad de la adolescencia, pero no lo


es en la época prepuberal de forma habitual. Las proporciones corporales son
eunucoides, con aumento desproporcionado de las extremidades inferiores y con
inversión del cociente: segmento superior/inferior. Cuando este cociente es inferior a 1
en la edad puberal, sugiere fuertemente la existencia de un SK si se asocia a un cuadro
de hipogonadismo aunque sea leve.

La velocidad de crecimiento suele aumentar a partir de los 4 – 5 años, y se adopta ya en la


infancia un cierto hábito eunucoide. La edad ósea es normal ó poco retrasada, coincidiendo
con el marcado retraso de otras variantes benignas de retraso puberal.

2) Alteraciones genitales: Los testes pequeños y duros en la edad prepuberal, pueden


presentarse en ocasiones, siendo por el contrario un hallazgo típico pasada la adolescencia.

La presencia de criptorquídea, micropene, hipospadias etc y a veces un fenotipo peculiar,


puede orientar al diagnóstico.
En la edad puberal, la atrofia testicular es un signo constante, los testes son pequeños y su
tamaño va involucionando con el tiempo, contrastando con un desarrollo normal del pene y
del relativamente normal desarrollo puberal. La histología del testículo demuestra atrofia
testicular con hialinización de los túbulos seminíferos.

3) Ginecomastia: Está presente en un elevado nº de casos (más del 60%), siendo el riesgo
de neoplasia mamaria 20 veces superior al de la población normal.

4) Desarrollo intelectual: la inteligencia en general suele ser normal, ó “bordeline”,


especialmente es el área verbal la más precoz y afectada, lo que condiciona
dificultades de aprendizaje y en general peor rendimiento escolar.

5) Alteraciones hormonales: En la edad prepuberal, tanto el nivel de gonadotrofinas como


de testosterona se encuentran en rango normal, tanto basal como tras estímulo. En la
pubertad se va estableciendo ya respuesta propia del hipogonadismo
hipergonadotrópico, especialmente el nivel de FSH que se eleva de forma llamativa.

6) Otras alteraciones asociadas: anomalías del tracto urinario, cúbito valgo etc. (se
recogen en la tabla adjunta). Especial relevancia por la ayuda que puede suponer para el
pediatra, es la relativamente aumentada incidencia de incontinentia pigmenti.

Son frecuentes las asociaciones del SK con neoplasias especialmente las de células
germinales mediastínicas, y con otros tumores en la edad adulta.
Bibliografía:

• Rossodivita A, Colabucci F. Short stature in a patient with Klinefelter syndrome


and growth hormone deficiency.Am J of Med Genet 1994; 49:244-6.

80.- Se trata de paciente masculino de 3 años de edad que acude a urgencias con una
historia de secreción nasal purulenta y de mal olor unilateral desde hace 5 días. El
diagnóstico más frecuente es:

a) Atresia de coanas unilateral.


b) Rinitis crónica por Rinovirus.
c) Cuerpo extraño intranasal.
d) Fibrosarcoma del correte nasal.

Una de las patologías más frecuentes que enfrenta el otorrinolaringólogo en su práctica


médica, es la presencia de cuerpos extraños animados o inanimados en vías aéreodigestivas
superiores. La mayoría de las veces su tratamiento (extracción) no reviste mayor
importancia sobre todo cuando no han sido manipulados por médicos de primer contacto,
pero en otras, se convierte en un procedimiento muy difícil de realizar, ya sea por las
complicaciones que se produjeron o por la presencia del mismo.

En la población infantil es más frecuente por la curiosidad propia de esa edad y la


tendencia a colocarse una variedad de elementos en las fosas nasales.
A veces existen ciertas dificultades en el diagnóstico, pero la tríada sintomática de
obstrucción nasal, rinorrea unilateral y halitosis, nos sugiere la posibilidad de un cuerpo
extraño hasta que no se demuestre lo contrario. Independientemente de su naturaleza
animada o inanimada, éstos pueden alojarse y/o impactarse en distintos niveles.
El tratamiento electivo es la extracción por vías naturales de los mismos; en los niños que
no colaboran es necesario recurrir a la anestesia general.

Bibliografía:

1. “Manual de Urgencias en Otorrinolaringología.” Manuel Tomás Barberán, Julio García-


Polo Alguacil, Guillermo Til Pérez.

También podría gustarte